*NURSING > HESI > NURS 420 Hesi Study Module 11 Comprehensive – Columbia College | NURS420 Hesi Study Module 11 Comp (All)

NURS 420 Hesi Study Module 11 Comprehensive – Columbia College | NURS420 Hesi Study Module 11 Comprehensive

Document Content and Description Below

NURS 420 Hesi Study Module 11 Comprehensive – Columbia College 1.ID: 383711499 Enalapril maleate (Vasotec) is prescribed for a hospitalized client. Which assessment does the nurse perform as a p... riority before administering the medication? Checking the client's blood pressure Correct Checking the client's peripheral pulses Checking the most recent potassium level Checking the client's intake-and-output record for the last 24 hours Rationale: Enalapril maleate is an angiotensin-converting enzyme (ACE) inhibitor used to treat hypertension. One common side effect is postural hypotension. Therefore the nurse would check the client’s blood pressure immediately before administering each dose. Checking the client’s peripheral pulses, the results of the most recent potassium level, and the intake and output for the previous 24 hours are not specifically associated with this mediation.

Test-Taking Strategy: Focus on the name of the medication and recall that medications that end in the letters “pril” are ACE inhibitors and that these medications are used to treat hypertension. This will direct you to the correct option. Review the action of enalapril maleate if you had difficulty with this question.

Reference: Lehne, R. (2010). Pharmacology for nursing care (7th ed., p. 477). St. Louis: Saunders.

Cognitive Ability: Analyzing

Client Needs: Physiological Integrity

Integrated Process: Nursing Process/Assessment

Content Area: Pharmacology Awarded 1.0 points out of 1.0 possible points. 2.ID: 383744011 A client is scheduled to undergo an upper gastrointestinal (GI) series, and the nurse provides instructions to the client about the test. Which statement by the client indicates a need for further instruction? "The test will take about 30 minutes." "I need to fast for 8 hours before the test." "I need to drink citrate of magnesia the night before the test and give myself a Fleet enema on the morning of the test." Correct "I need to take a laxative after the test is completed, because the liquid that I’ll have to drink for the test can be constipating." Rationale: An upper GI series involves visualization of the esophagus, duodenum, and upper jejunum by means of the use of a contrast medium. It involves swallowing a contrast medium (usually barium), which is administered in a flavored milkshake. Films are taken at intervals during the test, which takes about 30 minutes. No special preparation is necessary before a GI series, except that NPO status must be maintained for 8 hours before the test. After an upper GI series, the client is prescribed a laxative to hasten elimination of the barium. Barium that remains in the colon may become hard and difficult to expel, leading to fecal impaction.

Test-Taking Strategy: Use the process of elimination. Note the strategic words "need for further instruction." These words indicate a negative event query and the need to select the incorrect client statement. Focusing on the word "upper" in the name of the test will direct you to the correct option. Review preprocedure care for an upper GI series if you had difficulty with this question.

Reference: Ignatavicius, D., & Workman, M. (2010). Medical-surgical nursing: Patient-centered collaborative care (6th ed., p. 1226). St. Louis: Saunders.

Cognitive Ability: Evaluating

Client Needs: Physiological Integrity

Integrated Process: Teaching and Learning

Content Area: Adult Health/Gastrointestinal Awarded 0.0 points out of 1.0 possible points. 3.ID: 383705015 A nurse on the evening shift checks a physician's prescriptions and notes that the dose of a prescribed medication is higher than the normal dose. The nurse calls the physician's answering service and is told that the physician is off for the night and will be available in the morning. The nurse should: Call the nursing supervisor Ask the answering service to contact the on-call physician Correct Withhold the medication until the physician can be reached in the morning Administer the medication but consult the physician when he becomes available
  Rationale: The nurse has a duty to protect the client from harm. A nurse who believes that a physician’s prescription may be in error is responsible for clarifying the prescription before carrying it out. Therefore the nurse would not administer the medication; instead, the nurse would withhold the medication until the dose can be clarified. The nurse would not wait until the next morning to obtain clarification. It is premature to call the nursing supervisor.

Test-Taking Strategy: Use the process of elimination and your knowledge of the legal responsibilities of the nurse in regard to medication administration and physician’s prescriptions. Eliminate the options that are comparable or alike in that they avoid clarification of the prescription (administering the medication and holding the medication). To select from the remaining options, note that it is premature to call the nursing supervisor. Also note that the correct option is the only one that clarifies the prescription. Review legal responsibilities in regard to medication prescriptions if you had difficulty with this question.

Reference: Potter, P., & Perry, A. (2009). Fundamentals of nursing (7th ed., p. 336). St. Louis: Mosby.

Cognitive Ability: Applying

Client Needs: Safe and Effective Care Environment

Integrated Process: Nursing Process/Implementation

Content Area: Leadership and Management Awarded 0.0 points out of 1.0 possible points. 4.ID: 383708500 An emergency department (ED) nurse is monitoring a client with suspected acute myocardial infarction (MI) who is awaiting transfer to the coronary intensive care unit. The nurse notes the sudden onset of premature ventricular contractions (PVCs) on the monitor, checks the client's carotid pulse, and determines that the PVCs are not resulting in perfusion. The appropriate action by the nurse is: Documenting the findings Asking the ED physician to check the client Correct Continuing to monitor the client's cardiac status Informing the client that PVCs are expected after an MI
  Rationale: PVCs are a result of increased irritability of ventricular cells. Peripheral pulses may be absent or diminished with the PVCs themselves because the decreased stroke volume of the premature beats may in turn decrease peripheral perfusion. Because other rhythms also cause widened QRS complexes, it is essential that the nurse determine whether the premature beats are resulting in perfusion of the extremities. This is done by palpating the carotid, brachial, or femoral artery while observing the monitor for widened complexes or by auscultating for apical heart sounds. In the situation of acute MI, PVCs may be considered warning dysrhythmias, possibly heralding the onset of ventricular tachycardia or ventricular fibrillation. Therefore the nurse would not tell the client that the PVCs are expected. Although the nurse will continue to monitor the client and document the findings, these are not the most appropriate actions of those provided. The most appropriate action would be to ask the ED physician to check the client.

Test-Taking Strategy: Use the process of elimination. Recalling the significance of PVCs after acute MI and noting the strategic words "not perfusing" will direct you to the correct option. Review the significance of PVCs after acute MI if you had difficulty with this question.

Reference: Ignatavicius, D., & Workman, M. (2010). Medical-surgical nursing: Patient-centered collaborative care (6th ed., pp. 747, 748). St. Louis: Saunders.

Cognitive Ability: Applying

Client Needs: Physiological Integrity

Integrated Process: Nursing Process/Implementation

Content Area: Critical Care Awarded 0.0 points out of 1.0 possible points. 5.ID: 383704545 NPO status is imposed 8 hours before the procedure on a client scheduled to undergo electroconvulsive therapy (ECT) at 1 p.m. On the morning of the procedure, the nurse checks the client's record and notes that the client routinely takes an oral antihypertensive medication each morning. The nurse should: Administer the antihypertensive with a small sip of water Correct Withhold the antihypertensive and administer it at bedtime Administer the medication by way of the intravenous (IV) route Hold the antihypertensive and resume its administration on the day after the ECT Rationale: General anesthesia is required for ECT, so NPO status is imposed for 6 to 8 hours before treatment to help prevent aspiration. Exceptions include clients who routinely receive cardiac medications, antihypertensive agents, or histamine (H2) blockers, which should be administered several hours before treatment with a small sip of water. Withholding the antihypertensive and administering it at bedtime and withholding the antihypertensive and resuming administration on the day after the ECT are incorrect actions, because antihypertensives must be administered on time; otherwise, the risk for rebound hypertension exists. The nurse would not administer a medication by way of a route that has not been prescribed.

Test-Taking Strategy: Use the process of elimination. Use your knowledge of the principles of medication administration to help eliminate the option that involves administering the medication by way of a route other than the prescribed one. Recalling that antihypertensives must be administered on a regular schedule will assist you in eliminating the options that involve withholding the medication. Review preprocedure care for the client scheduled for ECT if you had difficulty with this question.

Reference: Stuart, G., & Laraia, M. (2009). Principles and practice of psychiatric nursing (9th ed., p. 539). St Louis: Mosby.

Cognitive Ability: Applying

Client Needs: Physiological Integrity

Integrated Process: Nursing Process/Implementation

Content Area: Mental Health Awarded 0.0 points out of 1.0 possible points. 6.ID: 383706660 A client who recently underwent coronary artery bypass graft surgery comes to the physician's office for a follow-up visit. On assessment, the client tells the nurse that he is feeling depressed. Which response by the nurse is therapeutic? "Tell me more about what you’re feeling." Correct "That’s a normal response after this type of surgery." "It will take time, but, I promise you, you will get over this depression." "Every client who has this surgery feels the same way for about a month." Rationale: When a client expresses feelings of depression, it is extremely important for the nurse to further explore these feelings with the client. In stating, "This is a normal response after this type of surgery" the nurse provides false reassurance and avoids addressing the client’s feelings. "It will take time, but, I promise you, you will get over the depression" is also a false reassurance, and it does not encourage the expression of feelings. "Every client who has this surgery feels the same way for about a month" is a generalization that avoids the client’s feelings.

Test-Taking Strategy: Use your knowledge of therapeutic communication techniques. Eliminate the options that are nontherapeutic and do not encourage the client to express feelings. Remember to always focus on the client’s feelings. Review therapeutic communication techniques if you had difficulty with this question.

Reference: Stuart, G., & Laraia, M. (2009). Principles and practice of psychiatric nursing (9th ed., pp. 27-31, 301). St Louis: Mosby.

Cognitive Ability: Applying

Client Needs: Psychosocial Integrity

Integrated Process: Communication and Documentation

Content Area: Adult Health/Cardiovascular Awarded 0.0 points out of 1.0 possible points. 7.ID: 383705009 A client in labor experiences spontaneous rupture of the membranes. The nurse immediately counts the fetal heart rate (FHR) for 1 full minute and then checks the amniotic fluid. The nurse notes that the fluid is yellow and has a strong odor. Which of the following actions should be the nurse’s priority? Contacting the physician Correct Documenting the findings Checking the fluid for protein Continuing to monitor the client and the FHR Rationale: The FHR is assessed for at least 1 minute when the membranes rupture. The nurse also checks the quantity, color, and odor of the amniotic fluid. The fluid should be clear (often with bits of vernix) and have a mild odor. Fluid with a foul or strong odor, cloudy appearance, or yellow coloration suggests chorioamnionitis and warrants notifying the physician. A large amount of vernix in the fluid suggests that the fetus is preterm. Greenish, meconium-stained fluid may be seen in cases of postterm gestation or placental insufficiency. Checking the fluid for protein is not associated with the data in the question. Although the nurse would continue to monitor the client and the FHR and would document the findings, contacting the physician is the priority.

Test-Taking Strategy: Focus on the data in the question and note the strategic word "priority." Noting the words "yellow and has a strong odor" will direct you to the correct option. Review the expected findings after rupture of the membranes if you had difficulty with this question.

Reference: Lowdermilk, D., Perry, S., & Cashion, K., (2010). Maternity nursing (8th ed., pp. 689, 690) St. Louis: Mosby.

Cognitive Ability: Applying

Client Needs: Physiological Integrity

Integrated Process: Nursing Process/Implementation

Content Area: Maternity/Intrapartum Awarded 0.0 points out of 1.0 possible points. 8.ID: 383705011 A nurse has assisted a physician in inserting a central venous access device into a client with a diagnosis of severe malnutrition who will be receiving parenteral nutrition (PN). After insertion of the catheter, the nurse immediately plans to: Call the radiography department to obtain a chest x-ray Correct Check the client's blood glucose level to serve as a baseline measurement Hang the prescribed bag of PN and start the infusion at the prescribed rate Infuse normal saline solution through the catheter at a rate of 100 mL/hr to maintain patency Rationale: One major complication associated with central venous catheter placement is pneumothorax, which may result from accidental puncture of the lung. After the catheter has been placed but before it is used for infusions, its placement must be checked with an x-ray. Hanging the prescribed bag of PN and starting the infusion at the prescribed rate and infusing normal saline solution through the catheter at a rate of 100 mL/hr to maintain patency are all incorrect because they could result in the infusion of solution into a lung if a pneumothorax is present. Although the nurse may obtain a blood glucose measurement to serve as a baseline, this action is not the priority.

Test-Taking Strategy: Note the strategic word “immediately.” Use the ABCs — airway, breathing, and circulation. Recalling that pneumothorax is a complication of the insertion of this type of catheter will direct you to the correct option. Review care after central venous catheter placement if you have difficulty with this question.

Reference: Ignatavicius, D., & Workman, M. (2010). Medical-surgical nursing: Patient-centered collaborative care (6th ed., pp. 217, 1400). St. Louis: Saunders.

Cognitive Ability: Applying

Client Needs: Physiological Integrity

Integrated Process: Nursing Process/Implementation

Content Area: Intravenous therapy Awarded 0.0 points out of 1.0 possible points. 9.ID: 383705041 A rape victim being treated in the emergency department says to the nurse, "I’m really worried that I’ve got HIV now." What is the appropriate response by the nurse? "HIV is rarely an issue in rape victims." "Every rape victim is concerned about HIV." "You’re more likely to get pregnant than to contract HIV." "Let's talk about the information that you need to determine your risk of contracting HIV." Correct Rationale: HIV is a concern of rape victims. Such concern should always be addressed, and the victim should be given the information needed to evaluate his or her risk. Pregnancy may occur as a result of rape, and pregnancy prophylaxis can be offered in the emergency department or during follow-up, once the results of a pregnancy test have been obtained. However, stating, “You’re more likely to get pregnant than to contract HIV” avoids the client’s concern. Similarly, "HIV is rarely an issue in rape victims” and "Every rape victim is concerned about HIV" are generalized responses that avoid the client’s concern.

Test-Taking Strategy: Use your knowledge of therapeutic communication techniques. Eliminate the options that are comparable or alike in that the nurse avoids addressing the client’s concern. Review the psychosocial issues of the rape victim if you had difficulty with this question.

References: Stuart, G., & Laraia, M. (2009). Principles and practice of psychiatric nursing (9th ed., pp. 27-31). St Louis: Mosby.
Varcarolis, E., & Halter, M. (2009). Essentials of psychiatric mental health nursing: A communication approach to evidence-based care (p. 408). St. Louis: Saunders.

Cognitive Ability: Applying

Client Needs: Psychosocial Integrity

Integrated Process: Communication and Documentation

Content Area: Mental Health Awarded 0.0 points out of 1.0 possible points. 10.ID: 383703603 A client is taking prescribed ibuprofen (Motrin), 300 mg orally four times daily, to relieve joint pain resulting from rheumatoid arthritis. The client tells the nurse that the medication is causing nausea and indigestion. The nurse should tell the client to: Contact the physician Stop taking the medication Take the medication with food Correct Take the medication twice a day instead of four times Rationale: Ibuprofen is a nonsteroidal antiinflammatory medication. Side effects include nausea (with or without vomiting) and dyspepsia (heartburn, indigestion, or epigastric pain). If gastrointestinal distress occurs, the client should be instructed to take the medication with milk or food. The nurse would not instruct the client to stop the medication or instruct the client to adjust the dosage of a prescribed medication; these actions are not within the legal scope of the role of the nurse. Contacting the physician is premature, because the client’s complaints are side effects that occasionally occur and can be relieved by taking the medication with milk or food.

Test-Taking Strategy: Use guidelines related to medication administration to assist you to eliminate the options that indicate to stop the medication or adjust the prescribed dose. To select from the remaining options, think about the side effects of the medication. Review the side effects of ibuprofen and the measures to relieve them if you had difficulty with this question.

Reference: Lehne, R. (2010). Pharmacology for nursing care (7th ed., p. 844). St. Louis: Saunders.

Cognitive Ability: Applying

Client Needs: Physiological Integrity

Integrated Process: Nursing Process/Implementation

Content Area: Pharmacology Awarded 0.0 points out of 1.0 possible points. 11.ID: 383704532 A client's oral intake of liquids includes 120 mL on the night shift, 800 mL on the day shift, and 650 mL on the evening shift. The client is receiving an intravenous (IV) antibiotic every 12 hours, diluted in 50 mL of normal saline solution. The nurse empties 700 mL of urine from the client's Foley catheter at the end of the day shift. Thereafter, 500 mL of urine is emptied at the end of the evening shift and 325 mL at the end of the night shift. Nasogastric tube drainage totals 155 mL for the 24-hour period, and the total drainage from the Jackson-Pratt device is 175 mL. What is the client's total intake during the 24-hour period? Type your answer in the space provided.

Answer: ________mL Incorrect Correct Responses: "1670" Awarded 0.0 out of 1.0 possible points. 12.ID: 383704537 Lorazepam (Ativan) 1 mg by way of intravenous (IV) injection (IV push) is prescribed for a client for the management of anxiety. The nurse prepares the medication as prescribed and administers the medication over a period of: 3 minutes Correct 10 seconds 15 seconds 30 minutes
  Rationale: Lorazepam is a benzodiazepine. When administered by IV injection, each 2 mg or fraction thereof is administered over a period of 1 to 5 minutes. Ten seconds and 30 seconds are brief periods. Thirty minutes is a lengthy period.

Test-Taking Strategy: Focus on the subject, administration of a medication by way of IV injection. Eliminate the options that indicate delivery times of 10 and 15 seconds, because these periods are very brief. Next eliminate the option of 30 minutes because of its lengthiness. Review the procedure for administering lorazepam by way of IV injection if you had difficulty with this question.

Reference: Gahart, B., & Nazareno, A. (2010). Intravenous medications (26th ed., p. 837). St. Louis: Mosby.

Cognitive Ability: Applying

Client Needs: Physiological Integrity

Integrated Process: Nursing Process/Implementation

Content Area: Pharmacology Awarded 0.0 points out of 1.0 possible points. 13.ID: 383706090 A nurse, conducting an assessment of a client being seen in the clinic for symptoms of a sinus infection, asks the client about medications that he is taking. The client tells the nurse that he is taking nefazodone hydrochloride (Serzone). On the basis of this information, the nurse determines that the client most likely has a history of: Depression Correct Diabetes mellitus Hyperthyroidism Coronary artery disease Rationale: Nefazodone hydrochloride is an antidepressant used as maintenance therapy to prevent relapse of an acute depression. Diabetes mellitus, hypethyroidism, and coronary artery disease are not treated with this medication.

Test-Taking Strategy: Knowledge regarding the use of this medication is required to answer this question correctly. Recalling that nefazodone hydrochloride is an antidepressant will direct you to the correct option. Review this medication if you had difficulty with this question.

Reference: Lehne, R. (2010). Pharmacology for nursing care (7th ed., p. 352). St. Louis: Saunders.

Cognitive Ability: Analyzing

Client Needs: Physiological Integrity

Integrated Process: Nursing Process/Assessment

Content Area: Pharmacology Awarded 0.0 points out of 1.0 possible points. 14.ID: 383707982 Phenelzine sulfate (Nardil) is prescribed for a client with depression. The nurse provides information to the client about the adverse effects of the medication and tells the client to contact the physician immediately if she experiences: Dry mouth Restlessness Feelings of depression Neck stiffness or soreness Correct Rationale: Phenelzine sulfate, a monoamine oxidase inhibitor (MAOI), is an antidepressant and is used to treat depression. Hypertensive crisis, an adverse effect of this medication, is characterized by hypertension, frontally radiating occipital headache, neck stiffness and soreness, nausea, vomiting, sweating, fever and chills, clammy skin, dilated pupils, and palpitations. Tachycardia, bradycardia, and constricting chest pain may also be present. The client is taught to be alert to any occipital headache radiating frontally and neck stiffness or soreness, which could be the first signs of a hypertensive crisis. Dry mouth and restlessness are common side effects of the medication.

Test-Taking Strategy: Use the process of elimination and focus on the subject, the symptoms that should prompt the client to contact the physician immediately. Recalling that the medication is an MAOI and the common and adverse effects of the medication will help direct you to the correct option. Review the side effects and adverse effects of this medication if you had difficulty with this question.

Reference: Lehne, R. (2010). Pharmacology for nursing care (7th ed., p. 358). St. Louis: Saunders.

Cognitive Ability: Applying

Client Needs: Physiological Integrity

Integrated Process: Teaching and Learning

Content Area: Mental Health Awarded 0.0 points out of 1.0 possible points. 15.ID: 383703621 Risperidone (Risperdal) is prescribed for a client hospitalized in the mental health unit for the treatment of a psychotic disorder. Which finding in the client’s medical record would prompt the nurse to contact the prescribing physician before administering the medication? The client has a history of cataracts. The client has a history of hypothyroidism. The client takes a prescribed antihypertensive. Correct The client is allergic to acetylsalicylic acid (aspirin). Rationale: Risperidone is an antipsychotic medication. Contraindications to the use of risperidone include cardiac disorders, cerebrovascular disease, dehydration, hypovolemia, and therapy with antihypertensive agents. Risperidone is used with caution in clients with a history of seizures. History of cataracts, hypothyroidism, or allergy to aspirin does not affect the administration of this medication.

Test-Taking Strategy: Knowledge of the contraindications to the use of risperidone is required to answer this question correctly. It is important to remember that one such contraindication is therapy with an antihypertensive medication. If you are unfamiliar with the contraindications to the use of risperidone, review this content.

Reference: Lehne, R. (2010). Pharmacology for nursing care (7th ed., pp. 334, 335). St. Louis: Saunders.

Cognitive Ability: Analyzing

Client Needs: Physiological Integrity

Integrated Process: Nursing Process/Implementation

Content Area: Pharmacology Awarded 0.0 points out of 1.0 possible points. 16.ID: 383707984 A client who has been undergoing long-term therapy with an antipsychotic medication is admitted to the inpatient mental health unit. Which of the following findings does the nurse, knowing that long-term use of an antipsychotic medication can cause tardive dyskinesia, monitor in the client? Fever Diarrhea Hypertension Tongue protrusion Correct Rationale: Tardive dyskinesia is a severe reaction associated with long-term use of antipsychotic medications. The clinical manifestations include abnormal movements (dyskinesia) and involuntary movements of the mouth, tongue (“flycatcher tongue”), and face. In its most severe form, tardive dyskinesia involves the fingers, arms, trunk, and respiratory muscles. When this occurs, the medication is discontinued. Fever, diarrhea, and hypertension are not characteristics of tardive dyskinesia.

Test-Taking Strategy: Knowledge of the clinical manifestations associated with tardive dyskinesia is needed to answer this question correctly. Recalling that the clinical manifestations of tardive dyskinesia include abnormal movements and involuntary movements will direct you to the correct option. If you had difficulty with this question, review the characteristics of tardive dyskinesia.

Reference: Stuart, G., & Laraia, M. (2009). Principles and practice of psychiatric nursing (9th ed., p. 528). St. Louis: Mosby.

Cognitive Ability: Analyzing

Client Needs: Physiological Integrity

Integrated Process: Nursing Process/Assessment

Content Area: Pharmacology Awarded 0.0 points out of 1.0 possible points. 17.ID: 383706064 A nurse is reviewing the record of a client scheduled for electroconvulsive therapy (ECT). Which of the following diagnoses, if noted on the client's record, would indicate a need to contact the physician who is scheduled to perform the ECT? Recent stroke Correct Hypothyroidism History of glaucoma Peripheral vascular disease Rationale: Several conditions pose risks in the client scheduled for ECT. Among them are recent myocardial infarction or stroke and cerebrovascular malformations or intracranial lesions. Hypothyroidism, glaucoma, and peripheral vascular disease are not contraindications to this treatment.

Test-Taking Strategy: Knowledge regarding the risks associated with ECT is required to answer this question correctly. Note the word "recent" in the correct option, which should help recognize the correct option. Review contraindications to ECT if you had difficulty with this question.

Reference: Stuart, G., & Laraia, M. (2009). Principles and practice of psychiatric nursing (9th ed., p. 536). St. Louis: Mosby.

Cognitive Ability: Analyzing

Client Needs: Physiological Integrity

Integrated Process: Nursing Process/Assessment

Content Area: Mental Health Awarded 0.0 points out of 1.0 possible points. 18.ID: 383712440 A client scheduled for suprapubic prostatectomy has listened to the surgeon's explanation of the surgery. The client later asks the nurse to explain again how the prostate is going to be removed. The nurse tells the client that the prostate will be removed through: A lower abdominal incision Correct An upper abdominal incision An incision made in the perineal area The urethra, with the use of a cutting wire Rationale: A lower abdominal incision is used in suprapubic or retropubic prostatectomy. An upper abdominal incision is not used to remove the prostate. An incision between the scrotum and anus is made when a perineal prostatectomy is performed. Transurethral resection is performed through the urethra; an instrument called a resectoscope is used to cut the tissue by means of a high-frequency current.

Test-Taking Strategy: To answer this question accurately, you must be familiar with this surgery and how it is performed. Focusing on the data in the question and noting the word "suprapubic" will direct you to the correct option. Review the procedure for performing a suprapubic prostatectomy if you had difficulty with this question.

Reference: Black, J., & Hawks, J. (2009). Medical-surgical nursing: Clinical management for positive outcomes (8th ed., p. 880). St. Louis: Saunders.

Cognitive Ability: Understanding

Client Needs: Physiological Integrity

Integrated Process: Nursing Process/Implementation

Content Area: Adult Health/Renal Awarded 0.0 points out of 1.0 possible points. 19.ID: 383707954 A nurse is preparing a poster for a health fair booth promoting primary prevention of skin cancer. Which of the following recommendations does the nurse include on the poster? Select all that apply. Seek medical advice if you find a skin lesion. Correct Use sunscreen with a low sun protection factor (SPF). Avoid sun exposure before 10 a.m. and after 4 p.m. Wear a hat, opaque clothing, and sunglasses when out in the sun. Correct Examine the body every 6 months for possibly cancerous or precancerous lesions. Rationale: Measures to prevent skin cancer include avoiding sun exposure between 10 a.m. and 4 p.m.; using sunscreen with a high SPF; wearing a hat, opaque clothing, and sunglasses when out in the sun; and examining the body every month for possibly cancerous or precancerous lesions. The client should also seek medical advice if any changes in a skin lesion are noted.

Test-Taking Strategy: Focus on the subject, the prevention of skin cancer. Read each option carefully. Eliminate the option that includes the words “low sun protection factor.” Next eliminate the option that includes “every 6 months.” To select from the remaining options, recall that the skin should be protected from the sun even more carefully between the hours of 10 a.m. and 4 p.m. Review the risk factors associated with skin cancer if you had difficulty with this question.

Reference: Ignatavicius, D., & Workman, M. (2010). Medical-surgical nursing: Patient-centered collaborative care (6th ed., p. 511). St. Louis: Saunders.

Cognitive Ability: Applying

Client Needs: Health Promotion and Maintenance

Integrated Process: Teaching and Learning

Content Area: Adult Health/Oncology Awarded 0.0 points out of 1.0 possible points. 20.ID: 383702969 A nurse reviewing the medical record of a client with a diagnosis of infiltrating ductal carcinoma of the breast notes documentation of the presence of peau d'orange skin. On the basis of this notation, which finding would the nurse expect to note on assessment of the client’s breast? Correct Rationale: Peau d’orange (French for “orange peel”) is the term used to describe skin dimpling, resembling the skin of an orange, at the location of a breast mass. This change, along with increased vascularity, nipple retraction, or ulceration, may indicate advanced disease. Erythema, or reddening, of the breast indicates inflammation such as that resulting from cellulitis or a breast abscess. Paget’s disease is a rare type of breast cancer that is manifested as a red, scaly nipple; discharge; crusting lasting more than a few weeks. In nipple retraction, the nipple is pointed or pulled in an abnormal direction. It is suggestive of malignancy.

Test-Taking Strategy: Focus on the subject, peau d’orange. Recalling the appearance of this characteristic will help you find the correct option. Note the word “orange” and the appearance of the breast in the correct option. Review the characteristics of breast cancer if you had difficulty with this question.

Reference: Ignatavicius, D., & Workman, M. (2010). Medical-surgical nursing: Patient-centered collaborative care (6th ed., pp. 1663, 1664). St. Louis: Saunders.

Cognitive Ability: Analyzing

Client Needs: Physiological Integrity

Integrated Process: Nursing Process/Assessment

Content Area: Adult Health/Oncology Awarded 0.0 points out of 1.0 possible points. 21.ID: 383702971 The mother of an adolescent with type 1 diabetes mellitus tells the nurse that her child is a member of the school soccer team and expresses concern about her child's participation in sports. The nurse, after providing information to the mother about diet, exercise, insulin, and blood glucose control, tells the mother: To always administer less insulin on the days of soccer games That it is best not to encourage the child to participate in sports activities That the child should eat a carbohydrate snack about a half-hour before each soccer game Correct To administer additional insulin before a soccer game if the blood glucose level is 240 mg/dL or higher and ketones are present Rationale: The child with diabetes mellitus who is active in sports requires additional food intake in the form of a carbohydrate snack about a half-hour before the anticipated activity. Additional food will need to be consumed, often as frequently as every 45 minutes to 1 hour, during prolonged periods of activity. If the blood glucose level is increased (240 mg/dL or more) and ketones are present before planned exercise, the activity should be postponed until the blood glucose has been controlled. Moderate to high ketone values should be reported to the physician. There is no reason for the child to avoid participating in sports.

Test-Taking Strategy: Focus on the data in the question. First, eliminate the option that contains the closed-ended word "always." Next, recall the importance of the child’s participation in activities, which will help you eliminate the option involving avoidance of activities. To select from the remaining options, recall the association of insulin, diet, and exercise in the control of the blood glucose level, which will direct you to the correct option. Review the diet, exercise, and medication in the treatment of diabetes mellitus if you had difficulty with this question.

Reference: McKinney, E., James, S., Murray, S., & Ashwill, J. (2009). Maternal-child nursing (3rd ed., pp. 1444, 1447). St. Louis: Elsevier.

Cognitive Ability: Applying

Client Needs: Physiological Integrity

Integrated Process: Teaching and Learning

Content Area: Child Health – Metabolic/Endocrine Awarded 0.0 points out of 1.0 possible points. 22.ID: 383705039 A client with chronic renal failure who will require dialysis three times a week for the rest of his life says to the nurse, "Why should I even bother to watch what I eat and drink? It doesn't really matter what I do if I’m never going to get better!" On the basis of the client's statement, the nurse determines that the client is experiencing which problem? Anxiety Powerlessness Correct Ineffective coping Disturbed body image Rationale: Powerlessness is present when a client believes that he or she has no control over the situation or that his or her actions will not affect an outcome in any significant way. Anxiety is a vague uneasy feeling of apprehension. Some factors in anxiety include a threat or perceived threat to physical or emotional integrity or self-concept, changes in role function, and a threat to or change in socioeconomic status. Ineffective coping is present when the client exhibits impaired adaptive abilities or behaviors in meeting the demands or roles expected. Disturbed body image is diagnosed when there is an alteration in the way the client perceives his or her own body image.

Test-Taking Strategy: Use the process of elimination. Note that the statement "It doesn’t really matter what I do...." implies that the client has a sense of no control of the situation. This will direct you to the correct option. Review the defining characteristics of powerlessness if you had difficulty with this question.

References: Ignatavicius, D., & Workman, M. (2010). Medical-surgical nursing: Patient-centered collaborative care (6th ed., p. 1635). St. Louis: Saunders.
Swearingen, P. (2008). All-in-one care planning resource: Medical-surgical, pediatric, maternity & psychiatric nursing care plans (2nd ed., pp. 77, 78). St. Louis: Mosby.

Cognitive Ability: Analyzing

Client Needs: Psychosocial Integrity

Integrated Process: Nursing Process/Analysis

Content Area: Adult Health/Renal Awarded 0.0 points out of 1.0 possible points. 23.ID: 383710055 A nurse is providing morning care to a client in end-stage renal failure. The client is reluctant to talk and shows little interest in participating in hygiene care. Which statement by the nurse would be therapeutic? "What are your feelings right now?" Correct "Why don't you feel like washing up?" "You aren’t talking today. Cat got your tongue?" "You need to get yourself cleaned up. You have company coming today." Rationale: Asking, "What are your feelings right now?" encourages the client to identify his or her emotions or feelings, which is a therapeutic communication technique. In stating, "Why don't you feel like washing up?" the nurse is requesting an explanation of feelings and behaviors for which the client may not know the reason. Requesting an explanation is a nontherapeutic communication technique. "You aren’t talking today. Cat got your tongue?" is a nontherapeutic cliché. The statement "You need to get yourself cleaned up. You have company coming today" is demanding, demeaning to the client, and nontherapeutic.

Test-Taking Strategy: Use your knowledge of therapeutic communication techniques to answer the question. Remembering to focus on the client’s feelings will direct you to the correct option. Review therapeutic communication techniques if you had difficulty with this question.

References: Ignatavicius, D., & Workman, M. (2010). Medical-surgical nursing: Patient-centered collaborative care (6th ed., pp. 1600, 1601). St. Louis: Saunders.
Potter, P., & Perry, A. (2009). Fundamentals of nursing (7th ed., pp. 352-357). St. Louis: Mosby.

Cognitive Ability: Applying

Client Needs: Psychosocial Integrity

Integrated Process: Communication and Documentation

Content Area: Adult Health/Renal Awarded 0.0 points out of 1.0 possible points. 24.ID: 383704543 Empyema develops in a client with an infected pleural effusion, and the nurse prepares the client for thoracentesis. What characteristics of the fluid removed during thoracentesis should the nurse, assisting the physician with the procedure, expect to note? Clear and yellow Thick and opaque Correct White and odorless Clear, with a foul odor Rationale: Empyema is the accumulation of pus in the pleural space. Empyema fluid is thick, opaque, exudative, and intensely foul-smelling. Clear and yellow, white and odorless, and clear and foul-smelling are incorrect descriptions of the fluid that occurs in this disorder.

Test-Taking Strategy: Use the process of elimination. Focusing on the words "empyema" and "infected pleural effusion" will assist in directing you to the correct option. In this disorder, the fluid is not clear or odorless. Review the characteristics of empyema if you had difficulty with this question.

Reference: Ignatavicius, D., & Workman, M. (2010). Medical-surgical nursing: Patient-centered collaborative care (6th ed., p. 674). St. Louis: Saunders.

Cognitive Ability: Analyzing

Client Needs: Physiological Integrity

Integrated Process: Nursing Process/Analysis

Content Area: Adult Health/Respiratory Awarded 0.0 points out of 1.0 possible points. 25.ID: 383706054 An emergency department nurse is told that a client with carbon monoxide poisoning resulting from a suicide attempt is being brought to the hospital by emergency medical services. Which intervention will the nurse carry out as a priority upon arrival of the client? Administering 100% oxygen Correct Having a crisis counselor available Instituting suicide precautions for the client Obtaining blood for determination of the client’s carboxyhemoglobin level Rationale: A client with carbon monoxide poisoning is treated with inhalation of 100% oxygen to shorten the half-life of carbon monoxide to around an hour. Hyperbaric oxygen may be required to reduce the half-life to minutes by forcing the carbon monoxide off the hemoglobin molecule. Because the poisoning occurred as a result of a suicide attempt, a crisis counselor should be consulted, but this is not the priority. Suicide precautions should be instituted once emergency interventions have been completed and the client has been admitted to the hospital. The diagnosis is confirmed with a measurement of the carboxyhemoglobin level in the client’s blood. Obtaining a blood specimen in which measure the carboxyhemoglobin level is a priority; however, the nurse would immediately administer 100% oxygen to the client.

Test-Taking Strategy: Note the strategic word "priority" and use the ABCs — airway, breathing, and circulation. This will direct you to the correct option. Review care of the client with carbon monoxide poisoning and care after a suicide attempt if you had difficulty with this question.

Reference: Ignatavicius, D., & Workman, M. (2010). Medical-surgical nursing: Patient-centered collaborative care (6th ed., pp. 529-530). St. Louis: Saunders.

Cognitive Ability: Applying

Client Needs: Physiological Integrity

Integrated Process: Nursing Process/Implementation

Content Area: Critical Care Awarded 0.0 points out of 1.0 possible points. 26.ID: 383703691 A nurse is caring for a client with sarcoidosis. The client is upset because he has missed work and worried about how he will care financially for his wife and three small children. On the basis of the client's concern, which problem does the nurse identify? Anxiety Correct Powerlessness Disruption of thought processes Inability to maintain health Rationale: Anxiety is a vague, uneasy feeling of apprehension. Some related factors include a threat or perceived threat to physical or emotional integrity or self-concept, changes in function in one’s role, and threats to or changes in socioeconomic status. The client experiencing powerlessness expresses feelings of having no control over a situation or outcome. Disruption of thought processes involves disturbance of cognitive abilities or thought. Inability to maintain health is being incapable of seeking out help needed to maintain health.

Test-Taking Strategy: Use the process of elimination. Focusing on the data in the question and noting the words “upset” and “worried” will direct you to the correct option. Review the defining characteristics of anxiety if you had difficulty with this question.

Reference: Ignatavicius, D., & Workman, M. (2010). Medical-surgical nursing: Patient-centered collaborative care (6th ed., pp. 638, 639). St. Louis: Saunders.

Cognitive Ability: Analyzing

Client Needs: Psychosocial Integrity

Integrated Process: Nursing Process/Analysis

Content Area: Adult Health/Respiratory Awarded 0.0 points out of 1.0 possible points. 27.ID: 383709211 A nurse, performing an assessment of a client who has been admitted to the hospital with suspected silicosis, is gathering both subjective and objective data. Which question by the nurse would elicit data specific to the cause of this disorder? "Do you chew tobacco?" "Do you smoke cigarettes?" "Have you ever worked in a mine?" Correct "Are you frequently exposed to paint products?" Rationale: Silicosis is a chronic fibrotic disease of the lungs caused by the inhalation of free crystalline silica dust over a long period. Mining and quarrying are each associated with a high incidence of silicosis. Hazardous exposure to silica dust also occurs in foundry work, tunneling, sandblasting, pottery-making, stone masonry, and the manufacture of glass, tile, and bricks. The finely ground silica used in soaps, polishes, and filters also presents a risk. The assessment questions noted in the other options are unrelated to the cause of silicosis.

Test-Taking Strategy: Use the process of elimination. Eliminate the options that are comparable or alike in that they are related to the use of tobacco. To select correctly from the remaining options, it is necessary to recall that silicosis is caused by exposure to dust. Review the causes of silicosis if you had difficulty with this question.

Reference: Ignatavicius, D., & Workman, M. (2010). Medical-surgical nursing: Patient-centered collaborative care (6th ed., p. 640). St. Louis: Saunders.

Cognitive Ability: Analyzing

Client Needs: Physiological Integrity

Integrated Process: Nursing Process/Assessment

Content Area: Adult Health/Respiratory Awarded 0.0 points out of 1.0 possible points. 28.ID: 383706004 A physician prescribes a dose of morphine sulfate 2.5 mg stat to be administered intravenously to a client in pain. The nurse preparing the medication notes that the label on the vial of morphine sulfate solution for injection reads “4 mg/mL.” How many milliliters (mL) must the nurse draw into a syringe for administration to the client? Type the answer in the space provided.

Answer: _____mL Incorrect Correct Responses: "0.625, .625, 1" Awarded 0.0 out of 1.0 possible points. 29.ID: 383708582 A client undergoing therapy with carbidopa/levodopa (Sinemet) calls the nurse at the clinic and reports that his urine has become darker since he started taking the medication. The nurse should tell the client: To call his physician That he needs to drink more fluids That this is an occasional side effect of the medication Correct That this may be a sign of developing toxicity of the medication Rationale: Carbidopa/levodopa, an antiparkinson agent, may cause darkening of the urine or sweat. The client should be reassured that this is a harmless side effect of the medication and that the medication’s use should be continued. Although fluid intake is important, telling the client that he needs to drink more fluid is incorrect and unnecessary. Telling the client that the darkening of his urine may signal developing medication toxicity is incorrect and might alarm the client unnecessarily. There is no need for the client to call the physician.

Test-Taking Strategy: Use the process of elimination. Eliminate first the options that are comparable or alike (i.e., if toxicity is developing, the physician would need to be notified). To select from the remaining options, recall the side effects of carbidopa/levodopa therapy, which will direct you to the correct option. Review the side effects of this medication if you had difficulty with this question.

Reference: Kee, J., Hayes, E., & McCuistion, L. (2009). Pharmacology: A nursing process approach (6th ed., p. 336). Philadelphia: Saunders.

Cognitive Ability: Applying

Client Needs: Physiological Integrity

Integrated Process: Nursing Process/Implementation

Content Area: Pharmacology Awarded 0.0 points out of 1.0 possible points. 30.ID: 383703617 A client with myasthenia gravis is taking neostigmine bromide (Prostigmin). The nurse determines that the client is gaining a therapeutic effect from the medication after noting: Bradycardia Increased heart rate Decreased blood pressure Improved swallowing function Correct Rationale: Neostigmine bromide, a cholinergic medication that prevents the destruction of acetylcholine, is used to treat myanthenia gravis. The nurse would monitor the client for a therapeutic response, which includes increased muscle strength, an easing of fatigue, and improved chewing and swallowing function. Bradycardia, increased heart rate, and decreased blood pressure are signs of an adverse reaction to the medication.

Test-Taking Strategy: Note the strategic words "a therapeutic effect." Focusing on the client’s diagnosis and recalling the pathophysiology associated with this disorder will direct you to the correct option. Review the therapeutic and adverse effects of this medication if you had difficulty with this question.

Reference: Lehne, R. (2010). Pharmacology for nursing care (7th ed., pp. 133, 137, 138). St. Louis: Saunders.

Cognitive Ability: Evaluating

Client Needs: Physiological Integrity

Integrated Process: Nursing Process/Evaluation

Content Area: Pharmacology Awarded 0.0 points out of 1.0 possible points. 31.ID: 383703637 A nurse is assessing a client who has been taking amantadine hydrochloride (Symmetrel) for the treatment of Parkinson's disease. Which finding from the history and physical examination would cause the nurse to determine that the client may be experiencing an adverse effect of the medication? Insomnia Rigidity and akinesia Bilateral lung wheezes Correct Orthostatic hypotension Rationale: Amantadine hydrochloride is an antiparkinson agent that potentiates the action of dopamine in the central nervous system (CNS). The medication is used to treat rigidity and akinesia. Insomnia and orthostatic hypotension are side effects of the medication. Adverse effects include congestive heart failure (evidenced by bilateral lung wheezes), leukopenia, neutropenia, hyperexcitability, convulsions, and ventricular dysrhythmias.

Test-Taking Strategy: Use the process of elimination and focus on the subject, an adverse effect. Recalling that the medication is used to treat rigidity and akinesia will assist you in eliminating this option. To select from the remaining options, use the ABCs — airway, breathing, and circulation. Review the adverse effects of amantadine hydrochloride if you had difficulty with this question.

Reference: Lehne, R. (2010). Pharmacology for nursing care (7th ed., pp. 194, 1083). St. Louis: Saunders.

Cognitive Ability: Evaluating

Client Needs: Physiological Integrity

Integrated Process: Nursing Process/Evaluation

Content Area: Adult Health/Neurological Awarded 0.0 points out of 1.0 possible points. 32.ID: 383703669 A nurse who will be staffing a booth at a health fair is preparing pamphlets containing information regarding the risk factors for osteoporosis. Which of the following risk factors does the nurse include in the pamphlet? Select all that apply. Smoking Correct A high-calcium diet High alcohol intake Correct White or Asian ethnicity Correct Participation in physical activities that promote flexibility and muscle strength Rationale: Osteoporosis is a chronic metabolic disease in which bone loss results in decreased density and sometimes fractures. Risk factors include being 65 years or older in women, 75 years or older in men, family history of the disorder, history of fracture after age 50, white or Asian ethnicity, low body weight and slender build, chronically low calcium intake, a history of smoking, high alcohol intake, and lack of physical exercise or prolonged immobility.

Test-Taking Strategy: Focus on the subject, a risk factor. Recalling that this disorder involves bone loss resulting in decreased density and sometimes fractures will direct you to the correct options. Review the risk factors for osteoporosis if you are not familiar with them.

Reference: Ignatavicius, D., & Workman, M. (2010). Medical-surgical nursing: Patient-centered collaborative care (6th ed., pp. 1153, 1154, 1157). St. Louis: Saunders.

Cognitive Ability: Applying

Client Needs: Health Promotion and Maintenance

Integrated Process: Teaching and Learning

Content Area: Adult Health/Musculoskeletal Awarded 0.0 points out of 1.0 possible points. 33.ID: 383706656 A nurse is providing instruction to a client with osteoporosis regarding appropriate foods to include in the diet. The nurse tells the client that one food item high in calcium is: Corn Cocoa Peaches Sardines Correct Rationale: Osteoporosis is a chronic metabolic disease in which bone loss results in decreased density and sometimes fractures. Foods high in calcium include milk and milk products, dark-green leafy vegetables, tofu and other soy products, sardines, and hard water. Corn, cocoa, and peaches do not contain appreciable amounts of calcium.

Test-Taking Strategy: Focus on the subject, foods high in calcium. Thinking about the nutritional components of each food item in the options and recalling the foods that contain calcium will assist you in answering correctly. Review food items high in calcium if you had difficulty with this question.

Reference: Nix, S. (2009). Williams’ basic nutrition and diet therapy (13th ed., p. 129). St. Louis: Mosby.

Cognitive Ability: Applying

Client Needs: Health Promotion and Maintenance

Integrated Process: Teaching and Learning

Content Area: Adult Health/Musculoskeletal Awarded 0.0 points out of 1.0 possible points. 34.ID: 383705045 A nurse is providing information to a client with acute gout about home care. Which of the following measures does the nurse tell the client to take? Select all that apply. Drinking 2 to 3 L of fluid each day Correct Applying heat packs to the affected joint Resting and immobilizing the affected area Correct Consuming foods high in purines Performing range-of-motion exercise to the affected joint three times a day Rationale: Gout is a systemic disease in which urate crystals are deposited in the joints and other tissues, resulting in inflammation. In acute gout, rest and immobilization are recommended until the acute attack and inflammation have subsided. Local application of cold may help relieve the pain. The application of heat is avoided because it may worsen the inflammatory process. Dietary instructions include reducing or eliminating alcohol intake and avoiding excessive intake of foods containing purines (e.g., sweetbreads, yeast, heart, herring, herring roe, sardines). The client is encouraged to drink 2 to 3 L of fluid per day to help eliminate uric acid and to prevent the formation of renal calculi.

Test-Taking Strategy: Focus on the the client’s diagnosis and think about its pathophysiology. Noting the strategic word "acute" will also assist you in answering correctly. Review care of the client with gout if you had difficulty with this question.

References: Black, J., & Hawks, J. (2009). Medical-surgical nursing: Clinical management for positive outcomes (8th ed., p. 497). St. Louis: Saunders.
Ignatavicius, D., & Workman, M. (2010). Medical-surgical nursing: Patient-centered collaborative care (6th ed., p. 354). St. Louis: Saunders.

Cognitive Ability: Applying

Client Needs: Physiological Integrity

Integrated Process: Teaching and Learning

Content Area: Adult Health/Musculoskeletal Awarded 0.0 points out of 1.0 possible points. 35.ID: 383702989 A nurse is gathering subjective and objective data from a client with suspected rheumatoid arthritis (RA). Which early manifestations of RA would the nurse expect to note? Select all that apply. Fatigue Correct Anemia Weight loss Low-grade fever Correct Joint deformities Rationale: Rheumatoid arthritis is a chronic, progressive, systemic and inflammatory autoimmune disease process that affects the synovial joints, resulting in their destruction. Early manifestations of RA include fatigue, low-grade fever, weakness, anorexia, and paresthesias. Anemia, weight loss, and joint deformities are some of the late manifestations.

Test-Taking Strategy: Note the strategic word “early.” Think about the pathophysiology of RA and focus on the strategic word to find the correct options. Review the early manifestations of RA if you had difficulty with this question.

Reference: Ignatavicius, D., & Workman, M. (2010). Medical-surgical nursing: Patient-centered collaborative care (6th ed., pp. 337, 338). St. Louis: Saunders.

Cognitive Ability: Analyzing

Client Needs: Physiological Integrity

Integrated Process: Nursing Process/Assessment

Content Area: Adult Health/Immune Awarded 0.0 points out of 1.0 possible points. 36.ID: 383709208 A nurse is reviewing the medical record of a client with a suspected systemic lupus erythematosus (SLE). Which manifestations of SLE would the nurse expect to find noted in the client’s medical record? Select all that apply. Fever Correct Vasculitis Correct Weight gain Increased energy Abdominal pain Correct Rationale: Systemic lupus erythematosus is a chronic, progressive, inflammatory disorder of the connective tissue that can cause the failure of major organs and body systems. Manifestations include fever, fatigue, anorexia, weight loss, vasculitis, discoid lesions, and abdominal pain. Erythema, usually in a butterfly pattern (hence the nickname “butterfly rash”), appears over the cheeks and bridge of the nose. Other manifestations include nephritis, pericarditis, the Raynaud phenomenon, pleural effusions, joint inflammation, and myositis.

Test-Taking Strategy: Think about the pathophysiology of SLE. Recalling that SLE is a chronic, progressive, inflammatory disorder of the connective tissue that can result in the failure of major organs and body systems will assist you in answering correctly. Review the early manifestations of SLE if you had difficulty with this question.

Reference: Ignatavicius, D., & Workman, M. (2010). Medical-surgical nursing: Patient-centered collaborative care (6th ed., p. 348). St. Louis: Saunders.

Cognitive Ability: Analyzing

Client Needs: Physiological Integrity

Integrated Process: Nursing Process/Assessment

Content Area: Adult Health/Immune Awarded 0.0 points out of 1.0 possible points. 37.ID: 383702983 A nurse is providing dietary instructions to a client who is taking tranylcypromine sulfate (Parnate). Which of the following foods does the nurse tell the client to avoid while she is taking this medication? Select all that apply. Beer Correct Apples Yogurt Correct Baked haddock Pickled herring Correct Roasted fresh potatoes Rationale: Tranylcypromine sulfate is a monoamine oxidase inhibitor (MAOI) used to treat depression. The client must follow a tyramine-restricted diet while taking the medication to help prevent hypertensive crisis, a life-threatening effect of the medication. Foods to be avoided include meats prepared with tenderizer, smoked or pickled fish, beef or chicken liver, and dry sausages (e.g., salami, pepperoni, bologna). In addition, figs, bananas, aged cheeses, yogurt and sour cream, beer, red wine, alcoholic beverages, soy sauce, yeast extract, chocolate, caffeine, and aged, pickled, fermented, or smoked foods must be avoided. Many over-the-counter medications contain tyramine and must be avoided as well.

Test-Taking Strategy: First recall that tranylcypromine sulfate is an MAOI and that tyramine-containing items must be avoided. Next, think about the foods that contain tyramine, which will direct you to the correct options. Review tranylcypromine sulfate and foods high in tyramine if you had difficulty with this question.

References: Lehne, R. (2010). Pharmacology for nursing care (7th ed., p. 358). St. Louis: Saunders.
Nix, S. (2009). Williams’ basic nutrition and diet therapy (13th ed., p. 333). St. Louis: Mosby.

Cognitive Ability: Applying

Client Needs: Physiological Integrity

Integrated Process: Teaching and Learning

Content Area: Pharmacology Awarded 0.0 points out of 1.0 possible points. 38.ID: 383703659 The blood serum level of imipramine is determined in a client who is being treated for depression with Tofranil-PM. The laboratory test indicates a concentration of 250 ng/mL. On the basis of this result, the nurse should: Contact the physician Hold the next dose of imipramine Document the laboratory result in the client's record Correct Have another blood sample drawn and ask the laboratory to recheck the imipramine level Rationale: Imipramine is a tricyclic antidepressant that is often used to treat depression. The therapeutic blood serum level is between 225 and 300 ng/mL, so the nurse would simply document the laboratory result in the client’s record. Asking the laboratory to recheck the level and withholding the next dose of the imipramine and contacting the physician are unnecessary.

Test-Taking Strategy: Use the process of elimination and recall the therapeutic blood serum level. Eliminate the options that are comparable or alike in that they indicate that the result is abnormal. Review the therapeutic blood serum level of imipramine if you had difficulty with this question.

References: Hodgson, B., & Kizior, R. (2010). Saunders nursing drug handbook 2009 (p. 600). St. Louis: Saunders.
Varcarolis, E., & Halter, M. (2009). Essentials of psychiatric mental health nursing: A communication approach to evidence-based care (p. 232). St. Louis: Saunders.

Cognitive Ability: Applying

Client Needs: Physiological Integrity

Integrated Process: Nursing Process/Implementation

Content Area: Pharmacology Awarded 0.0 points out of 1.0 possible points. 39.ID: 383706617 A nurse provides instructions to a client who has been prescribed lithium carbonate (Lithobid) for the treatment of bipolar disorder. Which of these statements by the client indicate a need for further instruction? Select all that apply. "I need to avoid salt in my diet." Correct "It’s fine to take any over-the-counter medication with the lithium." Correct "I need to come back the clinic to have my lithium blood level checked." " I should drink 2 to 3 quarts of liquid every day." “Diarrhea and muscle weakness are to be expected, and if these occur I don’t need to be concerned.” Correct Rationale: Lithium carbonate is a mood stabilizer used to treat manic-depressive illness. Equilibrium of sodium and potassium must be maintained at the intracellular membrane to maintain therapeutic effects. Lithium competes with sodium in the cell. Therefore the client should maintain a normal salt intake and drink 2 to 3 quarts of fluid each day. Many over-the-counter medications contain sodium and would therefore affect the lithium concentration, possibly pushing it out of the therapeutic range. For this reason, over-the-counter medications must be avoided. The blood level of lithium should be tested every 3 or 4 days during the initial phase of therapy and every 1 to 2 months during maintenance therapy. Vomiting, diarrhea, muscle weakness, tremors, drowsiness, and ataxia are signs of toxicity; if any of these problems occur, the physician must be notified.

Test-Taking Strategy: Note the strategic words "need for further instruction." These words indicate a negative event query and the need to select the statements reflecting incorrect understanding on the part of the client. Recalling the pathophysiology related to the action of this medication will direct you to the correct option. Review teaching points for the client taking lithium carbonate if you had difficulty with this question.

Reference: Varcarolis, E., & Halter, M. (2009). Essentials of psychiatric mental health nursing: A communication approach to evidence-based care (p. 263). St. Louis: Saunders.

Cognitive Ability: Evaluating

Client Needs: Physiological Integrity

Integrated Process: Teaching and Learning

Content Area: Pharmacology Awarded 0.0 points out of 1.0 possible points. 40.ID: 383706670 A client who is taking lithium carbonate (Lithobid) complains of mild nausea, voiding in large volumes, and thirst. On assessment, the nurse notes that the client is complaining of mild thirst. On the basis of these findings, the nurse should: Contact the physician Document the findings Correct Institute seizure precautions Have a blood specimen drawn immediately for serum lithium testing Rationale: Lithium carbonate is a mood stabilizer that is used to treat manic-depressive illness. Side effects include polyuria, mild thirst, and mild nausea, and therefore the nurse would simply document the findings. Because the client’s complaints are side effects, not toxic effects, contacting the physician, instituting seizure precautions, and having a specimen drawn immediately for a serum lithium determination are all unnecessary. Vomiting, diarrhea, muscle weakness, tremors, drowsiness, and ataxia are signs of toxicity and if these occur the physician needs to be notified.

Test-Taking Strategy: Focus on the data in the question. Eliminate the options that are comparable or alike in that they indicate that the client is experiencing a toxic effect of the medication. Review the side effects and the signs lithium carbonate toxicity if you had difficulty with this question.

Reference: Varcarolis, E., & Halter, M. (2009). Essentials of psychiatric mental health nursing: A communication approach to evidence-based care (pp. 261, 262). St. Louis: Saunders.

Cognitive Ability: Applying

Client Needs: Physiological Integrity

Integrated Process: Nursing Process/Implementation

Content Area: Pharmacology Awarded 0.0 points out of 1.0 possible points. 41.ID: 383710528 A client with agoraphobia will undergo systematic desensitization through graduated exposure. In explaining the treatment to the client, the nurse tells the client that this technique involves: Having the client perform a healthy coping behavior Having the client perform a ritualistic or compulsive behavior Providing a high degree of exposure of the client to the stimulus that the client finds undesirable Gradually introducing the client to a phobic object or situation in a predetermined sequence of least to most frightening Correct Rationale: The technique of systematic desensitization involves gradually introducing the client to a phobic object or situation in a predetermined sequence of least to most frightening with the goal of defusing the phobia. Having the client perform a healthy coping behavior is the description of modeling. Performing ritualistic or compulsive behaviors is a behavior characteristic of clients with obsessive-compulsive disorder. Having the client perform a ritualistic or compulsive behavior may not be therapeutic; additionally, it is not associated with systematic desensitization. Providing a high degree of exposure to a stimulus that the client finds undesirable is the technique known as flooding.

Test-Taking Strategy: Use the process of elimination, noting the relationship between the words "graduated exposure" in the question and "gradually introducing" in the correct option. Review systematic desensitization if you had difficulty with this question.

Reference: Varcarolis, E., & Halter, M. (2009). Essentials of psychiatric mental health nursing: A communication approach to evidence-based care (p. 30). St. Louis: Saunders.

Cognitive Ability: Applying

Client Needs: Psychosocial Integrity

Integrated Process: Nursing Process/Implementation

Content Area: Mental Health Awarded 0.0 points out of 1.0 possible points. 42.ID: 383707986 A nurse is caring for a client who has just undergone esophagogastroduodenoscopy (EGD). The client says to the nurse, "I’m really thirsty — may I have something to drink?" Before giving the client a drink, the nurse should: Check the client's vital signs Check for the presence of a gag reflex Correct Assess the client for the presence of bowel sounds Ask the client to gargle with a warm saline solution Rationale: In preparation for EGD, the client’s throat is usually sprayed with an anesthetic to dampen the gag reflex and permit the introduction of the endoscope used to visualize the gastrointestinal structures. After EGD, the nurse places the highest priority on assessing the client for the return of the gag reflex. No food or oral fluids are given to the client until the gag reflex is fully intact.Vital signs are checked frequently, but this action is not associated with giving the client oral fluids. The client may be asked to use throat lozenges or a saline gargle to relieve a sore throat after the test, but neither action is related to giving the client oral fluids; additionally, neither action would be taken until the gag reflex had been detected again. Bowel sounds are not affected by this test.

Test-Taking Strategy: Think about what the procedure for an EGD involves and remember the ABCs — airway, breathing, and circulation. Assessing the client for the presence of a gag reflex helps ensure a patent airway before the client is allowed to drink. Review procedure care after EGD if you are unfamiliar with this procedure.

Reference: Ignatavicius, D., & Workman, M. (2010). Medical-surgical nursing: Patient-centered collaborative care (6th ed., p. 1227). St. Louis: Saunders.

Cognitive Ability: Applying

Client Needs: Safe and Effective Care Environment

Integrated Process: Nursing Process/Implementation

Content Area: Adult Health/Gastrointestinal Awarded 0.0 points out of 1.0 possible points. 43.ID: 383706613 A nurse is developing a plan of care for a pregnant client with sickle-cell disease. Which concern does the nurse recognize as the priority? Inability to cope Decreased nutrition Decreased fluid volume Correct Inability to tolerate activity Rationale: Sickle cell disease is a genetic disorder that is manifested as chronic anemia, pain, disability, organ damage, increased risk for infection, and early death. In this disorder the red blood cells assume a sickle shape, become rigid, and clump together. Dehydration can precipitate sickling of the red blood cells. Sickling can lead to life-threatening consequences for the pregnant woman and the fetus, including interruption of blood flow to the respiratory system and placenta. Decreased fluid volume is the priority concern in this situation, followed by decreased nutrition. Inability to tolerate activity and inability to cope compete for third priority, depending on the client’s specific symptoms at the time.

Test-Taking Strategy: Use Maslow’s Hierarchy of Needs theory to prioritize, remembering that physiological needs come first. This will help you eliminate the option of inability to cope. To select from the remaining options, use your knowledge of the pathophysiology of sickle cell disease to select the correct option. In this case, decreased fluid volume is the most serious physiological consequence of sickling. Review sickle cell disease if you had difficulty with this question.

Reference: McKinney, E., James, S., Murray, S., & Ashwill, J. (2009). Maternal-child nursing (3rd ed., p. 649). St. Louis: Elsevier.

Cognitive Ability: Analyzing

Client Needs: Physiological Integrity

Integrated Process: Nursing Process/Planning

Content Area: Maternity/Antepartum Awarded 0.0 points out of 1.0 possible points. 44.ID: 383710582 A nurse is preparing a pregnant client in the third trimester for an amniocentesis. The nurse explains to the client that amniocentesis is often performed during the third trimester to determine: The sex of the fetus Genetic characteristics An accurate age for the fetus The degree of fetal lung maturity Correct Rationale: Amniocentesis is the aspiration of fluid from the amniotic sac for examination. Common indications for amniocentesis during the third trimester include assessment of fetal lung maturity and evaluation of fetal condition when the woman has Rh isoimmunization. A common purpose of amniocentesis in the second trimester is to examine fetal cells in the amniotic fluid to identify chromosomal abnormalities. Other methods of genetic analysis, such as those for metabolic defects in the fetus, may be performed on the cells as well. The sex and age of the fetus are not determined with the use of amniocentesis.

Test-Taking Strategy: Use the process of elimination. Note the relationship between the word "third trimester" in the question and "maturity" in the correct option. Review the purpose of amniocentesis in the third trimester if you had difficulty with this question.

Reference: Lowdermilk, D., Perry, S., & Cashion, K., (2010). Maternity nursing (8th ed., pp. 569-571). St. Louis: Mosby.

Cognitive Ability: Understanding

Client Needs: Physiological Integrity

Integrated Process: Nursing Process/Implementation

Content Area: Maternity/Intrapartum Awarded 0.0 points out of 1.0 possible points. 45.ID: 383709202 A nurse provides instruction to a pregnant woman about foods containing folic acid. Which of these foods does the nurse tell the client to consume as sources of folic acid? Select all that apply. Bananas Potatoes Spinach Correct Legumes Correct Whole grains Correct Milk products Rationale: Folic acid is needed during pregnancy for healthy cell growth and repair. A pregnant woman should have at least four servings of folic acid–rich foods per day. Some foods high in folic acid are glandular meats, yeast, dark-green leafy vegetables, legumes, and whole grains. Bananas provide potassium. Potatoes provide vitamin B6, and milk products are a source of calcium.

Test-Taking Strategy: Knowledge regarding foods high in folic acid will direct you to the correct options. Use of the process of elimination and think about the components of each of the foods in the options to answer correctly. Review the food sources of folic acid if you had difficulty with this question.

References: Lowdermilk, D., Perry, S., & Cashion, K., (2010). Maternity nursing (8th ed., p. 242). St. Louis: Mosby.
McKinney, E., James, S., Murray, S., & Ashwill, J. (2009). Maternal-child nursing (3rd ed., p. 302). St. Louis: Elsevier.

Cognitive Ability: Applying

Client Needs: Physiological Integrity

Integrated Process: Teaching and Learning

Content Area: Maternity/Antepartum Awarded 0.0 points out of 1.0 possible points. 46.ID: 383706672 A nurse caring for a client with preeclampsia prepares for the administration of an intravenous infusion of magnesium sulfate. Which of the following substances does the nurse ensure is available at the client's bedside? Vitamin K Protamine sulfate Potassium chloride Calcium gluconate Correct Rationale: Magnesium sulfate, which has anticonvulsant properties, is used for a client with preeclampsia to help prevent seizures (eclampsia). It also causes central nervous system depression, however, so toxicity is a concern. Calcium gluconate should be available at the bedside of a client receiving an intravenous infusion of magnesium sulfate to reverse magnesium toxicity and prevent respiratory arrest if the serum magnesium level becomes too high. Vitamin K is the antidote for warfarin sodium (Coumadin). Protamine sulfate is the antidote for heparin. Potassium chloride is used to treat potassium deficiency.

Test-Taking Strategy: Knowledge regarding the antidotes to various medications is needed to answer this question. Use the process of elimination and remember that calcium gluconate is the antidote to magnesium sulfate. Review care of the client receiving magnesium sulfate if you had difficulty with this question.

Reference: Lowdermilk, D., Perry, S., & Cashion, K., (2010). Maternity nursing (8th ed., p. 685). St. Louis: Mosby.

Cognitive Ability: Applying

Client Needs: Physiological Integrity

Integrated Process: Nursing Process/Implementation

Content Area: Maternity/Intrapartum Awarded 0.0 points out of 1.0 possible points. 47.ID: 383705033 A nurse is monitoring a client receiving terbutaline (Brethine) by intravenous infusion to stop preterm labor. The nurse notes that the client's heart rate is 120 beats/min and that the fetal heart rate is 170 beats/min. The appropriate action by the nurse is: Contacting the physician Correct Documenting the findings Continuing to monitor the client Increasing the rate of the infusion Rationale: Terbutaline may be used to stop preterm labor. It stimulates beta-adrenergic receptors of the sympathetic nervous system, resulting in bronchodilation and inhibition of uterine muscle activity. The nurse monitors the client for adverse effects and notifies the physician if the maternal heart rate is faster than 110 beats/min, respiration is faster than 24 breaths/min, systolic blood pressure is less than 90 mm Hg, the fetal heart rate is faster than 160 beats/min, or the client complains of chest pain or dyspnea. Increasing the rate of infusion and continuing to monitor the client and are inappropriate and delay necessary interventions. Although the nurse would document the findings, the most appropriate action in this scenario is to contact the physician.

Test-Taking Strategy: Use the process of elimination. Focusing on the data in the question and recalling the normal maternal heart rate and normal fetal heart rate will direct you to the correct option. Review care of the client receiving terbutaline if you had difficulty with this question.

Reference: McKinney, E., James, S., Murray, S., & Ashwill, J. (2009). Maternal-child nursing (3rd ed., p. 682). St. Louis: Elsevier.

Cognitive Ability: Applying

Client Needs: Physiological Integrity

Integrated Process: Nursing Process/Implementation

Content Area: Pharmacology Awarded 0.0 points out of 1.0 possible points. 48.ID: 383702979 A nurse provides home care instructions to a client with mild preeclampsia. The nurse tells the client that: Sodium intake is restricted Fluid intake must be limited to 1 quart each day Urine output must be measured and that the physician should be notified if output is less than 500 mL in a 24-hour period Correct Urinary protein must be measured and that the physician should be notified if the results indicate a trace amount of protein Rationale: Preeclampsia is considered mild when the diastolic blood pressure does not exceed 100 mm Hg, proteinuria is no more than 500 mg/day (trace to 1+), and symptoms such as headache, visual disturbances, and abdominal pain are absent. The diet should provide ample protein and calories, and fluid and sodium should not be limited. The disease is considered severe when the blood pressure is higher than 160/110 mm Hg, proteinuria is greater than 5 g/24 hr (3+ or more), and oliguria is present (500 mL or less in 24 hours). Therefore, urine output of less than 500 mL/24 hr should prompt the client to notify the physician.

Test-Taking Strategy: Use the process of elimination. Thinking about the nutritional needs of the pregnant client will assist you in eliminating the options addressing sodium and fluid intake. To select from the remaining options, focus on the client’s diagnosis, mild preeclampsia, and recall that a trace amount of protein is expected. Review home care measures for the client with mild preeclampsia if you had difficulty with this question.

Reference: McKinney, E., James, S., Murray, S., & Ashwill, J. (2009). Maternal-child nursing (3rd ed., p. 628). St. Louis: Elsevier.

Cognitive Ability: Applying

Client Needs: Physiological Integrity

Integrated Process: Teaching and Learning

Content Area: Maternity/Antepartum Awarded 0.0 points out of 1.0 possible points. 49.ID: 383712409 A nurse is monitoring a hospitalized client who is being treated for preeclampsia. Which items of the following information elicited during the assessment indicate that the condition has not yet resolved? Type the option number that is the correct answer.

Answer: ______ Nursing Progress Notes Hyperreflexia is present. Urinary protein is not detectable. Urine output is 45 mL/hr. Blood pressure is 128/78 mm Hg. Incorrect Correct Responses: "1" Awarded 0.0 out of 1.0 possible points. 50.ID: 383710524 A nurse is caring for a client who sustained a missed abortion during the second trimester of pregnancy. For which finding indicating the need for further evaluation does the nurse monitor the client? Spontaneous bruising Correct Decrease in uterine size Urine output of 30 mL/hr Brownish vaginal discharge Rationale: Missed abortion is the term used to describe when a fetus dies during the first half of pregnancy but is retained in the uterus. When the fetus dies, the early symptoms of pregnancy (e.g., nausea, breast tenderness, urinary frequency) disappear. The uterus stops growing and begins to shrink. Red or brownish vaginal bleeding may or may not occur. A major complication of a missed abortion is disseminated intravascular coagulation (DIC). Bleeding at the sites of intravenous needle insertion or laboratory blood draws, nosebleeds, and spontaneous bruising may be early indicators of DIC; they should be reported and require further evaluation.

Test-Taking Strategy: Focus on the client’s diagnosis and recall the complications associated with a missed abortion. Noting the strategic words "need for further evaluation" will assist in directing you to the correct option. Review the complications associated with a missed abortion and the signs of DIC if you had difficulty with this question.

Reference: McKinney, E., James, S., Murray, S., & Ashwill, J. (2009). Maternal-child nursing (3rd ed., p. 609). St. Louis: Elsevier.

Cognitive Ability: Evaluating

Client Needs: Physiological Integrity

Integrated Process: Nursing Process/Evaluation

Content Area: Maternity/Postpartum Awarded 0.0 points out of 1.0 possible points. 51.ID: 383706056 A client is receiving an intravenous infusion of oxytocin (Pitocin) to stimulate labor. The nurse monitoring the client notes uterine hypertonicity and immediately: Stops the oxytocin infusion Correct Checks the vagina for crowning Encourages the client to take short, deep breaths Increases the rate of the oxytocin infusion and calls the physician Rationale: Oxytocin is a synthetic compound identical to the natural hormone secreted from the posterior pituitary gland. It is used to induce or augment labor at or near term. The nurse monitors uterine activity for the establishment of an effective labor pattern and for complications associated with the use of the medication. If uterine hypertonicity or a nonreassuring fetal heart rate pattern is detected, the nurse must intervene to reduce uterine activity and increase fetal oxygenation. The nurse would stop the oxytocin infusion and increase the rate of the nonadditive solution, position the client in a side-lying position, and administer oxygen with the use of a snug face mask at 8 to 10 L/min. The nurse would also notify the physician. Checking the vagina for crowning; encouraging the client to take short, deep breaths; and increasing the rate of the oxytocin infusion are not the immediate actions.

Test-Taking Strategy: Use the process of elimination and note the strategic word “immediately.” Recalling the action and purpose of oxytocin and noting the words "uterine hypertonicity" will direct you to the correct option. Review the expected outcomes and the signs of complications associated with oxytocin infusion if you had difficulty with this question.

Reference: Lowdermilk, D., Perry, S., & Cashion, K. (2010). Maternity nursing (8th ed., pp. 701, 702). St. Louis: Mosby.

Cognitive Ability: Applying

Client Needs: Physiological Integrity

Integrated Process: Nursing Process/Implementation

Content Area: Maternity/Intrapartum Awarded 0.0 points out of 1.0 possible points. 52.ID: 383711479 A nurse is monitoring a pregnant woman in labor and notes this finding on the fetal-monitor tracing (see figure). Which of the following actions should the nurse take as a result of this observation?

 Repositioning the mother Documenting the finding Correct Notifying the nurse-midwife Taking the mother's vital signs Rationale: The nurse sees evidence of accelerations. Accelerations are transient increases in the fetal heart rate that often accompany contractions and are normally caused by fetal movement. Accelerations are thought to be a sign of fetal well-being and adequate oxygen reserve. Repositioning the mother, notifying the nurse-midwife, and taking the mother’s vital signs are all unnecessary actions.

Test-Taking Strategy: Examine the fetal monitor tracing and see that it shows accelerations. Recalling that the presence of accelerations indicates fetal well-being will direct you to the correct option. Review the significance of accelerations on fetal monitoring if you had difficulty with this question.

Reference: McKinney, E., James, S., Murray, S., & Ashwill, J. (2009). Maternal-child nursing (3rd ed., pp. 393, 395). St. Louis: Elsevier.

Cognitive Ability: Applying

Client Needs: Physiological Integrity

Integrated Process: Nursing Process/Implementation

Content Area: Maternity/Intrapartum Awarded 0.0 points out of 1.0 possible points. 53.ID: 383702973 A client with cervical cancer who is undergoing chemotherapy with cisplatin (Platinol). For which adverse effect of cisplatin will the nurse assess the client? Nausea Bloody urine Hearing loss Correct Electrocardiographic changes Rationale: Cisplatin is a platinum-based agent used to treat various types of cancer. One adverse effect of cisplatin is ototoxicity, and the nurse would monitor the client for tinnitus and hearing loss. Nausea occurs with the use of several chemotherapeutic agents and is not necessarily an adverse effect. Cyclophosphamide causes hemorrhagic cystitis, evidenced by bloody urine. Doxorubicin (Adriamycin) causes cardiotoxicity.

Test-Taking Strategy: Focus on the subject, an adverse effect. This question may be difficult to answer unless you have some specific knowledge regarding cisplatin. Remember that this chemotherapeutic agent causes ototoxicity. Review the adverse effects of cisplatin if you had difficulty with this question.

Reference: Hodgson, B., & Kizior, R. (2010). Saunders nursing drug handbook 2010 (p. 243). St. Louis: Saunders.

Cognitive Ability: Analyzing

Client Needs: Physiological Integrity

Integrated Process: Nursing Process/Assessment

Content Area: Pharmacology Awarded 0.0 points out of 1.0 possible points. 54.ID: 383709219 A nurse is monitoring a pregnant client with suspected partial placenta previa who is experiencing vaginal bleeding. Which of the following findings would the nurse expect to note on assessment of the client? Painful vaginal bleeding Sustained tetanic contractions Complaints of abdominal pain Soft, relaxed, nontender uterus Correct Rationale: Partial placenta previa is incomplete coverage of the internal os by the placenta. One characteristic of placenta previa is painless vaginal bleeding. The abdominal assessment would reveal a soft, relaxed, nontender uterus with normal tone. Vaginal bleeding and uterine pain and tenderness accompany placental abruption, especially with a central abruption and blood trapped behind the placenta. In placental abruption, the abdomen feels hard and boardlike on palpation as the blood penetrates the myometrium, resulting in uterine irritability. A sustained tetanic contraction may occur if the client is in labor and the uterine muscle cannot relax.

Test-Taking Strategy: Use the process of elimination and focus on the client’s diagnosis, placenta previa. It is easy to confuse placenta previa and abruption; remember that the difference involves the presence of uterine pain and tenderness with an abruption, versus painless bleeding and a soft, relaxed, and nontender uterus with placenta previa. Review the difference between placenta previa and placental abruption if you had difficulty with this question.

Reference: McKinney, E., James, S., Murray, S., & Ashwill, J. (2009). Maternal-child nursing (3rd ed., pp. 614, 615). St. Louis: Elsevier.

Cognitive Ability: Analyzing

Client Needs: Physiological Integrity

Integrated Process: Nursing Process/Assessment

Content Area: Maternity/Intrapartum Awarded 0.0 points out of 1.0 possible points. 55.ID: 383706609 A nurse assisting with a delivery is monitoring the client for placental separation after the delivery of a viable newborn. Which of the following observations indicates to the nurse that placental separation has occurred? A discoid uterus Sudden sharp vaginal pain Shortening of the umbilical cord A sudden gush of dark blood from the introitus Correct Rationale: Placental separation occurs when the placenta separates from the uterus. Signs of placental separation include lengthening of the umbilical cord, a sudden gush of dark blood from the introitus, a firmly contracted uterus, and a change in uterine shape from discoid to globular. The client may experience vaginal fullness but sudden sharp vaginal pain is not usual.

Test-Taking Strategy: Use the process of elimination and focus on the subject, placental separation. Try visualizing this physiological process as a means of finding the correct option. Review the signs of placental separation if you had difficulty with this question.

Reference: Lowdermilk, D., Perry, S., & Cashion, K. (2010). Maternity nursing (8th ed., pp. 376, 377). St. Louis: Mosby.

Cognitive Ability: Analyzing

Client Needs: Physiological Integrity

Integrated Process: Nursing Process/Assessment

Content Area: Maternity/Intrapartum Awarded 0.0 points out of 1.0 possible points. 56.ID: 383702959 A nurse is conducting a preoperative psychosocial assessment of a client who is scheduled for a mastectomy. Which of the following findings would cause the nurse to conclude that the client is at risk for poor sexual adjustment after the mastectomy? The client reports a history of sexual abuse by her father. Correct The client reports that her relationship with her spouse is stable. The client reports a satisfying intimate relationship with her spouse. The client reports that her and her spouse have never been able to conceive children Rationale: Clients at risk for self-esteem problems and poor sexual adjustment after mastectomy include those who report a lack of support from a spouse or partner; the existence of an unhappy, unstable intimate relationship; or a history of sexual problems or of sexual abuse, such as rape or incest. Clients with problems involving intimate relationships and sexuality should be referred for counseling. The remaining options are unrelated to the problem of poor sexual adjustment.

Test-Taking Strategy: Use the process of elimination. Noting the strategic words "at risk for poor sexual adjustment" will direct you to the correct option. Review the psychosocial issues related to mastectomy if you had difficulty with this question.

References: Black, J., & Hawks, J. (2009). Medical-surgical nursing: Clinical management for positive outcomes (8th ed., pp. 954, 955). St. Louis: Saunders.
Ignatavicius, D., & Workman, M. (2010). Medical-surgical nursing: Patient-centered collaborative care (6th ed., p. 1618). St. Louis: Saunders.

Cognitive Ability: Analyzing

Client Needs: Psychosocial Integrity

Integrated Process: Nursing Process/Assessment

Content Area: Mental Health Awarded 0.0 points out of 1.0 possible points. 57.ID: 383705047 A nurse provides instructions to a client who is preparing for discharge after a radical vulvectomy for the treatment of cancer. Which statement by the client indicates a need for further instruction? "I can resume sexual activity in 4 to 6 weeks." "I need to avoid straining when I have a bowel movement." "I should wear support hose for 6 months and elevate my legs frequently." "I need to contact my surgeon immediately if I feel any numbness in my genital area." Correct Rationale: After radical vulvectomy, the client is instructed to wear support hose for 6 months and to elevate the legs frequently. The client should avoid straining during defecation and should be told that alteration in the direction of urine flow may occur. The client may resume sexual activity in 4 to 6 weeks; the nurse should discuss the possible need for lubrication and position changes during coitus. Genital numbness may be present, but it is not necessary to notify the surgeon immediately if numbness occurs.

Test-Taking Strategy: Use the process of elimination. Note the strategic words "need for further instruction." Thinking about the surgical involvement with this procedure and noting the word "immediately" in the correct option will direct you to this option. Review instructions for the client after radical vulvectomy if you had difficulty with this question.

Reference: Black, J., & Hawks, J. (2009). Medical-surgical nursing: Clinical management for positive outcomes (8th ed., pp. 936, 937). St. Louis: Saunders.

Cognitive Ability: Evaluating

Client Needs: Physiological Integrity

Integrated Process: Teaching and Learning

Content Area: Adult Health/Oncology Awarded 0.0 points out of 1.0 possible points. 58.ID: 383703653 An adult client with an ileostomy is admitted to the hospital with a diagnosis of isotonic dehydration. What findings does the nurse expect to note during the admission assessment? Select all that apply. Skin tenting Correct Flat neck veins Correct Weak peripheral pulses Correct Moist oral mucous membranes A heart rate of 88 beats/min A respiratory rate of 18 breaths/min Rationale: Isotonic dehydration decreases circulating blood volume (hypovolemia), leading to inadequate tissue perfusion. The nurse would expect to note tachycardia, tachypnea, and dry oral mucous membranes. The oral mucous membranes may be covered with a thick, sticky, pastelike coating and may exhibit fissures. The client may also experience weight loss, lethargy or headache, sunken eyes, poor skin turgor (e.g., tenting), flat neck and peripheral veins, and low blood pressure. Peripheral pulses are weak, difficult to find, and easily obstructed with light pressure.

Test-Taking Strategy: Focus on the client’s diagnosis, isotonic dehydration. Recalling the pathophysiology of this type of dehydration and remembering that it leads to inadequate tissue perfusion will direct you to the correct options. Also note that the correct options are all normal findings. Review the signs of dehydration if you had difficulty with this question.

Reference: Ignatavicius, D., & Workman, M. (2010). Medical-surgical nursing: Patient-centered collaborative care (6th ed., p. 177). St. Louis: Saunders.

Cognitive Ability: Analyzing

Client Needs: Physiological Integrity

Integrated Process: Nursing Process/Assessment

Content Area: Adult Health/Gastrointestinal Awarded 0.0 points out of 1.0 possible points. 59.ID: 383707941 An adult client with renal failure who is oliguric and undergoing hemodialysis is under a fluid restriction of 700 mL/day. How many milliliters of fluid does the nurse allow the client to have between 7 a.m. and 3 p.m.? Type your answer in the space provided.

Answer ____mL Incorrect Correct Responses: "350" Awarded 0.0 out of 1.0 possible points. 60.ID: 383712466 A client with advanced chronic renal failure (CRF) and oliguria has been taught about sodium and potassium restriction between dialysis treatments. The nurse determines that the client understands this restriction if the client states that it is acceptable to use: Salt substitutes Herbs and spices Correct Salt with cooking only Processed foods as desired Rationale: Most clients with renal failure retain sodium. The client with renal failure is instructed not to add salt at the table or during food preparation. Herbs and spices may be used as an alternative to salt to enhance the flavor of food. The client with advanced CRF is instructed to limit potassium intake. The client is also instructed to avoid salt substitutes, many of which are composed of potassium chloride, if oliguria is present. Processed foods are discouraged because they are high in sodium.

Test-Taking Strategy: Use the process of elimination and focus on the subject, sodium and potassium restrictions. Eliminate the options that contain the words "only" and "as desired." To select from the remaining options, recall that salt substitutes contain potassium, which will assist you in eliminating this option. Review dietary instructions for the client with CRF if you had difficulty with this question.

Reference: Ignatavicius, D., & Workman, M. (2010). Medical-surgical nursing: Patient-centered collaborative care (6th ed., p. 1617). St. Louis: Saunders.

Cognitive Ability: Evaluating

Client Needs: Physiological Integrity

Integrated Process: Nursing Process/Evaluation

Content Area: Adult Health/Renal Awarded 0.0 points out of 1.0 possible points. 61.ID: 383702999 A nurse provides dietary instruction to a hospitalized client with chronic obstructive pulmonary disease (COPD). Which of the following menu selections by the client tells the nurse that the client understands the instructions? Coffee Broccoli Cheeseburger Correct Chocolate milk Rationale: The client with COPD is encouraged to eat a high-calorie, high-protein diet and to choose foods that are easy to chew and do not promote gas formation. Dry foods stimulate coughing, and foods such as milk and chocolate may increase the thickness of saliva and other secretions. The nurse advises the client to avoid these foods, as well as caffeinated beverages, which promote diuresis, contributing to dehydration, and may increase nervousness.

Test-Taking Strategy: Use the process of elimination and recall the pathophysiology of COPD. Eliminate the options that are comparable or alike in that they contain caffeine (i.e., coffee and chocolate). To select from the remaining options, eliminate the option that encourages gas formation (i.e., broccoli). Review care of the client with COPD if you had difficulty with this question.

Reference: Ignatavicius, D., & Workman, M. (2010). Medical-surgical nursing: Patient-centered collaborative care (6th ed., p. 633). St. Louis: Saunders.

Cognitive Ability: Evaluating

Client Needs: Physiological Integrity

Integrated Process: Nursing Process/Evaluation

Content Area: Adult Health/Respiratory Awarded 0.0 points out of 1.0 possible points. 62.ID: 383703649 Chlorpromazine (Thorazine) has been prescribed to a client with Huntington's disease for the relief of choreiform movements. Of which common side effect does the nurse warn the client? Headache Drowsiness Correct Photophobia Urinary frequency Rationale: Chlorpromazine is an antipsychotic, antiemetic, antianxiety, and antineuralgia adjunct. Common side effects of chlorpromazine include drowsiness, blurred vision, hypotension, defective color vision, impaired night vision, dizziness, decreased sweating, constipation, dry mouth, and nasal congestion. Headache, photophobia, and urinary frequency are not specific side effects of this medication.

Test-Taking Strategy: Knowledge of the common side effects of chlorpromazine is required to answer this question. Recalling the classification of this medication will direct you to the correct option. If you are unfamiliar with chloropromazine and its side effects, review this information.

Reference: Lehne, R. (2010). Pharmacology for nursing care (7th ed., pp. 323, 332, 333). St. Louis: Saunders.

Cognitive Ability: Analyzing

Client Needs: Physiological Integrity

Integrated Process: Nursing Process/Implementation

Content Area: Pharmacology Awarded 0.0 points out of 1.0 possible points. 63.ID: 383706684 A client who has sustained an acute myocardial infarction (AMI) is receiving intravenous reteplase (Retavase). For which adverse effect of the medication does the nurse monitor the client? Diarrhea Vomiting Epistaxis Correct Epigastric pain Rationale: Reteplase is a thrombolytic medication that promotes the fibrinolytic mechanism (i.e., conversion of plasminogen to plasmin, which destroys the fibrin in the blood clot). The thrombus, or blood clot, disintegrates when a thrombolytic medication is administered within 4 hours of an AMI. Necrosis resulting from blockage of the artery is prevented or minimized, and hospitalization may be shortened. Bleeding, a major adverse effect of thrombolytic therapy, may be superficial or internal and may be spontaneous. Epigastric pain, vomiting, and diarrhea are not adverse effects of this therapy.

Test-Taking Strategy: Use the process of elimination. Eliminate the options that are comparable or alike in that they involve gastrointestinal problems. Also focus on the name of the medication and recall that medications that end with the letters “ase” are thrombolytic agents and that they can cause bleeding. If you had difficulty with this question, review this medication and the adverse effects of thrombolytic therapy.

Reference: Gahart, B., & Nazareno, A. (2010). Intravenous medications (26th ed., p. 1180). St. Louis: Mosby.

Cognitive Ability: Analyzing

Client Needs: Physiological Integrity

Integrated Process: Nursing Process/Assessment

Content Area: Pharmacology Awarded 0.0 points out of 1.0 possible points. 64.ID: 383708588 A home care nurse visits a mother who delivered a healthy newborn 4 days ago and assesses how the mother is doing breastfeeding her infant. What does the nurse ask the mother to do to permit assessment of whether the infant is receiving an adequate amount of milk? Count the number of times that the infant swallows during a feeding Weigh the infant every day and check for a daily weight gain of 2 oz Count wet diapers to be sure that the infant is having at least six to 10 each day Correct Pump the breasts, place the milk in a bottle, measure the amount, and then bottle-feed the infant Rationale: The mother should be taught to count wet and soiled diapers to help determine whether the infant is receiving enough milk. Generally an infant should have at least 6 to 10 wet diapers (after the first 2 days of life) and at least 4 stools each day. The mother may also assess the swallowing and nutritive suckling of the infant, but this would not provide the best indication of adequate milk intake. Counting the number of times that the infant swallows during a feeding is an inadequate indicator of milk intake. The mother is not usually encouraged to weigh the infant at home, because this focuses too much attention on weight gain. Infants generally gain approximately 15 to 30 g (0.5 to 1 oz) each day after the early months of life. Pumping the breasts, placing the milk in a bottle, measuring the amount, and then bottle-feeding the infant constitute an assessment of the mother’s bottle-feeding technique.

Test-Taking Strategy: Focus on the subject of the question and use the process of elimination. Eliminate the option that addresses bottle-feeding rather than breastfeeding. Recalling that the infant gains approximately 1 oz/day will help you eliminate the option that involves weighing the infant. To select from the remaining options, note the relationship between adequate amount of milk intake addressed in the question and the infant’s output in the correct option. Review infant nutrition if you had difficulty with this question.

Reference: Lowdermilk, D., Perry, S., & Cashion, K., (2010). Maternity nursing (8th ed., p. 540) St. Louis: Mosby.

Cognitive Ability: Applying

Client Needs: Physiological Integrity

Integrated Process: Teaching and Learning

Content Area: Maternity/Postpartum Awarded 0.0 points out of 1.0 possible points. 65.ID: 383709213 A child who has just been found to have scoliosis will need to wear a thoracolumbosacral orthotic (TLSO) brace, and the nurse provides information to the mother about the brace. Which statement by the mother indicates a need for further information? "My child will need to do exercises." "My child needs to wear the brace 18 to 23 hours per day." "Wearing the brace is really important in curing the scoliosis." Correct "I need to check my child's skin under the brace to be sure it doesn't break down." Rationale: Scoliosis is a lateral curvature of the spine. Bracing is not curative of scoliosis but may slow the progression of the curvature to allow skeletal growth and maturation. A brace needs to be worn 18 to 23 hours a day, but it may be removed at night for sleep if this is prescribed. To be more cosmetically acceptable, a brace is usually worn under loose-fitting clothing. Back exercises are important in maintaining and strengthening the abdominal and spinal muscles. The child’s skin must be meticulously monitored for signs of breakdown.

Test-Taking Strategy: Use the process of elimination and note the strategic words "need for further information." These words indicate a negative event query and the need to select the incorrect statement. Read each option carefully. Noting the word "curing" will direct you to the correct option. Review the effects of bracing in the treatment of scoliosis if you had difficulty with this question.

Reference: Hockenberry, M., & Wilson, D. (2009). Wong’s essentials of pediatric nursing (8th ed., pp. 1131-1133). St. Louis: Mosby.

Cognitive Ability: Evaluating

Client Needs: Physiological Integrity

Integrated Process: Teaching and Learning

Content Area: Child Health/Musculoskeletal Awarded 0.0 points out of 1.0 possible points. 66.ID: 383710534 Ferrous sulfate (Feosol) is prescribed for a client. The nurse tells the client that it is best to take the medication with: Milk Water Any meal Tomato juice Correct Rationale: Ferrous sulfate is an iron product. Absorption of iron is promoted when the supplement is taken with orange juice or another food source of vitamin C or ascorbic acid. Calcium and phosphorus in milk decrease iron absorption. Water has no effect on the absorption of vitamin C. Telling the client to take the medication with any meal of the day does not guarantee that the iron will be taken with a food source of vitamin C or ascorbic acid. Additionally, it is best to take the iron supplement between meals with a drink high in ascorbic acid.

Test-Taking Strategy: Use the process of elimination. Recalling that ferrous sulfate is an iron product and that ascorbic acid increases the absorption of iron will direct you to the correct option. Review client instruction regarding iron supplements if you had difficulty with this question.

Reference: Kee, J., Hayes, E., & McCuistion, L. (2009). Pharmacology: A nursing process approach (6th ed., p. 813). Philadelphia: Saunders.

Cognitive Ability: Applying

Client Needs: Physiological Integrity

Integrated Process: Nursing Process/Implementation

Content Area: Pharmacology Awarded 0.0 points out of 1.0 possible points. 67.ID: 383702965 A client with type 1 diabetes mellitus has just been told that she is 6 weeks pregnant. The nurse provides information to the client about dietary and insulin needs and tells the client that during the first trimester, insulin needs generally: Increase Decrease Correct Remain unchanged Double from what they normally are Rationale: Insulin needs generally decrease during the first trimester of pregnancy because the secretion of placental hormones antagonistic to insulin remains low. An increase in insulin need, lack of change in insulin need, and doubling of insulin need are all incorrect.

Test-Taking Strategy: Use the process of elimination and your knowledge of the pathophysiology of diabetes mellitus and pregnancy to assist you in answering the question. Recalling that the need for insulin generally decreases in the first half of pregnancy and increases in the second half will direct you to the correct option. Review the effects of diabetes mellitus on pregnancy and insulin needs if you had difficulty with this question.

Reference: Lowdermilk, D., Perry, S., & Cashion, K., (2010). Maternity nursing (8th ed., p. 583). St. Louis: Mosby.

Cognitive Ability: Understanding

Client Needs: Physiological Integrity

Integrated Process: Teaching and Learning

Content Area: Maternity/Antepartum Awarded 0.0 points out of 1.0 possible points. 68.ID: 383703601 A nurse is assessing a pregnant woman for the presence of edema. The nurse places a thumb on the top of the client's foot, then exerts pressure and releases it and notes that the thumb has left a persistent depression. On the basis of this finding, the nurse concludes that: No edema is present The client is dehydrated Pitting edema is present Correct Blood is not pooling in the extremities Rationale: Edema in the lower extremities reflects pooling of blood, which results in a shift of intravascular fluid into the interstitial spaces. Dehydration is not likely to cause pitting edema. When pressure exerted with a finger or thumb leaves a persistent depression, the client is said to have "pitting edema." Therefore the other options identify incorrect interpretations.

Test-Taking Strategy: Use the process of elimination. Noting the words "persistent depression" in the question will help direct you to the correct option. If you had difficulty with this question, review the technique for assessing edema and the interpretation of the findings.

Reference: Perry, S., Hockenberry, M., Lowdermilk, D., & Wilson, D. (2010). Maternal-child nursing care (4th ed., pp. 340, 341). St. Louis: Mosby.

Cognitive Ability: Analyzing

Client Needs: Physiological Integrity

Integrated Process: Nursing Process/Assessment

Content Area: Maternity/Antepartum Awarded 0.0 points out of 1.0 possible points. 69.ID: 383705049 A nurse assessing the deep tendon reflexes of a pregnant client notes that the reflexes are 1+ (i.e., reflex present, hypoactive). On the basis of this finding, the nurse would: Contact the physician Document the findings Correct Ask the client to walk for 5 minutes, then recheck the reflexes Perform active and passive range-of-motion exercises of the client's lower extremities, then recheck the reflexes Rationale: The scale for rating deep tendon reflexes is as follows: 0 = absent; 1+ = present, hypoactive; 2+ = normal; 3+ = hyperactive; 4+ = hyperactive with clonus. Deep tendon reflexes should be 1+ or 2+. Reflexes that are brisker than average and hyperactive reflexes (3+ to 4+) suggest preeclampsia and must be reported to the physician. It is not necessary to contact the physician, because the finding is normal. Likewise, rechecking the client’s reflexes after ambulation and performing active and passive ROM exercises incorrect and unnecessary actions.

Test-Taking Strategy: Use the process of elimination. Eliminate the options that are comparable or alike in that they involve the rechecking of reflexes. To select from the remaining options, note that the finding is normal, which will direct you to the correct option. If you had difficulty with this question, review the scale for rating deep tendon reflexes.

Reference: McKinney, E., James, S., Murray, S., & Ashwill, J. (2009). Maternal-child nursing (3rd ed., pp. 623, 624). St. Louis: Elsevier.

Cognitive Ability: Applying

Client Needs: Physiological Integrity

Integrated Process: Nursing Process/Implementation

Content Area: Maternity/Antepartum Awarded 0.0 points out of 1.0 possible points. 70.ID: 383713169 After delivering a normal, healthy newborn, a client complains of severe pelvic pain and a feeling of extreme fullness in the vagina, and uterine inversion is suspected. For which immediate intervention does the nurse prepare the client? Hysterectomy Insertion of an indwelling catheter Administration of oxytocin (Pitocin) Replacement of the uterus through the vagina into a normal position Correct Rationale: If uterine inversion is suspected, the nurse immediately prepares the client for replacement of the uterus through the vagina. If this is not possible or effective, laparotomy with replacement is performed. Hysterectomy may be required. Intravenous lines are established to allow rapid fluid and blood replacement. A tocolytic medication or general anesthesia is usually needed to relax the uterus enough to replace it. Once the uterus has been replaced and the placenta removed, oxytocin is given to induce uterine contraction and control blood loss. To help prevent trapping of the inverted fundus in the cervix, oxytocin is not given until the uterus has been repositioned. An indwelling catheter is often inserted to aid monitoring of fluid balance and keep the bladder empty so that the uterus can contract fully, but this is not the action that would be taken immediately.

Test-Taking Strategy: Note the strategic word "immediate" in the query of the question. Focusing on the diagnosis of uterine inversion and thinking about what this complication involves will direct you to the correct option as the immediate action. Review immediate interventions for the client with uterine inversion if you had difficulty with this question.

Reference: Lowdermilk, D., Perry, S., & Cashion, K. (2010). Maternity nursing (8th ed., p. 728). St. Louis: Mosby.

Cognitive Ability: Applying

Client Needs: Physiological Integrity

Integrated Process: Nursing Process/Implementation

Content Area: Maternity/Postpartum Awarded 0.0 points out of 1.0 possible points. 71.ID: 383703685 A nurse in the postpartum unit is caring for a client who delivered a healthy newborn 12 hours ago. The nurse checks the client's temperature and notes that it is 100.4° F (38° C). On the basis of this finding, the nurse would: Notify the physician Recheck the temperature in 4 hours Correct Encourage the client to breastfeed the newborn Institute strict bedrest for the client and notify the physician Rationale: A temperature of 100.4° F (38° C) is common during the 24 hours after childbirth and may be the result of dehydration or normal postpartum leukocytosis. If the increased temperature persists for more than 24 hours or exceeds 100.4° F (38° C), infection is a possibility, and the fever is reported. The nurse would recheck the temperature in 4 hours. There is no reason to restrict place the client to strict bedrest or to notify the physician. Although the client would be encouraged to breastfeed her newborn, this action is unrelated to the client’s temperature.

Test-Taking Strategy: Use the process of elimination. Eliminate the options that are comparable or alike in that they involve notifying the physician. To select from the remaining options, focus on the data in the question. Recalling that a temperature of 100.4° F (38° C) is common during the 24 hours after childbirth will direct you to the correct option. Review normal and abnormal findings in the postpartum period if you had difficulty with this question.

Reference: Lowdermilk, D., Perry, S., & Cashion, K. (2010). Maternity nursing (8th ed., p. 390). St. Louis: Mosby.

Cognitive Ability: Applying

Client Needs: Physiological Integrity

Integrated Process: Nursing Process/Implementation

Content Area: Maternity/Postpartum Awarded 0.0 points out of 1.0 possible points. 72.ID: 383704541 A nurse checking the fundus of a postpartum woman notes that it is above the expected level, at the umbilicus, and that it has shifted from the midline position to the right. The nurse’s initial action should be: Documenting the findings Encouraging the woman to walk Helping the woman empty her bladder Correct Massaging the fundus gently until it becomes firm Rationale: In the postpartum period, the fundus should be firmly contracted and at or near the level of the umbilicus. If the uterus is found to be higher than the expected level or shifted from the midline position (usually to the right), the bladder may be distended. The location of the fundus should be rechecked after the woman has emptied her bladder. If the fundus is difficult to locate or is boggy (soft), the nurse stimulates the uterine muscle to contract by gently massaging the uterus. Encouraging the woman to walk is inappropriate at this time. The nurse would document fundal position, consistency, and height and any other interventions taken (e.g., uterine massage) after the woman has emptied her bladder.

Test-Taking Strategy: Use the process of elimination and note the strategic words "initial action." Focusing on the data in the question and noting the words “shifted from the midline position to the right” will direct you to the correct option. Review expected findings in regard to fundal height and position in the postpartum period if you had difficulty with this question.

Reference: Lowdermilk, D., Perry, S., & Cashion, K. (2010). Maternity nursing (8th ed., p. 380). St. Louis: Mosby.

Cognitive Ability: Applying

Client Needs: Physiological Integrity

Integrated Process: Nursing Process/Implementation

Content Area: Maternity/Postpartum Awarded 0.0 points out of 1.0 possible points. 73.ID: 383712414 A nurse is preparing to care for a client who was admitted to the antepartum unit at 34 weeks' gestation after an episode of vaginal bleeding resulting from total placenta previa. In report, the nurse is told that the client's vital signs are stable, that the fetal heart rate is 140 beats/min with a reassuring pattern, and that both the client and her husband are anxious about the condition of the fetus. On reviewing the client's plan of care, which client concern does the nurse identify as the priority at this time? Anxiety Correct Premature grief Fluid volume loss Fluid volume overload Rationale: Anxiety is vague uneasiness or discomfort that warns of trouble and enables an individual to approach and deal with the threat. Fluid volume loss indicates a hypovolemic state, whereas fluid volume overload indicates a hypervolemic state. Premature grief is a state in which an individual grieves before an actual loss. There is no information in the question to indicate that fluid volume loss, fluid volume overload, or premature grief are factors for concern.

Test-Taking Strategy: Use the process of elimination. Noting the words "anxious" and "at this time" in the question will direct you to the correct option. There is no information in the question to indicate that the concerns in the other options are applicable. Review the defining characteristics of anxiety if you had difficulty with this question.

Reference: Lowdermilk, D., Perry, S., & Cashion, K. (2010). Maternity nursing (8th ed., pp. 200, 590). St. Louis: Mosby.

Cognitive Ability: Analyzing

Client Needs: Psychosocial Integrity

Integrated Process: Nursing Process/Analysis

Content Area: Maternity/Antepartum Awarded 0.0 points out of 1.0 possible points. 74.ID: 383702995 A nurse reviews the laboratory results of a hospitalized pregnant client with a diagnosis of sepsis who is at risk for disseminated intravascular coagulopathy (DIC). Which laboratory finding would indicate to the nurse that DIC has developed in the client? Increased platelet count Shortened prothrombin time Positive result on d-dimer study Correct Decreased fibrin-degradation products Rationale: DIC is a life-threatening defect in coagulation. As plasma factors are consumed, the circulating blood becomes deficient in clotting factors and unable to clot. Even as anticoagulation is occurring, inappropriate coagulation is also taking place in the microcirculation, and tiny clots form in the smallest blood vessels, blocking blood flow to the organs and causing ischemia. Laboratory studies help establish a diagnosis. The fibrinogen value and platelet count are usually decreased, prothrombin and activated partial thromboplastin times may be prolonged, and levels of fibrin degradation products (the most sensitive measurement) are increased. The d-dimer study is used to confirm the presence of fibrin split products; a positive result is indicative of DIC.

Test-Taking Strategy: Specific knowledge regarding the laboratory findings in DIC is needed to answer this question. Using the process of elimination and thinking about the pathophysiology associated with DIC will direct you to the correct option. Review the laboratory findings noted in DIC if you had difficulty with this question.

Reference: McKinney, E., James, S., Murray, S., & Ashwill, J. (2009). Maternal-child nursing (3rd ed., p. 1299). St. Louis: Elsevier.

Cognitive Ability: Analyzing

Client Needs: Physiological Integrity

Integrated Process: Nursing Process/Analysis

Content Area: Maternity/Antepartum Awarded 0.0 points out of 1.0 possible points. 75.ID: 383706674 A nurse is caring for a client with a diagnosis of abruptio placentae. For which early signs of hypovolemic shock does the nurse closely monitor the client? Select all that apply. Tachycardia Correct Cool, clammy skin Decreased respiratory rate Diminished peripheral pulses Correct Urine output of less than 30 mL/hr Rationale: When hypovolemic shock develops, the body attempts to compensate for decreased blood volume and to maintain oxygenation of essential organs by increasing the rate and effort of the heart and lungs by shunting blood from less essential organs, such as the skin and extremities, to more essential ones, such as the brain and kidneys. This compensatory mechanism results in the early signs and symptoms of hypovolemic shock, which include tachycardia, diminished peripheral pulses, normal or slightly decreased blood pressure, increased respiratory rate, and cool, pale skin and mucous membranes. The compensatory mechanism fails if hypovolemic shock progresses and there is insufficient blood to perfuse the brain, heart, and kidneys. Later signs of hypovolemic shock include decreasing blood pressure, pallor, cold and clammy skin, and urine output of less than 30 mL/hr.

Test-Taking Strategy: Note the strategic word "early." Recall that the heart rate and respiratory rate increase as the body attempts to maintain oxygenation to essential organs. This will direct you to the correct options. Review the early and late signs of hypovolemic shock if you had difficulty with this question.

Reference: McKinney, E., James, S., Murray, S., & Ashwill, J. (2009). Maternal-child nursing (3rd ed., p. 618). St.Louis: Elsevier.

Cognitive Ability: Analyzing

Client Needs: Physiological Integrity

Integrated Process: Nursing Process/Assessment

Content Area: Maternity/Antepartum Awarded 0.0 points out of 1.0 possible points. 76.ID: 383705013 A nurse developing a nursing care plan for a client with abruptio placentae includes initial nursing measures to be implemented in the event of the development of shock. After contacting the physician, which of the following does the nurse specify as the first action in the event of shock? Checking the client’s urine output Inserting an intravenous (IV) line Obtaining informed consent for a cesarean delivery Placing the client in a lateral position with the bed flat Correct Rationale: If the client exhibits signs of hypovolemic shock, the nurse would contact the physician. The nurse would monitor fetal status closely and take action to minimize the effects of hypovolemic shock and promote tissue oxygenation. The client would be placed in a lateral position, with the head of the bed flat to increase cardiac return and thus increase circulation and oxygenation of the placenta and other vital organs. After positioning the client, the nurse would insert IV lines in accordance with the physician’s prescriptions and hospital protocols so that blood and replacement fluids may be administered. Quick preparation of the client for cesarean delivery may be necessary, but obtaining informed consent for the procedure is not the first action. Urine output is monitored to ensure an output of at least 30 mL/hr but, again, this is not the first action.

Test-Taking Strategy: Note the strategic word "first." Use the process of elimination and your knowledge of the ABCs — airway, breathing, and circulation. Repositioning the client will increase circulation and oxygenation of the placenta and other vital organs. Review the first actions to take for shock if you had difficulty with this question.

Reference: McKinney, E., James, S., Murray, S., & Ashwill, J. (2009). Maternal-child nursing (3rd ed., p. 619). St. Louis: Elsevier.

Cognitive Ability: Applying

Client Needs: Physiological Integrity

Integrated Process: Nursing Process/Implementation

Content Area: Delegating/Prioritizing Awarded 0.0 points out of 1.0 possible points. 77.ID: 383705051 A postpartum nurse provides information to a client who has delivered a healthy newborn about normal and abnormal characteristics of lochia. Which of the following findings does the nurse tells the client to report to the physician? Pink lochia on postpartum day 4 White lochia on postpartum day 11 Bloody lochia on postpartum day 2 Reddish lochia on postpartum day 8 Correct Rationale: Lochia is the postdelivery vaginal discharge from the uterus consisting of blood from the vessels of the placental site and debris from the deciduas. Rubra is the bright-red lochial discharge that appears from delivery day to day 3. Serosa is the brownish-pink lochial discharge that appears on days 4 to 10. Alba is the white lochial discharge that appears on days 10 to 14. Reddish lochia on postpartum day 8 is an abnormal finding and would be reported to the physician.

Test-Taking Strategy: Use the process of elimination and recall the characteristics of lochia during the first 2 weeks after delivery. Recalling that the lochia changes from bloody to brownish-pink and then to white will direct you to the correct option. Reddish lochia on postpartum day 8 is abnormal. Review the expected characteristics of lochia if you had difficulty with this question.

Reference: Lowdermilk, D., Perry, S., & Cashion, K. (2010). Maternity nursing (8th ed., pp. 380, 384, 385). St. Louis: Mosby.

Cognitive Ability: Applying

Client Needs: Health Promotion and Maintenance

Integrated Process: Teaching and Learning

Content Area: Maternity/Postpartum Awarded 0.0 points out of 1.0 possible points. 78.ID: 383712001 A nurse in a physician's office is conducting a 2-week postpartum assessment of a client. During abdominal assessment, the nurse is unable to palpate the uterine fundus. This finding would prompt the nurse to: Document the findings Correct Ask the physician to see the client immediately Ask another nurse to check for the uterine fundus Place the client in the supine position for 5 minutes, then recheck the abdomen Rationale: Involution is the progressive descent of the uterus into the pelvic cavity after delivery. Twenty-four hours after birth, descent of the fundus begins at a rate of approximately 1 fingerbreadth, or approximately 1 cm, per day. By the 10th to 14th day, the fundus is in the pelvic cavity and cannot be palpated abdominally. Asking the physician to see the client immediately, having another nurse check for the uterine fundus, and placing the client in the supine position for 5 minutes and rechecking the abdomen are all incorrect and unnecessary actions in light of the assessment finding.

Test-Taking Strategy: Knowledge regarding the process of involution is required to answer this question. Focusing on the data in the question and noting that the client gave birth 2 weeks earlier will direct you to the correct option. Review the process of involution if you had difficulty with this question.

Reference: Lowdermilk, D., Perry, S., & Cashion, K. (2010). Maternity nursing (8th ed., p. 384). St. Louis: Mosby.

Cognitive Ability: Applying

Client Needs: Physiological Integrity

Integrated Process: Nursing Process/Implementation

Content Area: Maternity/Postpartum Awarded 0.0 points out of 1.0 possible points. 79.ID: 383707947 A maternity nurse providing an education session to a group of expectant mothers describes the purpose of the placenta. Which statement by one of the women attending the session indicates a need for further discussion of the purpose of the placenta? "Many of my antibodies are passed through the placenta." "The placenta maintains the body temperature of my baby." Correct "Glucose, vitamins, and electrolytes pass through the placenta." "It provides an exchange of oxygen and carbon dioxide between me and my baby." Rationale: Many of the immunoglobulin G (IgG) class of antibodies are passed from mother to fetus through the placenta. Glucose, fatty acids, vitamins, and electrolytes pass readily across the placenta; glucose is the major source of energy for fetal growth and metabolic activities. The placenta provides an exchange of nutrients and waste products between the mother and fetus. Oxygen and carbon dioxide pass through the placental membrane by way of simple diffusion. The amniotic fluid surrounds, cushions, and protects the fetus and maintains the body temperature of the fetus.

Test-Taking Strategy: Use the process of elimination, noting the strategic words "need for further discussion." These words indicate a negative event query and the need to select the incorrect statement. Recall the anatomic location of the placenta and its purpose to direct you to the correct option. Remember, amniotic fluid surrounds, cushions, protects, and maintains the body temperature of the fetus. If you had difficulty with this question, review the structure and function of the placenta and amniotic fluid.

Reference: Lowdermilk, D., Perry, S., & Cashion, K. (2010). Maternity nursing (8th ed., pp. 155, 156). St. Louis: Mosby.

Cognitive Ability: Evaluating

Client Needs: Physiological Integrity

Integrated Process: Teaching and Learning

Content Area: Maternity/Antepartum Awarded 0.0 points out of 1.0 possible points. 80.ID: 383708578 A client arrives at the clinic for her first prenatal assessment. The client tells the nurse that the first day of her last menstrual period (LMP) was September 25, 2012. Using Nagele’s rule, the nurse determines that the estimated date of delivery (EDD) is: June 2, 2013 July 2, 2013 Correct October 2, 2013 September 18, 2013 Rationale: Accurate use of Nagele’s rule requires that the woman have a regular 28-day menstrual cycle. To calculate the EDD with the use of this rule, the nurse would subtract 3 months from the date of the first day of her LMP, add 7 days, and then adjust the year. First day of the LMP, September 25, 2012; subtract 3 months, June 25, 2012; add 7 days, July 2, 2012; add 1 year, July 2, 2013.

Test-Taking Strategy: Knowledge of how to use Nagele’s rule is required to answer this question. Use caution when following the steps to determine the estimated date of delivery. Read all of the options carefully, noting the dates in the options and remembering that there are 30 days in the month of June. Review Nagele’s rule if you had difficulty with this question.

Reference: Lowdermilk, D., Perry, S., & Cashion, K. (2010). Maternity nursing (8th ed., pp. 191, 192). St. Louis: Mosby.

Cognitive Ability: Understanding

Client Needs: Physiological Integrity

Integrated Process: Nursing Process/Assessment

Content Area: Maternity/Antepartum Awarded 0.0 points out of 1.0 possible points. 81.ID: 383709200 A client has been given a prescription for lovastatin (Mevacor). Which of the following foods does the nurse instruct the client to limit consumption of while taking this medication? Steak Correct Spinach Chicken Oranges Rationale: Lovastatin is a lipid-lowering agent. The client is instructed to consume foods that are low in fat, cholesterol, and complex sugars. The item highest in fat here is steak; therefore the client should limit the intake of steak. Fruits, vegetables, and chicken are low in fat.

Test-Taking Strategy: Use the process of elimination, noting the strategic word "limit." Recalling that lovastatin is a lipid-lowering agent will direct you to the correct option. Review this medication and foods high in fat if you had difficulty with this question.

Reference: Kee, J., Hayes, E., & McCuistion, L. (2009). Pharmacology: A nursing process approach (6th ed., p. 697). Philadelphia: Saunders.

Cognitive Ability: Applying

Client Needs: Physiological Integrity

Integrated Process: Teaching and Learning

Content Area: Pharmacology Awarded 0.0 points out of 1.0 possible points. 82.ID: 383711483 A nurse is reviewing the laboratory results of a client with ovarian cancer who is undergoing chemotherapy. Which finding indicates to the nurse that the client is experiencing an adverse effect of the chemotherapy? Sodium 140 mEq/L Hemoglobin 12.5 g/dL Blood urea nitrogen (BUN) 20 mg/dL White blood cell count of 2500 cells/mm3 Correct Rationale: The normal white blood cell count ranges from 5,000 to 10,000/mm3. A white blood cell count of 2500/mm3 is low and puts the client at risk for infection. All of the other values are within normal limits. The normal sodium level is 135 to 145 mEq/L. The normal hemoglobin level ranges from 12 to 14 g/dL, depending on whether the client is male or female. The normal BUN concentration ranges from 10 to 20 mg/dL.

Test-Taking Strategy: Use the process of elimination and knowledge of normal laboratory values. Eliminate the options that are comparable or alike in that they are normal values. The correct option is the only value that is not normal. Review the adverse effects of chemotherapy and normal laboratory values if you had difficulty with this question.

Reference: Ignatavicius, D., & Workman, M. (2010). Medical-surgical nursing: Patient-centered collaborative care (6th ed., pp. 426, 565). St. Louis: Saunders.

Cognitive Ability: Analyzing

Client Needs: Physiological Integrity

Integrated Process: Nursing Process/Assessment

Content Area: Adult Health/Oncology Awarded 0.0 points out of 1.0 possible points. 83.ID: 383705007 Which finding in a client’s history indicates the greatest risk of cervical cancer to the nurse? Nulliparity Early menarche Multiple sexual partners Correct Hormone-replacement therapy Rationale: Risk factors for cervical cancer include multiple sexual partners, a history of human papillomavirus infection, first sexual intercourse before the age of 16, cigarette smoking, environmental tobacco smoke exposure, and use of oral contraceptives for more than 5 years. Nulliparity, early menarche, and the use of hormone-replacement therapy are risk factors for ovarian rather than cervical cancer.

Test-Taking Strategy: Focus on the subject, cervical cancer. Reviewing each option carefully and focusing on the subject will help you answer correctly. Review the risk factors for cervical cancer if you had difficulty with this question.

Reference: Ignatavicius, D., & Workman, M. (2010). Medical-surgical nursing: Patient-centered collaborative care (6th ed., p. 1700). St. Louis: Saunders.

Cognitive Ability: Analyzing

Client Needs: Health Promotion and Maintenance

Integrated Process: Nursing Process/Assessment

Content Area: Adult Health/Oncology Awarded 0.0 points out of 1.0 possible points. 84.ID: 383703613 A nurse caring for a woman in labor is reading the fetal monitor tracing (see figure). How does the nurse interpret this finding?

 Umbilical cord compression Pressure on the fetal head during a contraction Uteroplacental insufficiency during a contraction Correct Inadequate pacemaker activity of the fetal heart Rationale: The observation that the has nurse noted in this tracing is late decelerations. Late decelerations constitute an ominous pattern in labor because they suggest uteroplacental insufficiency, possibly associated with a contraction. Early decelerations result from pressure on the fetal head during a contraction. Variable decelerations suggest umbilical cord compression. The term short-term variability refers to the difference between successive heartbeats, indicating that the natural pacemaker function of the fetal heart is working properly.

Test-Taking Strategy: Examine the fetal monitor tracing and see that it shows late decelerations. Recalling that the presence of late decelerations suggests uteroplacental insufficiency will direct you to the correct option. Review the significance of late decelerations on the fetal monitoring strip if you had difficulty with this question.

Reference: Lowdermilk, D., Perry, S., & Cashion, K. (2010). Maternity nursing (8th ed., pp. 325, 326). St. Louis: Mosby.

Cognitive Ability: Analyzing

Client Needs: Physiological Integrity

Integrated Process: Nursing Process/Analysis

Content Area: Maternity/Intrapartum Awarded 0.0 points out of 1.0 possible points. 85.ID: 383708566 A client who has undergone abdominal hysterectomy asks the nurse when she will be able to resume sexual intercourse. The nurse tells the client that sexual intercourse may be resumed: At any time after the surgery When menstruation resumes When pelvic sensation and response to stimuli return In about 6 weeks, when the vaginal vault is satisfactorily healed Correct Rationale: After abdominal hysterectomy, the client is instructed to avoid sexual intercourse until the vaginal vault is satisfactorily healed. This takes about 6 weeks. A woman who has undergone this procedure must adjust to changes in the nature of pelvic sensations and stimuli during sexual intercourse; however, this is not related to when sexual intercourse may be resumed. The client will not have menstrual periods after abdominal hysterectomy.

Test-Taking Strategy: Focus on the subject, when sexual intercourse may be resumed. Think about the type of surgery addressed in the question and use general principles of postoperative home care instructions to find the correct option. Review home care instructions after hysterectomy if you had difficulty with this question.

Reference: Ignatavicius, D., & Workman, M. (2010). Medical-surgical nursing: Patient-centered collaborative care (6th ed., p. 1698). St. Louis: Saunders.

Cognitive Ability: Applying

Client Needs: Physiological Integrity

Integrated Process: Teaching and Learning

Content Area: Reproductive Awarded 0.0 points out of 1.0 possible points. 86.ID: 383703611 A nurse is preparing to care for a client who has undergone abdominal hysterectomy for the treatment of endometrial cancer. The nurse determines that the priority in the 24 hours after surgery is: Monitoring the client for signs of returning peristalsis Instructing the client in dietary changes to prevent constipation Encouraging the client to deep-breathe, cough, and use an incentive spirometer Correct Encouraging the client to talk about the effects of the surgery on her femininity and sexuality Rationale: Care after abdominal hysterectomy includes maintenance of a patent airway, promotion of circulation and oxygenation, promotion of comfort, monitoring of output and drainage, promotion of elimination, and discharge teaching with regard to medications and therapeutic regimens. The priority is the maintenance of a patent airway and promotion of oxygenation and circulation. Monitoring the client for signs of returning peristalis, instructing her in dietary habits to prevent constipation, and encouraging her to talk about the effects of her surgery are also components of care after this surgery but are of lower priority than encouraging the client to deep-breathe, cough, and use an incentive spirometer.

Test-Taking Strategy: Note the strategic word "priority." Use the ABCs — airway, breathing, and circulation. This will direct you to the correct option. Review care of the client who has undergone abdominal hysterectomy if you had difficulty with this question.

Reference: Ignatavicius, D., & Workman, M. (2010). Medical-surgical nursing: Patient-centered collaborative care (6th ed., p. 1696). St. Louis: Saunders.

Cognitive Ability: Analyzing

Client Needs: Physiological Integrity

Integrated Process: Nursing Process/Planning

Content Area: Reproductive Awarded 0.0 points out of 1.0 possible points. 87.ID: 383705084 A nurse is caring for a client with community-acquired pneumonia who is being treated with levofloxacin (Levaquin). For which of the following findings, indicating an adverse reaction to the medication, does the nurse monitor the client? Fever Correct Dizziness Flatulence Drowsiness Rationale: Levofloxacin is an antibiotic of the fluoroquinolone class. Pseudomembranous colitis is an adverse reaction associated with the use of this medication. It is characterized by severe abdominal pain or cramps, severe watery diarrhea, and fever. Dizziness, flatulence, and drowsiness are side effects of the medication.

Test-Taking Strategy: Use the process of elimination. Focusing on the subject, an adverse effect, will direct you to the correct option. Review the adverse effects of levofloxacin if you had difficulty with this question.

References: Hodgson, B., & Kizior, R. (2010). Saunders nursing drug handbook 2010 (p. 671). St. Louis: Saunders.
Skidmore-Roth, L. (2010). Mosby’s nursing drug reference (23rd ed., p. 640). St. Louis: Mosby.

Cognitive Ability: Analyzing

Client Needs: Physiological Integrity

Integrated Process: Nursing Process/Assessment

Content Area: Pharmacology Awarded 0.0 points out of 1.0 possible points. 88.ID: 383708598 A nurse is providing instructions to a client with glaucoma who will be using acetazolamide (Diamox) daily. Which of the following findings, an adverse effect, does the nurse instruct the client to report to the physician? Nausea Dark urine Correct Urinary frequency Decreased appetite Rationale: Acetazolamide is a carbonic anhydrase inhibitor. Nephrotoxicity and hepatotoxicity may occur, manifesting as dark urine and stools, lower back pain, jaundice, dysuria, crystalluria, renal colic, and calculi. Bone marrow depression may also occur as an adverse effect. Nausea, urinary frequency, and decreased appetite are side effects of the medication.

Test-Taking Strategy: Focus on the strategic words "adverse effect" in the query of the question. Remembering that acetazolamide is nephrotoxic and hepatotoxic will direct you to the correct option. Review the adverse effects of acetazolamide if you had difficulty with this question.

Reference: Lehne, R. (2010). Pharmacology for nursing care (7th ed., p. 688). St. Louis: Saunders.

Cognitive Ability: Applying

Client Needs: Physiological Integrity

Integrated Process: Teaching and Learning

Content Area: Pharmacology Awarded 0.0 points out of 1.0 possible points. 89.ID: 383703657 A nurse is caring for a client with a cuffed endotracheal tube who is undergoing mechanical ventilation. Which intervention to prevent a tracheoesophageal fistula, a complication of this type of tube, does the nurse implement? Frequent suctioning Maintaining cuff pressure Correct Maintaining mechanical ventilation settings Alternating the use of a cuffed tube with a cuffless tube on a daily basis Rationale: Necrosis of the tracheal wall caused by the cuff of an endotracheal tube can lead to the development of an opening between the posterior trachea and esophagus, a complication known as tracheoesophageal fistula. The fistula allows air to escape into the stomach, resulting in abdominal distention. It also leads to the aspiration of gastric contents. To prevent this complication, the nurse must maintain cuff pressure, monitor the amount of air needed for cuff inflation, and help the client progress to a deflated cuff or cuffless tube as soon as possible as prescribed by the physician. Suctioning should be performed only as needed; frequent suctioning can cause mucosal damage. Maintenance of mechanical ventilation settings ensures that the client is adequately oxygenated, but this intervention is not a measure for the prevention of tracheoesophageal fistula. Alternating the use of a cuffed tube and a cuffless tube on a daily basis is incorrect, because the endotracheal tube would not be removed and replaced on a daily basis.

Test-Taking Strategy: Use your knowledge of anatomy and medical terminology to answer this question. A fistula is an artificial opening. The term "tracheoesophageal" indicates "trachea to esophagus." Next think about the intervention that will help prevent this complication. Note the word “pressure” in the correct option. Review the measures for the prevention of tracheoesophageal fistula if you had difficulty with this question.

Reference: Ignatavicius, D., & Workman, M. (2010). Medical-surgical nursing: Patient-centered collaborative care (6th ed., p. 581). St. Louis: Saunders.

Cognitive Ability: Applying

Client Needs: Physiological Integrity

Integrated Process: Nursing Process/Implementation

Content Area: Adult Health/Respiratory Awarded 0.0 points out of 1.0 possible points. 90.ID: 383712418 A nurse is preparing to insert a nasogastric tube into a client. In which position does the nurse place the client before inserting the tube? Correct Rationale: A nasogastric tube is inserted through the nose and into the stomach for the purpose of gastric decompression or feeding the client. The client is placed in the Fowler position before insertion of the tube to promote comfort and easy insertion. A flat position may be used for clients who are hypotensive. In the reverse Trendelenburg position, the entire bed frame is tilted with the foot of the bed down and may be used to promote gastric emptying or prevent esophageal reflux. A trendelenburg position is one in which the entire bed frame is tilted with the head of the bed down and may be used for postural drainage or to facilitate venous return in clients with poor peripheral perfusion.

Test-Taking Strategy: Use the process of elimination and focus on the subject, the position for insertion of anasogastric tube. Visualizing this procedure will direct you to the correct option. Review the procedure for inserting a nasogastric tube if you had difficulty with this question.

Reference: Potter, P. & Perry, A. (2009). Fundamentals of nursing (7th ed. p. 898). St. Louis: Mosby.

Cognitive Ability: Applying

Client Needs: Safe and Effective Care Environment

Integrated Process: Nursing Process/Implementation

Content Area: Adult Health/Gastrointestinal Awarded 0.0 points out of 1.0 possible points. 91.ID: 383706686 Aneurysm precautions are prescribed for a client with a cerebral aneurysm. Which interventions does the nurse implement? Select all that apply.

  Keeping the room slightly darkened Correct Placing the client in a room with a quiet roommate Encouraging isometric exercises if bed rest is prescribed Monitoring the client for changes in alertness or mental status Correct Restricting visits to close family members and significant others and keeping visits short Correct Rationale: A cerebral aneurysm is a thin-walled outpouching or dilation of an artery of the brain. When an aneurysm ruptures, bleeding into the subarachnoid space usually ensues. Aneurysm precautions are implemented to maintain a stable perfusion pressure and help prevent rupture. The client is placed in a quiet private room without a telephone. The room is kept slightly darkened, and bright lighting is avoided. Stool softeners are administered to help keep the client from straining during defecation. The client is monitored for changes in alertness or mental status. Visitors are restricted to close family members and significant others, and visits are kept short. Any contact with visitors who upset or excite the client is avoided. Isometric exercises and use of the Valsalva maneuver are avoided because both increase intrathoracic and intraabdominal pressure. Bed rest with the head of the bed elevated 30 degrees may be prescribed. Some physicians permit bathroom privileges for selected clients. If the client is allowed out of bed, the nurse stresses the importance of not bending over.

Test-Taking Strategy: Focus on the client’s diagnosis and the subject, aneurysm precautions. Note the strategic word “cerebral.” Thinking about the life-threatening complications that can occur as a result of this condition will assist you in identifying the correct nursing interventions. Review the interventions to be taken under aneurysm precautions if you had difficulty with this question.

References: Ignatavicius, D., & Workman, M. (2010). Medical-surgical nursing: Patient-centered collaborative care (6th ed., pp. 1031, 1039). St. Louis: Saunders.
Aneurysm precautions

Cognitive Ability: Applying

Client Needs: Physiological Integrity

Integrated Process: Nursing Process/Implementation

Content Area: Adult Health/Neurological Awarded 0.0 points out of 1.0 possible points. 92.ID: 383708592 A nurse, providing information to a client who has just been found to have diabetes mellitus, gives the client a list of symptoms of hypoglycemia. Which of the following answers by the client, on being asked to list the symptoms, tells the nurse that the client understands the information? Select all that apply. Hunger Correct Weakness Correct Blurred vision Correct Increased thirst Increased urine output Rationale: The manifestations of diabetes mellitus (hyperglycemia) include polydipsia, polyuria, and polyphagia. Symptoms of hypoglycemia include weakness, double vision, blurred vision, hunger, tachycardia, and palpitations.

Test-Taking Strategy: Focus on the subject, symptoms of hypoglycemia. Recalling the “three P’s” — polyuria, polydipsia, and polyphagia — associated with diabetes mellitus and hyperglycemia will direct you to the correct options. Also, think about the pathophysiology of hypoglycemia to help answer correctly. Review the symptoms of hypoglycemia and hyperglycemia if you had difficulty with this question.

Reference: Ignatavicius, D., & Workman, M. (2010). Medical-surgical nursing: Patient-centered collaborative care (6th ed., p. 1507). St. Louis: Saunders.

Cognitive Ability: Evaluating

Client Needs: Physiological Integrity

Integrated Process: Nursing Process/Evaluation

Content Area: Adult Health/Endocrine Awarded 0.0 points out of 1.0 possible points. 93.ID: 383705043 A nurse is planning to teach a crutch gait to a client who will be using wooden axillary crutches. The nurse reviews the physician's instructions, understanding that the gait was selected after assessment of the client's: Physical and functional abilities Correct Feelings about restricted mobility Uneasiness about using the crutches Understanding of the need for increased mobility Rationale: A crutch gait is selected after an assessment of the client’s physical and functional abilities and the disease or injury that resulted in the need for crutches. Assessing the client’s uneasiness about using crutches, feelings about being mobility restricted, and understanding of the need for increased mobility are also important considerations, but assessment of the client’s physical and functional abilities is most important in ensuring safety in ambulation.

Test-Taking Strategy: Use Maslow’s Hierarchy of Needs theory to answer the question. Remember that physiological needs are the priority, followed by safety needs and then psychosocial needs. This will direct you to the correct option. Review interventions related to crutch-aided walking if you had difficulty with this question.

References: Ignatavicius, D., & Workman, M. (2010). Medical-surgical nursing: Patient-centered collaborative care (6th ed., p. 1193). St. Louis: Saunders.
Potter, P., & Perry, A. (2009). Fundamentals of nursing (7th ed., pp. 803-807). St. Louis: Mosby.

Cognitive Ability: Understanding

Client Needs: Safe and Effective Care Environment

Integrated Process: Nursing Process/Assessment

Content Area: Adult Health/Musculoskeletal Awarded 0.0 points out of 1.0 possible points. 94.ID: 383711495 A client who has undergone extensive gastrointestinal surgery is receiving intermittent enteral tube feedings that will be continued after he is discharged home. When the nurse tells the client that he will be taught how to administer the feedings, the client states, "I don't think I’ll be able to do these feedings by myself." Which response by the nurse is appropriate? "Have you told your doctor how you feel?" "Tell me more about your concerns regarding the tube feedings." Correct "Don't worry. We’ll keep you in the hospital until you’re ready to do them by yourself." "We’ll ask the doctor about having a visiting nurse come to your home to give you your feedings." Rationale: A client often has fears about leaving the secure environment of the healthcare facility, where he or she is cared for. This client fears that he will not be able to care for himself at home by administering himself the tube feedings. An open statement such as "Tell me more about..." often elicits valuable information about the client and the client’s concerns. The remaining nursing responses are incorrect because they are nontherapeutic statements that do not address the client’s expressed concern.

Test-Taking Strategy: Use your knowledge of therapeutic communication techniques. Remembering to focus on the client’s feelings and maintain an open channel of communication will direct you to the correct option. Remember to address the client’s feelings first. Review therapeutic communication techniques if you had difficulty with this question.

Reference: Ignatavicius, D., & Workman, M. (2010). Medical-surgical nursing: Patient-centered collaborative care (6th ed., pp. 1401, 1402). St. Louis: Saunders.

Cognitive Ability: Applying

Client Needs: Psychosocial Integrity

Integrated Process: Caring

Content Area: Adult Health/Gastrointestinal Awarded 0.0 points out of 1.0 possible points. 95.ID: 383706605 A client is brought to the emergency department after sustaining smoke inhalation. Humidified oxygen is administered to the client by way of face mask, and arterial blood gases (ABGs) are measured. ABG analysis indicates arterial oxygenation (Pao2) of less than 60 mm Hg. On the basis of the ABG result, the nurse prepares to: Continue monitoring the client Increase the amount of humidified oxygen Continue administering humidified oxygen Assist in intubating the client and beginning mechanical ventilation Correct Rationale: A client who sustains smoke inhalation is immediately treated with 100% humidified oxygen, delivered by way of face mask. Endotracheal intubation with mechanical ventilation is needed if the client exhibits respiratory stridor, crowing, or dyspnea, all of which indicate airway obstruction. Normal arterial oxygenation is 80 to 100 mm Hg. An arterial oxygenation (Pao2) of less than 60 mm Hg is an indication for intubation and mechanical ventilation.

Test-Taking Strategy: Use the process of elimination. Eliminate the options that are comparable or alike in that they involve continued monitoring of the client or continuation of the prescribed treatment. To select from the remaining options, recall the normal arterial oxygen level will direct you to the correct option. Review care of the client who has sustained smoke inhalation if you had difficulty with this question.

Reference: Ignatavicius, D., & Workman, M. (2010). Medical-surgical nursing: Patient-centered collaborative care (6th ed., p. 529). St. Louis: Saunders.

Cognitive Ability: Applying

Client Needs: Physiological Integrity

Integrated Process: Nursing Process/Implementation

Content Area: Critical Care Awarded 0.0 points out of 1.0 possible points. 96.ID: 383706607 A nurse is caring for a client undergoing skeletal traction of the left leg. The client complains of severe pain in the leg. The nurse checks the client's alignment in bed and notes that proper alignment is being maintained. Which of the following actions should the nurse take next? Providing pin care Medicating the client Notifying the physician Correct Removing some weight from the traction Rationale: A client in traction who complains of severe pain may require realignment or may have traction weights that are too heavy. The nurse realigns the client and, if this is ineffective in relieving the pain, notifies the physician. Severe leg pain, once traction has been established, indicates a problem. Provision of pin care is not related to the problem as described. The client should be medicated after an attempt has been made to determine and treat the cause; the cause of the severe pain should be investigated first. The nurse should never remove the weights from the traction without a specific prescription to do so.

Test-Taking Strategy: Note the strategic word "severe" in the question. This indicates that a problem exists. Recalling the causes of severe pain in a client in skeletal traction and noting that the question indicates that the nurse has already ensured that the client is in proper alignment will direct you to the correct option. Review the complications associated with skeletal traction if you had difficulty with this question.

Reference: Ignatavicius, D., & Workman, M. (2010). Medical-surgical nursing: Patient-centered collaborative care (6th ed., p. 1190). St. Louis: Saunders.

Cognitive Ability: Applying

Client Needs: Physiological Integrity

Integrated Process: Nursing Process/Implementation

Content Area: Adult Health/Musculoskeletal Awarded 0.0 points out of 1.0 possible points. 97.ID: 383713100 A clinic nurse is assessing a client who has had a cast applied to the lower left arm 1 week ago. The client tells the nurse that the skin is being irritated by the edges of the cast. What is the appropriate action on the part of the nurse Bivalve the cast Ask the physician to reapply the cast Use a nail file to smooth the rough edges Place small pieces of tape over the rough edges of the cast Correct Rationale: If a client with a cast experiences skin irritation from the edges of the cast, the nurse should petal (place small pieces of tape over) the rough edges of the cast to minimize the irritation. Bivalving is performed if the limb swells occurs and the cast becomes too tight. Using a nail file to smooth the rough edges could cause pieces of the cast to fall into the cast, possibly resulting in the disruption of skin integrity. It is not necessary to contact the physician, and there is no reason to reapply the cast.

Test-Taking Strategy: Use the process of elimination. Contacting the physician is premature and there is no reason for the cast to be reapplied, so you can eliminate these options. To select from the remaining options, focus on the subject and think about the interventions associated with prevention of alterations in skin integrity for a client with a cast, which will direct you to the correct option. If you had difficulty with this question, review care of the client with an extremity cast.

Reference: Ignatavicius, D., & Workman, M. (2010). Medical-surgical nursing: Patient-centered collaborative care (6th ed., p. 1187). St. Louis: Saunders.

Cognitive Ability: Applying

Client Needs: Physiological Integrity

Integrated Process: Nursing Process/Implementation

Content Area: Adult Health/Musculoskeletal Awarded 0.0 points out of 1.0 possible points. 98.ID: 383706060 A client says to the nurse, "My doctor just left. He told me that my abdominal scan showed a mass in my pancreas and that it’s probably cancer. Does this mean I'm going to die?" The nurse interprets the client's initial reaction as: Fear Correct Denial Acceptance Preoccupation with self Rationale: Fear is a response to a threat that is consciously recognized as a danger. In this situation, the client’s reaction is one of fear, and the client verbalizes the object of fear (dying). There is no evidence of denial, acceptance, or preoccupation with self in the client’s statement.

Test-Taking Strategy: Focus on the subject of the question and the client’s statement. The client’s statement clearly identifies fear. Review psychosocial reactions related to a diagnosis if you had difficulty with this question.

Reference: Ignatavicius, D., & Workman, M. (2010). Medical-surgical nursing: Patient-centered collaborative care (6th ed., p. 116). St. Louis: Saunders.

Cognitive Ability: Analyzing

Client Needs: Psychosocial Integrity

Integrated Process: Caring

Content Area: Adult Health/Oncology Awarded 0.0 points out of 1.0 possible points. 99.ID: 383703623 A nurse notes documentation in the client’s medical record indicating that the client has a stage II pressure ulcer. On the basis of this information, which of the following findings does the nurse expect to note? Correct Rationale: A stage I ulcer is characterized by intact skin that is red and does not blanch under external pressure. A stage II ulcer is characterized by nonintact skin. There is partial-thickness skin loss, and the wound may appear as an abrasion, a shallow crater, or a blister. A stage III ulcer is characterized by full-thickness skin loss, and the subcutaneous tissue may be damaged or necrotic. The damage extends down to but not through the underlying tissues. A deep crater–like appearance or eschar is present. A stage IV ulcer is characterized by full-thickness skin loss with extensive destruction, tissue necrosis, or damage to muscle, bone, or supporting structures. Sinus tracts may develop.

Test-Taking Strategy: Use your knowledge of the different stages of pressure ulcer classification to answer this question. Focus on the data in the question and note that the question involves a stage II pressure ulcer. Next, recall that the stages range from I to IV, with stage I, the least extensive, characterized by intact skin and stage IV, characterized by full thickness skin loss, the most extensive. Review the stages of pressure ulcers and their characteristics if you had difficulty with this question.

Reference: Ignatavicius, D., & Workman, M. (2010). Medical-surgical nursing: Patient-centered collaborative care (6th ed., p. 489). St. Louis: Saunders.

Cognitive Ability: Analyzing

Client Needs: Physiological Integrity

Integrated Process: Nursing Process/Assessment

Content Area: Adult Health/Integumentary Awarded 0.0 points out of 1.0 possible points. 100.ID: 383710540 A nurse is providing instruction in how to perform Kegel exercises to a client with stress incontinence. The nurse tells the client to: Always perform the exercises while lying down Expect an improvement in the control of urine in about 1 week Tighten the pelvic muscles for as long as 5 minutes, three or four times a day Tighten the pelvic muscles for a slow count of 10, then relax for a slow count of 10 Correct Rationale: Kegel exercises strengthen the muscles of the pelvic floor. To perform the exercises, the client is taught to tighten the pelvic muscles to a slow count of 10, then relax to a slow count of 10. The client is also instructed to do this exercise 15 times while lying down, sitting up, and standing (a total of 45 repetitions). The client is told that an improvement in the control of urine will be noticed after several weeks of the exercises; some individuals report that improvement takes as long as 3 months.

Test-Taking Strategy: Use the process of elimination. Eliminate the option that includes the closed-ended word "always." Eliminate the option that indicates that improvement will be noted in 1 week; this period is too short. To select from the remaining options, visualize this exercise, which will direct you to the correct option. Also note that 5 minutes is a long time to keep the pelvic muscles tightened. Review the procedure for Kegel exercises if you had difficulty with this question.

Reference: Ignatavicius, D., & Workman, M. (2010). Medical-surgical nursing: Patient-centered collaborative care (6th ed., p. 1564). St. Louis: Saunders.

Cognitive Ability: Applying

Client Needs: Physiological Integrity

Integrated Process: Teaching and Learning

Content Area: Adult Health/Renal Awarded 0.0 points out of 1.0 possible points. 101.ID: 383706066 Ergotamine (Cafergot) is prescribed to a client with cluster headaches. Which of the following occurrences does the nurse tell the client to report to the physician if she experiences them while taking the medication? Cough Fatigue and lethargy Dizziness and fatigue Numbness and tingling of the fingers or toes Correct Rationale: Ergotamine is an antimigraine medication. Prolonged administration or an excessive dosage may produce ergotamine poisoning (ergotism). Symptoms include nausea, vomiting, weakness in the legs, pain in the limb muscles, and numbness and tingling of the fingers and toes. The client is instructed to report these symptoms to the physician if they occur. Cough, fatigue, lethargy, and dizziness are side effects not adverse effects of the medication.

Test-Taking Strategy: Note the strategic words “report to the physician.” Use the ABCs — airway, breathing, and circulation — to identify the correct option. Numbness and tingling of the fingers or toes is a sign of inadequate circulation. Review the adverse effects of ergotamine if you had difficulty with this question.

Reference: Lehne, R. (2010). Pharmacology for nursing care (7th ed., p. 315). St. Louis: Saunders.

Cognitive Ability: Applying

Client Needs: Physiological Integrity

Integrated Process: Teaching and Learning

Content Area: Pharmacology Awarded 0.0 points out of 1.0 possible points. 102.ID: 383703661 A client with post–traumatic stress disorder tells the nurse that he has stopped taking his prescribed medication because he didn't like how the medication was making him feel. Which of the following initial responses by the nurse is appropriate? "That's all right. I’d stop, too, if it made me feel funny." "Tell me more about how the medication was making you feel." Correct "Did you let your doctor know that you stopped taking the medication?" "It doesn't make sense to stop the medication. I don't know why you took it upon yourself to do that." Rationale: The correct response acknowledges the client’s feelings and opens the channel of communication between the nurse and client. "That's all right. I’d stop, too, if it made me feel funny," indicating approval, is a nontherapeutic response and is therefore inappropriate. "Did you let your doctor know that you stopped taking the medication?" may be an appropriate question at some point during the conversation, but it is not the most appropriate initial question. "It doesn't make sense to stop the medication. I don't know why you took it upon yourself to do that" demeans the client.

Test-Taking Strategy: Use your knowledge of therapeutic communication techniques to answer this question. Remembering to focus on the client’s feelings will direct you to the correct option. Review therapeutic communication techniques if you had difficulty with this question.

References: Stuart, G., & Laraia, M. (2009). Principles and practice of psychiatric nursing (9th ed., pp. 27-31). St. Louis: Mosby.
Varcarolis, E., & Halter, M. (2009). Essentials of psychiatric mental health nursing: A communication approach to evidence-based care (p. 466). St. Louis: Saunders.

Cognitive Ability: Applying

Client Needs: Psychosocial Integrity

Integrated Process: Communication and Documentation

Content Area: Mental Health Awarded 0.0 points out of 1.0 possible points. 103.ID: 383711497 A nurse provides information to a client with peripheral vascular disease about ways to limit the disease’s progression. Which of the following measures does the nurse tell the client to take? Select all that apply. Crossing the legs at the ankles only Engaging in exercise such as walking on a daily basis Correct Washing the feet daily with a mild soap and drying them well Correct Inspecting the feet at least once a week for injuries, especially abrasions Using a heating pad on the legs to help keep the blood vessels dilated Rationale: Long-term management of peripheral vascular disease consists of measures that increase peripheral circulation. The client is instructed to avoid crossing the legs at any level because it promotes vasoconstriction. The client is instructed to exercise regularly and is encouraged to walk for 20 minutes each day. Keeping the extremities warm is important; however, heating pads and hot water bottles should not be placed on the extremity. Sensitivity may be diminished in the affected extremity, increasing the risk for burns. Also, direct application of heat increases the oxygen and nutritional requirements of the tissue even further. The client is instructed to wash the feet daily with a mild soap, to dry the feet well, and to inspect the feet daily for injuries or abrasions.

Test-Taking Strategy: Focus on the client’s diagnosis and the strategic words "limit disease progression." Eliminate the option containing the closed-ended word "only." Next eliminate the option that indicates that foot inspection should be performed weekly. Recalling the effects of heat will help you eliminate the option involving the use of a heating pad on the legs. Review client teaching points for peripheral vascular disease if you had difficulty with this question.

Reference: Black, J., & Hawks, J. (2009). Medical-surgical nursing: Clinical management for positive outcomes (8th ed., p. 1313). St. Louis: Saunders.

Cognitive Ability: Applying

Client Needs: Health Promotion and Maintenance

Integrated Process: Teaching and Learning

Content Area: Adult Health/Cardiovascular Awarded 0.0 points out of 1.0 possible points. 104.ID: 383713167 A client with depression is anorexic. Which measure does the nurse take to assist the client in meeting nutritional needs? Providing food and fluid as the client requests Offering high-calorie and high-protein foods and fluids frequently throughout the day Correct Completing the dietary menu for the client to ensure that adequate nutrition is provided Weighing the client daily so that the client may determine whether the nutritional plan is working Rationale: The client should be offered high-calorie and high-protein foods and fluids frequently throughout the day. Small, frequent snacks are more easily tolerated than large plates of food when the client is anorexic. The client should be offered choices of foods and fluids he or she likes, because the client is more likely to consume foods he or she has selected. The client should be weighed weekly, not daily. Weight gain may not be noted daily, which may cause the client to view the interventions to improve nutritional status as useless.

Test-Taking Strategy: Use the process of elimination and focus on the subject of the question, meeting the nutritional needs of a depressed client. Recall that depressed clients often do not feel like eating; therefore, providing food and fluid to the client as the client requests is not helpful. It is inappropriate to complete the dietary menu for the client; the client should be given choices. Next, eliminate the option that involves weighing the client daily. Review care of the client with depression if you had difficulty with this question.

Reference: Varcarolis, E., & Halter, M. (2009). Essentials of psychiatric mental health nursing: A communication approach to evidence-based care (p. 229). St. Louis: Saunders.

Cognitive Ability: Applying

Client Needs: Physiological Integrity

Integrated Process: Nursing Process/Implementation

Content Area: Mental Health Awarded 0.0 points out of 1.0 possible points. 105.ID: 383702981 Disulfiram (Antabuse) is prescribed to a client with an alcohol abuse problem. The nurse provides information about the medication and tells the client: That driving is prohibited while the client is taking the medication To take the medication immediately if the desire to drink alcohol occurs That the effect of the medication ends as soon as the client stops taking the medication That the medication cannot be started until at least 12 hours has elapsed since the client's last ingestion of alcohol Correct Rationale: Disulfiram is an alcohol abuse deterrent prescribed to motivated clients who have shown the ability to stay sober. Driving is not prohibited; however, the client is instructed to use caution when driving and performing other tasks that require alertness. The medication is taken daily (not just when the client has a desire to drink alcohol), and the effects of the medication last 5 days to 2 weeks after the last dose is taken. The medication cannot be started until at least 12 hours has elapsed since the client’s last ingestion of alcohol. Otherwise, an alcohol-disulfiram reaction will occur, with effects consisting of facial flushing, sweating, a throbbing headache, neck pain, tachycardia, respiratory distress, a potentially serious decrease in blood pressure, and nausea and vomiting. This reaction may last 30 to 120 minutes.

Test-Taking Strategy: Use the process of elimination. Eliminate the option containing the word "prohibited." Knowing that the medication is taken daily will assist you in eliminating the option in which the nurse tells the client to take the medication only when the desire to consume alcohol occurs. From the remaining options, recalling that it takes approximately 12 hours for the medication to be removed from the client’s body will direct you to the correct option. Review the actions and uses of disulfiram if you had difficulty with this question.

Reference: Varcarolis, E., & Halter, M. (2009). Essentials of psychiatric mental health nursing: A communication approach to evidence-based care. (p. 355). St. Louis: Saunders.

Cognitive Ability: Applying

Client Needs: Physiological Integrity

Integrated Process: Teaching and Learning

Content Area: Mental Health Awarded 0.0 points out of 1.0 possible points. 106.ID: 383705088 A client with depression is being encouraged to attend art therapy as part of the treatment plan. The client refuses, stating, "I can't draw or paint." Which of the following responses by the nurse is therapeutic? "Why don't you really want to attend?" "This is what your physician has prescribed for you as part of the treatment plan." "OK, let's have you attend music therapy. You can sing there. How does that sound?" "Perhaps you could attend and talk to the other clients and see what they’re drawing and painting." Correct Rationale: The correct response encourages the client to socialize and deflects the client’s attention from the issue of drawing and painting. "Why don't you really want to attend?" challenges the client. "This is what your physician has prescribed for you as part of the treatment plan" ignores the client’s rights. "OK, let's have you attend music therapy. You can sing there. How does that sound?" does not address the client’s concern.

Test-Taking Strategy: Use your knowledge of therapeutic communication techniques. The correct option is the only one that addresses the client’s concern. Review therapeutic communication techniques if you had difficulty with this question.

Reference: Stuart, G., & Laraia, M. (2009). Principles and practice of psychiatric nursing (9th ed., pp. 27-31, 540). St Louis: Mosby.

Cognitive Ability: Applying

Client Needs: Psychosocial Integrity

Integrated Process: Communication and Documentation

Content Area: Mental Health Awarded 0.0 points out of 1.0 possible points. 107.ID: 383711487 A hospitalized female client with mania enters the unit community room and says to a client who is wearing a blue shirt, "Boys in blue are fun to do! Boys in blue are fun to do!" What is the appropriate response by the nurse? "Why are you saying that?" "Stop saying that. It's not true!" "You wouldn't like someone saying that to you. Would you?" "Don’t say that. If you can’t control yourself, we’ll help you." Correct Rationale: The nurse should respond using a firm, calm approach, providing the client with clear expectations. The correct option is the only one that involves a firm, calm approach and offers the client help if she needs it. The other three statements challenge the client.

Test-Taking Strategy: Use your knowledge of therapeutic communication techniques, focusing on the client’s diagnosis. The correct option is the only one that provides assistance to the client. Review therapeutic communication techniques if you had difficulty with this question.

References: Stuart, G., & Laraia, M. (2009). Principles and practice of psychiatric nursing (9th ed., pp. 27-31). St. Louis: Mosby.
Varcarolis, E., & Halter, M. (2009). Essentials of psychiatric mental health nursing: A communication approach to evidence-based care (p. 254). St. Louis: Saunders.

Cognitive Ability: Applying

Client Needs: Psychosocial Integrity

Integrated Process: Communication and Documentation

Content Area: Mental Health Awarded 0.0 points out of 1.0 possible points. 108.ID: 383706052 A nurse working the evening shift is helping clients get ready for sleep. A female client with mania is hyperactive and pacing the hallway. The appropriate nursing action is to: Stay with the client and observe her behavior Take the client to the bathroom and provide her with a warm bath Correct Tell the client that it is time for sleep and that she needs to go to her room Tell the client that other clients are trying to sleep and that she is being disruptive Rationale: For the client with mania, the nurse needs to promote relaxation, rest, and sleep and to minimize manic behavior. The nurse should encourage frequent rest periods during the day and keep the client in areas of low stimulation. At bedtime, the nurse should provide warm baths, soothing music, and medication when indicated. The client should not consume products containing caffeine. Staying with the client and observing her behavior, telling the client that it is time to go to sleep and to go to her room, and telling the client that other clients are trying to sleep and that she is being disruptive do not address the client’s needs and are not measures that will help the client relax and sleep.

Test-Taking Strategy: Focus on the subject, measures to enhance rest and foster sleep. The only option that addresses this subject is the one in which the nurse takes the client to the bathroom and provides her with a warm bath. Review measures to encourage sleep in the client with mania if you had difficulty with this question.

Reference: Varcarolis, E., & Halter, M. (2009). Essentials of psychiatric mental health nursing: A communication approach to evidence-based care (p. 260). St. Louis: Saunders.

Cognitive Ability: Applying

Client Needs: Psychosocial Integrity

Integrated Process: Nursing Process/Implementation

Content Area: Mental Health Awarded 0.0 points out of 1.0 possible points. 109.ID: 383712416 Colchicine has been prescribed for a client with a diagnosis of gout, and the nurse provides information to the client about the medication. Which statement by the client indicates to the nurse that the client understands the information? "I need to limit my intake of fluids while I’m taking this medication." "I need to stop the medication and call my doctor if I have severe diarrhea." Correct "I can expect skin redness and a rash when I take this medication." "I may get a burning feeling in my throat, but it’s normal and will go away." Rationale: Colchicine is classified as an antigout agent. It interferes with the capacity of the white blood cells to initiate and maintain an inflammatory response to monosodium urate crystals. The client should maintain a high fluid intake (eight to ten 8-oz glasses of fluid per day) while taking the medication. The client is instructed to report a rash, sore throat, fever, unusual bruising or bleeding, weakness, tiredness, or numbness. A burning sensation in the throat or skin, severe diarrhea, and abdominal pain are signs of overdose.

Test-Taking Strategy: Focus on the subject, an indication that the client understands the information. Eliminate the options that are comparable or alike in that they indicate that skin redness and a rash and a burning feeling in the throat are normal and expected. Next eliminate the option that involves limiting intake of fluids. Review client teaching points for colchicine if you had difficulty with this question.

Reference: Lehne, R. (2010). Pharmacology for nursing care (7th ed., p. 865). St. Louis: Saunders.

Cognitive Ability: Evaluating

Client Needs: Physiological Integrity

Integrated Process: Nursing Process/Evaluation

Content Area: Pharmacology Awarded 0.0 points out of 1.0 possible points. 110.ID: 383706678 A client is admitted to the nursing unit with a diagnosis of avoidant personality disorder. Which of the following behaviors is a characteristic of the disorder? Neediness Perfectionism Preoccupation with details Hypersensitivity to negative evaluation Correct Rationale: Avoidant personality disorder is a psychiatric condition in which a person feels extremely shy, inadequate, and sensitive to rejection. Other characteristics of avoidant personality disorder include excessive anxiety in social situations and hypersensitivity to negative evaluation. Neediness is a characteristic of dependent personality disorder. Perfectionism and preoccupation with details are characteristics of obsessive-compulsive disorder.

Test-Taking Strategy: Use the process of elimination. Eliminate the options that are comparable or alike (perfectionist and preoccupied with details). Focusing on the client’s diagnosis, avoidant personality disorder, will direct you to the correct option. Review the characteristics of avoidant personality disorder if you had difficulty with this question.

Reference: Varcarolis, E., & Halter, M. (2009). Essentials of psychiatric mental health nursing: A communication approach to evidence-based care (p. 184). St. Louis: Saunders.

Cognitive Ability: Understanding

Client Needs: Psychosocial Integrity

Integrated Process: Nursing Process/Assessment

Content Area: Mental Health Awarded 0.0 points out of 1.0 possible points. 111.ID: 383705059 A female client admitted to the mental health unit tells the nurse that she cannot leave the house without checking to be sure that she has shut off the coffee maker and unplugged her curling iron. The client states that she even leaves the house, gets into her car, and then has to go back into the house to check these appliances again and that these behaviors are interfering with her work and social commitments. With which of the following anxiety disorders does the nurse associate this client's symptoms? Agoraphobia Avoidant personality disorder Obsessive-compulsive disorder Correct Dependent personality disorder Rationale: Obsessive-compulsive disorder is an anxiety disorder characterized by intrusive thoughts that produce uneasiness, apprehension, fear, or worry; by repetitive behaviors aimed at reducing anxiety; or by a combination of such thoughts (obsessions) and behaviors (compulsions). The client is inflexible and rigid, and is highly critical of self and others. The characteristics of dependent personality disorder include neediness and self-sacrificing and submissive behaviors. The client with avoidant personality disorder is extremely shy, feels inadequate, and is sensitive to rejection. Agoraphobia is the fear of open spaces.

Test-Taking Strategy: Use the process of elimination and think about the characteristics of each disorder identified in the options. Note the relationship between the data in the question and the disorder in the correct option. Review the characteristics of obsessive-compulsive disorder if you had difficulty with this question.

Reference: Varcarolis, E., & Halter, M. (2009). Essentials of psychiatric mental health nursing: A communication approach to evidence-based care (p. 139). St. Louis: Saunders.

Cognitive Ability: Analyzing

Client Needs: Psychosocial Integrity

Integrated Process: Nursing Process/Analysis

Content Area: Mental Health Awarded 0.0 points out of 1.0 possible points. 112.ID: 383708502 A nurse is developing a plan of care for a client admitted to the nursing unit with a diagnosis of paranoid personality disorder. On which characteristic of the disorder does the nurse base the plan of care? Inflexible and rigid Self-sacrificing and submissive Highly critical of self and others Projecting blame, possibly becoming hostile Correct Rationale: A client with paranoid personality disorder projects blame, is suspicious of others, and may become hostile or violent. The client also experiences cognitive or perceptual distortions. A client who is inflexible and rigid and is highly critical of self and others is showing signs of obsessive-compulsive disorder. Being self-sacrificing and submissive is a characteristic of a client with dependent personality disorder.

Test-Taking Strategy: Use the process of elimination. Focusing on the strategic word "paranoid" in the question will direct you to the correct option. Review the characteristics of paranoid personality disorder if you had difficulty with this question.

Reference: Varcarolis, E., & Halter, M. (2009). Essentials of psychiatric mental health nursing: A communication approach to evidence-based care (p. 178). St. Louis: Saunders.

Cognitive Ability: Applying

Client Needs: Psychosocial Integrity

Integrated Process: Nursing Process/Planning

Content Area: Mental Health Awarded 0.0 points out of 1.0 possible points. 113.ID: 383707980 A client on the mental health unit says to the nurse, "Everything is contaminated." The client scrubs her hands if she is forced to touch any object. While planning care, the nurse remembers that compulsive behavior: Temporarily eases anxiety in the client Correct Is an attempt on the client's part to punish herself Is an attempt on the client's part to seek the attention of others Is a response by the client to voices telling her that everything is contaminated and that she must engage in this behavior Rationale: Obsessions are thoughts, impulses, or images that persist and recur so that they cannot be dismissed from the mind. Compulsions are ritualistic behaviors that an individual feels driven to perform in an attempt to reduce anxiety. The other options identify interpretations of the client’s compulsive behavior.

Test-Taking Strategy: Knowledge regarding the characteristics of obsessive-compulsive disorder is required to answer this question. First, focus on the data in the question to determine the client’s diagnosis. Next, remember that the client performs compulsive acts in an attempt to relieve anxiety. Review the characteristics of obsessive-compulsive disorder if you had difficulty with this question.

Reference: Varcarolis, E., & Halter, M. (2009). Essentials of psychiatric mental health nursing: A communication approach to evidence-based care (p. 141). St. Louis: Saunders.

Cognitive Ability: Applying

Client Needs: Psychosocial Integrity

Integrated Process: Nursing Process/Planning

Content Area: Mental Health Awarded 0.0 points out of 1.0 possible points. 114.ID: 383710059 A male client arrives at the emergency department and reports to the nurse, "I woke up this morning and couldn't move my arms." He also tells the nurse that he works in a factory and witnessed an accident 3 weeks ago in which a fellow employee's hands were severed by a machine. What is the priority response by the nurse? Assessing the client for organic causes of loss of arm movement Correct Calling the crisis intervention team and asking them to assess the client Performing active and passive range-of-motion (ROM) exercises of the client's arms Asking the client to move his arms and documenting the loss of movement he has experienced Rationale: The priority is ruling out any neurological disorders. After it has been determined that there is no physiological basis for the problem, further psychiatric evaluation can be done. Encouraging the client to move his arms and performing active and passive ROM exercises have no beneficial effect in this situation. In fact, either option could be harmful if there is a physiological basis for the client’s problem.

Test-Taking Strategy: Note the strategic word "priority." Use Maslow’s Hierarchy of Needs theory and the nursing process to answer the question. The correct option relates to a physiological need and addresses the first step of the nursing process, assesment. Review care to the client who experiences a physiological disorder as a result of emotional trauma if you had difficulty with this question.

Reference: Stuart, G., & Laraia, M. (2009). Principles and practice of psychiatric nursing (9th ed., pp. 219, 220, 267). St. Louis: Mosby.

Cognitive Ability: Applying

Client Needs: Physiological Integrity

Integrated Process: Nursing Process/Implementation

Content Area: Mental Health Awarded 0.0 points out of 1.0 possible points. 115.ID: 383703633 A nurse is assigned to conduct an admission assessment of a client with a diagnosis of bipolar disorder. What does the nurse plan to do first? Perform the physical assessment Tell the client about the nursing unit rules Establish a trusting nurse-client relationship Correct Tell the client that he or she will have to participate in self-care Rationale: It is most important to establish a trusting relationship, which will indicate to the client that the client is important. After a therapeutic relationship has been established, other interventions may be carried out. The nurse would perform a physical assessment, but this would not be the first intervention. The client should be informed of the nursing unit’s rules, but, again, this is not the first intervention. Telling the client that he or she will have to participate in self-care is inappropriate. The client with bipolar disorder requiring hospitalization is likely to need assistance with care.

Test-Taking Strategy: Note the strategic word "first." Use your knowledge of therapeutic communication techniques and the nature of the nurse-client relationship to find the correct option. Review the importance of the nurse-client relationship if you had difficulty with this question.

Reference: Varcarolis, E., & Halter, M. (2009). Essentials of psychiatric mental health nursing: A communication approach to evidence-based care (p. 259). St. Louis: Saunders.

Cognitive Ability: Applying

Client Needs: Psychosocial Integrity

Integrated Process: Caring

Content Area: Mental Health Awarded 0.0 points out of 1.0 possible points. 116.ID: 383706088 A client arrives in the emergency department and tells the nurse that she is experiencing tingling in both hands and is unable to move her fingers. The client states that she has been unable to work because of the problem. During the psychosocial assessment, the client reports that 2 days earlier her husband told her that he wanted a separation and that she would have to support herself financially. The nurse concludes that this client is exhibiting signs compatible with: Severe anxiety Conversion disorder Correct Posttraumatic stress disorder (PTSD) Obsessive-compulsive disorder Rationale: Conversion disorder is characterized by the presence of one or more symptoms suggesting a neurological problem that cannot be attributed to a medical disorder. Psychological factors such as stress and conflict are associated with the onset or exacerbation of the symptom. A person with severe anxiety may focus on a particular detail or many scattered details. The person may have difficulty noticing what is going on in the environment, even when it is pointed out by another. Learning and problem-solving are not possible at this level of anxiety, and the client may be dazed and confused. PTSD is characterized by repeated re-experiencing of a highly traumatic event that involved actual or threatened death or serious injury to self or others to which the individual responded with intense fear, helplessness, or horror. Obsessions are thoughts, impulses, or images that persist and recur so that they cannot be dismissed from the mind. Compulsions are ritualistic behaviors that an individual feels driven to perform in an attempt to reduce anxiety.

Test-Taking Strategy: Focus on the data in the question and use your knowledge of the characteristics of each disorder identified in the options. Recalling the characteristics associated with a conversion disorder will direct you to the correct option. Remember that a conversion disorder is characterized by the presence of a neurological problem that cannot be attributed to a medical disorder. Review the characteristics of the disorders identified in the options if you had difficulty with this question.

Reference: Varcarolis, E., & Halter, M. (2009). Essentials of psychiatric mental health nursing: A communication approach to evidence-based care (pp. 156-158). St. Louis: Saunders.

Cognitive Ability: Analyzing

Client Needs: Psychosocial Integrity

Integrated Process: Nursing Process/Assessment

Content Area: Mental Health Awarded 0.0 points out of 1.0 possible points. 117.ID: 383703627 A client experiencing delusions says to the nurse, "I am the only one who can save the world from all of the terrorists." What is the appropriate response by the nurse? "Tell me your plan for saving the world." "Why do you think that you can accomplish this by yourself?" "I don't think anyone can save the world from the terrorists by himself." Correct "You must be powerful. Do you really believe that you can do this by yourself?" Rationale: The nurse should not go along with or reinforce the client’s delusion. The nurse should respond to the client by presenting reality. "Tell me your plan for saving the world," "Why do you think that you can accomplish this by yourself?" and "You must be powerful. Do you really believe you can do this by yourself?” all reinforce the delusion and encourage further conversation about it.

Test-Taking Strategy: Use your knowledge of therapeutic communication techniques and focus on the subject, a delusion. Eliminate the options that are comparable or alike in that they reinforce the delusion and encourage further conversation about it. Review care of the client with delusions if you had difficulty with this question.

Reference: Stuart, G., & Laraia, M. (2009). Principles and practice of psychiatric nursing (9th ed., pp. 27-31). St. Louis: Mosby.
Varcarolis, E., & Halter, M. (2009). Essentials of psychiatric mental health nursing: A communication approach to evidence-based care (p. 279). St. Louis: Saunders.

Cognitive Ability: Applying

Client Needs: Psychosocial Integrity

Integrated Process: Communication and Documentation

Content Area: Mental Health Awarded 0.0 points out of 1.0 possible points. 118.ID: 383705053 A client with adenocarcinoma of the ovary is scheduled to undergo chemotherapy with cyclophosphamide after total abdominal hysterectomy with bilateral salpingo-oophorectomy. What does the nurse instruct the client to do during chemotherapy? Select all that apply. Eat foods that are low in fat and protein Obtain pneumococcal and influenza vaccines Drink copious amounts of fluid and void frequently Correct Avoid contact with any individual who has signs or symptoms of a cold Correct Avoid contact with all individuals other than immediate family members Rationale: Hemorrhagic cystitis is an adverse effect of this medication. The client is encouraged to drink copious amounts of fluid at least 24 hours before, during, and after chemotherapy. The client is also encouraged to void frequently to prevent cystitis. The client is not to receive immunizations without the physician’s approval, because they could diminish the body’s resistance, putting the client at increased risk for infection. It is not necessary for the client to avoid contact with all individuals other than immediate family members. The client should, however, avoid contact with individuals who are ill, have a cold, or have recently received a live-virus vaccine. Encouraging adequate dietary intake is appropriate, but a low-protein or low-fat diet is not necessary.

Test-Taking Strategy: Eliminate the option that uses the closed-ended word “all.” Next, use your knowledge of the adverse effects of this chemotherapeutic agent and the general adverse effects of chemotherapy to answer correctly. Remember that cyclophosphamide can cause hemorrhagic cystitis and that chemotherapy can cause neutropenia and increase the client’s risk for infection. Review the adverse effects of cyclophosphamide if you had difficulty with this question.

Reference: Lehne, R. (2010). Pharmacology for nursing care (7th ed., p. 1186). St. Louis: Saunders.

Cognitive Ability: Applying

Client Needs: Physiological Integrity

Integrated Process: Teaching and Learning

Content Area: Pharmacology Awarded 0.0 points out of 1.0 possible points. 119.ID: 383710532 A client who is scheduled to undergo chemotherapy asks the nurse, "Is my hair going to fall out?" The nurse responds by telling the client that: Her hair will definitely fall out She should not be worrying about her hair at this point Her hair may fall out but will regrow after the chemotherapy is discontinued Correct Vigorous hair-brushing is important while the client is undergoing chemotherapy to prevent hair loss Rationale: Some chemotherapeutic agents cause hair loss, and the client should be informed of this possibility. The client should also be reassured that chemotherapy-related hair loss is temporary and that the hair will regrow after the chemotherapy is discontinued, perhaps in a different shade, color, or texture. Telling the client that her hair will definitely fall out is incorrect and telling her that she should not be worrying about her hair at this point ignores the client’s concern. Hair dyes, permanents, and vigorous hair-brushing are avoided to minimize thinning.

Test-Taking Strategy: Use the process of elimination. Eliminate the option containing the closed-ended word "definitely" and the option containing the word "vigorous." Next eliminate the option that ignores the client’s concern. Review the effects of chemotherapy on the hair if you had difficulty with this question.

Reference: Ignatavicius, D., & Workman, M. (2010). Medical-surgical nursing: Patient-centered collaborative care (6th ed., pp. 428, 430). St. Louis: Saunders.

Cognitive Ability: Applying

Client Needs: Psychosocial Integrity

Integrated Process: Nursing Process/Implementation

Content Area: Adult Health/Oncology Awarded 0.0 points out of 1.0 possible points. 120.ID: 383706072 A nurse has given a client with viral hepatitis instructions about home care. Which of the following statements by the client indicates to the nurse that the client needs further teaching? “I can’t drink alcohol.” “I have to avoid having sex until the test for antibodies comes back negative.” “I need to rest a lot during the day and get enough sleep at night.” “I need to eat three meals a day with foods high in protein, fat, and carbs.” Correct Rationale: The client with hepatitis is easily fatigued and may require several weeks to reach his or her former activity level. The client should avoid hepatotoxic substances such as alcohol. Sexual intercourse is avoided until antibody testing results are negative. It is important for the client to get adequate rest both during the day and at night so that the liver may heal. The client should consume a high-carbohydrate, low-fat diet.

Test-Taking Strategy: Note the strategic words "needs further teaching." These words indicate a negative event query and the need to select the incorrect client statement. Recall that the liver is the organ affected in hepatitis and think about the anatomy and physiology of the liver to answer correctly. Review home care measures for the client with hepatitis if you had difficulty with this question.

Reference: Ignatavicius, D., & Workman, M. (2010). Medical-surgical nursing: Patient-centered collaborative care (6th ed., p. 1360). St. Louis: Saunders.

Cognitive Ability: Evaluating

Client Needs: Physiological Integrity

Integrated Process: Teaching and Learning

Content Area: Adult Health/Gastrointestinal Awarded 0.0 points out of 1.0 possible points. 121.ID: 383703607 A nurse provides home care instructions to a client who has undergone fluorescein angiography. The nurse determines that the client needs further instruction if the client states that he must: Drink fluids to eliminate the dye Contact the physician if the skin appears yellow Correct Expect that the urine will be bright green until the dye has been excreted Wear sunglasses and avoid direct sunlight until pupil dilation returns to normal Rationale: Fluorescein angiography provides a detailed image and permanent record of eye circulation. Photographs are taken in rapid succession after the intravenous administration of dye. After the test, the client may feel weak and nauseated. Once the nausea has resolved, the client is encouraged to drink fluids to eliminate the dye. The nurse also encourages rest and emphasizes that any yellow discoloration of the skin will disappear in a few hours. After the test, the urine will be bright green until the dye has been excreted. The client is instructed to avoid direct sunlight until pupil dilation returns to normal.

Test-Taking Strategy: Note the strategic words "needs further instruction." These words indicate a negative event query and the need to select the incorrect statement. Recalling that in fluorescein angiography a dye is injected and that the pupils are dilated will direct you to the correct option. Review home care instructions for a client who has undergone fluorescein angiography if you had difficulty with this question.

Reference: Ignatavicius, D., & Workman, M. (2010). Medical-surgical nursing: Patient-centered collaborative care (6th ed., pp. 1081, 1082). St. Louis: Saunders.

Cognitive Ability: Evaluating

Client Needs: Physiological Integrity

Integrated Process: Teching and Learning

Content Area: Adult Health/Eye Awarded 0.0 points out of 1.0 possible points. 122.ID: 383713177 An emergency department nurse is assessing a client with acute closed-angle glaucoma. Which of the following characteristics of the disorder does the nurse expect the client to exhibit? Select all that apply. Nausea Correct Eye pain Correct Vomiting Correct Headache Correct Diminished central vision Increased light perception Rationale: In acute closed-angle glaucoma, the onset of symptoms is acute and the client complains of sudden excruciating pain around the eyes that radiates over the sensory distribution of the fifth cranial nerve. Headache or brow ache, nausea, vomiting, and abdominal discomfort may also occur. Other symptoms of glaucoma include seeing colored halos around lights, sudden blurred vision with decreased light perception, and loss of peripheral vision.

Test-Taking Strategy: Use the process of elimination and your knowledge of the symptoms associated with glaucoma. Noting the word "acute" in the diagnosis presented in the question will help direct you to the correct options. Review the manifestations of acute closed-angle glaucoma if you had difficulty with this question.

Reference: Ignatavicius, D., & Workman, M. (2010). Medical-surgical nursing: Patient-centered collaborative care (6th ed., p. 1097). St. Louis: Saunders.

Cognitive Ability: Analyzing

Client Needs: Physiological Integrity

Integrated Process: Nursing Process/Assessment

Content Area: Adult Health/Eye Awarded 0.0 points out of 1.0 possible points. 123.ID: 383702950 A nurse is measuring intraocular pressure by means of tonometry in a client who has just been found to have open-angle glaucoma. Which tonometry reading would the nurse expect to note in this client? 8 mm Hg 14 mm Hg 20 mm Hg 28 mm Hg Correct Rationale: Intraocular pressure, measured with the use of tonometry, is increased in glaucoma. The normal tonometry reading is 10 to 21 mm Hg. In open-angle glaucoma, the tonometry reading is slightly higher than normal, usually between 22 and 32 mm Hg.

Test-Taking Strategy: Recall the normal range of intraocular readings. Next, note the client’s diagnosis and recall that the problem in glaucoma is increased intraocular pressure. This will direct you to the correct option, the one that identifies the highest pressure. Review normal intraocular pressure readings if you had difficulty with this question.

Reference: Ignatavicius, D., & Workman, M. (2010). Medical-surgical nursing: Patient-centered collaborative care (6th ed., p. 1097). St. Louis: Saunders.

Cognitive Ability: Understanding

Client Needs: Physiological Integrity

Integrated Process: Nursing Process/Assessment

Content Area: Adult Health/Eye Awarded 0.0 points out of 1.0 possible points. 124.ID: 383705061 An emergency department nurse assessing a client with Bell's palsy collects subjective and objective data. Which of the following findings does the nurse expect to note? A symmetrical smile Tightening of all facial muscles Ability to wrinkle the forehead on request Complaints of inability to close the eye on the affected side Correct Rationale: The onset of Bell’s palsy is acute. Maximal paralysis occurs within 5 days in almost all clients. Pain behind the ear or on the face may precede paralysis by a few hours or days. The disorder is characterized by a drawing sensation and paralysis, not tightening, of all facial muscles on the affected side. The client cannot close the eye, wrinkle the forehead, smile, whistle, or grimace. The face appears masklike and sags. Taste is usually impaired to some degree, but this symptom seldom persists beyond the second week of paralysis. Loss of peripheral vision is not associated with Bell’s palsy.

Test-Taking Strategy: Focus on the client’s diagnosis and note the strategic word "palsy." Note the relationship of the word “palsy” and the data in the correct option, an inability to close the eye on the affected side. Review the manifestations of Bell’s palsy if you had difficulty with this question.

Reference: Ignatavicius, D., & Workman, M. (2010). Medical-surgical nursing: Patient-centered collaborative care (6th ed., p. 1026). St. Louis: Saunders.

Cognitive Ability: Analyzing

Client Needs: Physiological Integrity

Integrated Process: Nursing Process/Assessment

Content Area: Adult Health/Neurological Awarded 0.0 points out of 1.0 possible points. 125.ID: 383703643 A nurse is preparing to care for a preschool-age child with sickle cell anemia who is experiencing vasoocclusive pain. Which method of assessing the degree of pain the child is experiencing is most appropriate? Asking the child to describe the intensity of the pain Asking the child to use a numeric rating scale of 0 to 100 Asking the child whether the patient-controlled analgesia (PCA) pump is relieving the pain Asking the child to point to the face, on a spectrum ranging from smiling to very sad, that best describes the pain Correct Rationale: A client of preschool age has the cognitive ability to recognize happy and sad faces and to correlate them with the level of pain he or she is experiencing. It may be too complicated for some preschoolers to come up with words to describe varying intensities of pain. Many preschool children are not yet able to count to 100 or to understand the value of numbers in relation to other numbers. A child of preschool age is too young to control a PCA pump.

Test-Taking Strategy: Use your knowledge of growth and development and developmental stages to answer the question. With this in mind, eliminate each of the incorrect options and choose the one that is the simplest and easiest to teach and use. Review the concepts of growth and development and pain assessment in a child if you had difficulty with this question.

Reference: McKinney, E., James, S., Murray, S., & Ashwill, J. (2009). Maternal-child nursing (3rd ed., pp. 993, 1285). St. Louis: Elsevier.

Cognitive Ability: Applying

Client Needs: Health Promotion and Maintenance

Integrated Process: Nursing Process/Assessment

Content Area: Child Health – Hematological Awarded 0.0 points out of 1.0 possible points. 126.ID: 383710579 A school nurse observing a child with Down syndrome is participating in a physical education class and notes that the child is experiencing a diminution in motor abilities. The nurse asks to see the child and conducts an assessment, during which the child complains of neck pain and loss of bladder control. What is the appropriate action by the nurse in this situation? Contacting the child's physician to report the findings Correct Administering acetaminophen (Tylenol) to the child to relieve the pain Asking that the child not attend the physical education class until the neck pain has subsided Teaching the child how to use peripads to prevent embarrassment resulting from loss of bladder control Rationale: Children with Down syndrome who participate in sports that may involve stress on the neck should be evaluated radiologically for atlantoaxial instability. Symptoms of the disorder include neck pain, weakness, and torticollis. Affected children are at risk for spinal cord compression. Signs of spinal cord compression include persistent neck pain, loss of established motor skills and bladder or bowel control, and changes in sensation. If any of these signs are noted, it must be reported immediately to the physician. Administering (Tylenol) to relieve the pain and not allowing the child to attend physical education class until the pain has subsided are inappropriate actions that will delay necessary interventions. Teaching the child to use peripads to prevent embarrassment resulting from loss of bladder control may be appropriate in certain scenarios, but in this situation the child is exhibiting signs of spinal cord compression, requiring immediate intervention.

Test-Taking Strategy: Focus on the data in the question. Noting that the child is complaining of neck pain and loss of bladder control will help direct you to the correct option. Also note that each incorrect option addresses only one of the child’s complaints. Review care of the child with Down syndrome and the signs of spinal cord compression if you had difficulty with this question.

References: McKinney, E., James, S., Murray, S., & Ashwill, J. (2009). Maternal-child nursing (3rd ed., p. 1546). St. Louis: Elsevier.
Perry, S., Hockenberry, M., Lowdermilk, D., & Wilson, D. (2010). Maternal-child nursing care (4th ed., p. 1185). St. Louis: Mosby.

Cognitive Ability: Applying

Client Needs: Physiological Integrity

Integrated Process: Nursing Process/Implementation

Content Area: Child Health/Neurological Awarded 0.0 points out of 1.0 possible points. 127.ID: 383706603 A client is taking gentamicin sulfate (Genoptic) for the treatment of pelvic inflammatory disease. What does the nurse ask the client during assessment for adverse effects of the medication? "When was your last menstrual period?" "When was your last bowel movement?" "Are you having any difficulty hearing?" Correct "Are you having any difficulty breathing?" Rationale: Gentamicin sulfate is an aminoglycoside. It inhibits bacterial protein synthesis and has a bactericidal effect. Serious adverse reactions to aminoglycosides include ototoxicity and nephrotoxicity. The nurse must assess the client for changes in hearing, balance, and urine output. The remaining assessment questions are not associated with the adverse effects of this medication.

Test-Taking Strategy: Focus on the name of the medication and recall that it belongs to the aminoglycoside class of medications. Recalling that the adverse effects of aminoglycosides are ototoxicity and nephrotoxicity will direct you to the correct option. Review the adverse effects of gentamicin and aminoglycosides if you had difficulty with this question.

Reference: Kee, J., Hayes, E., & McCuistion, L. (2009). Pharmacology: A nursing process approach (6th ed., p. 448). Philadelphia: Saunders.

Cognitive Ability: Analyzing

Client Needs: Physiological Integrity

Integrated Process: Nursing Process/Assessment

Content Area: Pharmacology Awarded 0.0 points out of 1.0 possible points. 128.ID: 383710084 A nurse is providing instruction about insulin therapy and its administration to an adolescent client who has just been found to have diabetes mellitus. Which statement by the client indicates a need for further instruction? "It’s important to rotate injection sites." "I need to store the insulin in a cool, dry place." "I need to keep any unopened bottles of insulin in the freezer." Correct "I need to check the expiration date on the insulin before I use it." Rationale: Insulin is stored in a cool, dry place. It should not be placed in the freezer or exposed to excess heat or agitation. Injection sites should be rotated to ensure adequate insulin absorption and to prevent complications of insulin administration. Once a bottle of insulin has been opened, it is dated and discarded as recommended. The client should check the expiration date on the insulin vial before using it.

Test-Taking Strategy: Use the process of elimination and note the strategic words "need for further instruction." These words indicate a negative event query and the need to select the incorrect client statement. Think about the chemical composition and stability of insulin and recall that insulin should not be frozen. Review client teaching points for insulin if you had difficulty with this question.

Reference: McKinney, E., James, S., Murray, S., & Ashwill, J. (2009). Maternal-child nursing (3rd ed., p. 1444). St. Louis: Elsevier.

Cognitive Ability: Evaluating

Client Needs: Physiological Integrity

Integrated Process: Teaching and Learning

Content Area: Child Health/Endocrine Awarded 0.0 points out of 1.0 possible points. 129.ID: 383702977 A nurse is providing information on the glycosylated hemoglobin assay and its purpose to a client with diabetes mellitus. The nurse tells the client that this blood test: Is a measure of the client's hematocrit level Is a measure of the client's hemoglobin level Helps predict the risk for the development of chronic complications of diabetes mellitus Correct Provides a determination of short-term glycemic control in the client with diabetes mellitus Rationale: Glycosylated hemoglobin is the best indicator of the average blood glucose level. Because glucose attaches itself to the hemoglobin molecule, measurement of glycosylated hemoglobin indicates the average blood glucose level during the previous 120 days, the lifespan of the red blood cell. The test is used to assess long-term glycemic control, as well as to predict the risk for the development of chronic complications.

Test-Taking Strategy: Use the process of elimination. Eliminate the options that are comparable or alike (i.e., hemoglobin and hematocrit levels). To select from the remaining options, recall that this test serves as an assessment of long-term glycemic control, which will direct you to the correct option. Review the purpose of testing the glycosylated hemoglobin level if you had difficulty with this question.

Reference: Ignatavicius, D., & Workman, M. (2010). Medical-surgical nursing: Patient-centered collaborative care (6th ed., pp. 1473, 1474). St. Louis: Saunders.

Cognitive Ability: Applying

Client Needs: Physiological Integrity

Integrated Process: Nursing Process/Implementation

Content Area: Adult Health/Endocrine Awarded 0.0 points out of 1.0 possible points. 130.ID: 383706080 A client living in a long-term care facility shouts at the nurse, "Get out of my room! I don't need your help!" What is the appropriate way for the nurse to document this occurrence in the client's record? Writing that the client is very agitated Writing that the client yelled at the nurse Writing that the client is able to perform her own care Writing down the client's words and placing them in quotation marks Correct Rationale: An objective description is the result of direct observation and measurement. Documenting inferences without supporting factual data is not acceptable, because a client’s statements may be misunderstood. Documenting the client’s words verbatim and placing them in quotations ensures accurate data. The remaining options do not provide objective descriptions.

Test-Taking Strategy: Use your knowledge of the basic principles of documentation to answer the question. Eliminate the options that are comparable or alike in that they constitute inferences rather than factual data. Review the principles of documentation if you had difficulty with this question.

Reference: Potter, P., & Perry, A. (2009). Fundamentals of nursing (7th ed., pp. 234, 352-357). St. Louis: Mosby.

Cognitive Ability: Applying

Client Needs: Safe and Effective Care Environment

Integrated Process: Communication and Documentation

Content Area: Leadership and Management Awarded 0.0 points out of 1.0 possible points. 131.ID: 383706664 A nurse in the cardiac care unit is told that a client with a diagnosis of myocardial infarction will be admitted from the emergency department. Which item does the nurse give priority to placing at the client's bedside? Bedside commode Suctioning equipment Electrocardiography machine Oxygen cannula and flowmeter Correct Rationale: The client will require oxygen therapy after myocardial infarction to improve oxygen supply to the myocardium and ease the pain resulting from ischemia. Therefore the oxygen cannula and flowmeter are the priority. Suctioning equipment is not the priority item but may be needed if a complication occurs. An electrocardiogram machine and bedside commode may be necessary but are not the priority items.

Test-Taking Strategy: Note the strategic word “priority.” Use the ABCs — airway, breathing, and circulation — to answer the question. This will direct you to the correct option. Review care of the client who has sustained a myocardial infarction if you had difficulty with this question.

Reference: Ignatavicius, D., & Workman, M. (2010). Medical-surgical nursing: Patient-centered collaborative care (6th ed., p. 856). St. Louis: Saunders.

Cognitive Ability: Applying

Client Needs: Physiological Integrity

Integrated Process: Nursing Process/Planning

Content Area: Critical Care Awarded 0.0 points out of 1.0 possible points. 132.ID: 383704556 Cascara sagrada has been prescribed for a client with diminished colonic motor response as a means of promoting defecation. The nurse provides information to the client about the medication and tells the client to: Increase fluid intake Correct Consume low-fiber foods Consume foods that are low in potassium Contact the physician if the urine turns yellow-brown Rationale: Cascara sagrada is a stimulant laxative. The nurse encourages the client to increase fluid intake, to consume a high-fiber diet, and to exercise. Hypokalemia may result from use of the medication, so the nurse encourages the client to consume foods high in potassium. The client’s urine may turn pink-red, red-violet, red-brown, or yellow-brown, but the client is told that this is a temporary, harmless effect.

Test-Taking Strategy: Use the process of elimination. Focus on the name of the medication and note that it is prescribed to promote defecation. Recalling that this medication is a laxative will direct you to the correct option. Review the side effects of cascara sagrada and measures to promote defecation if you had difficulty with this question.

Reference: Kee, J., Hayes, E., & McCuistion, L. (2009). Pharmacology: A nursing process approach (6th ed., p. 716). Philadelphia: Saunders.

Cognitive Ability: Applying

Client Needs: Physiological Integrity

Integrated Process: Nursing Process/Implementation

Content Area: Pharmacology Awarded 0.0 points out of 1.0 possible points. 133.ID: 383706076 Cyclobenzaprine (Flexeril) is prescribed to a client with multiple sclerosis for the treatment of muscle spasms. For which common side effect of this medication does the nurse monitor the client? Diarrhea Drowsiness Correct Abdominal pain Increased salivation Rationale: Cyclobenzaprine (Flexeril) is a centrally acting skeletal muscle relaxant used in the management of muscle spasm accompanying a variety of conditions. Drowsiness, dizziness, and dry mouth are the most frequent side effects of cyclobenzaprine. Rare side effects include fatigue, tiredness, blurred vision, headache, nervousness, confusion, nausea, constipation, dyspepsia, and an unpleasant taste in the mouth.

Test-Taking Strategy: Focus on the subject, a frequent side effect. Recalling that this medication is a muscle relaxant will direct you to the correct option. Review the common side effects of this medication if you had difficulty with this question.

Reference: Lehne, R. (2010). Pharmacology for nursing care (7th ed., p. 243). St. Louis: Saunders.

Cognitive Ability: Analyzing

Client Needs: Physiological Integrity

Integrated Process: Nursing Process/Assessment

Content Area: Pharmacology Awarded 0.0 points out of 1.0 possible points. 134.ID: 383706611 A nurse administers nitroglycerin sublingually to a client with angina pectoris who complains of chest pain. The medication is ineffective, so the nurse prepares to administer a second dose. Before administering the nitroglycerin, which action does the nurse make a priority? Checking the client's blood pressure Correct Obtaining blood levels of cardiac enzymes Asking the client whether he has a headache Obtaining a 12-lead electrocardiogram (ECG)
  Rationale: Nitroglycerin is a nitrate that dilates the coronary arteries. One adverse effect of the medication is hypotension, and the nurse would assess the blood pressure and apical pulse before administration and periodically after the dose is given. Blood levels of cardiac enzymes are obtained if prescribed, but the priority checking the client’s blood pressure. Headache is a frequent side effect of the medication, mostly early in therapy and usually disappearing with continued treatment. It is not necessary to obtain a 12-lead ECG before administering a second dose of nitroglycerin unless this is prescribed by the physician. However, the client receiving intravenous nitroglycerin must have continuous ECG monitoring.

Test-Taking Strategy: Focus on the action of nitroglycerin. Recall that nitroglycerin is a vasodilator and recall the effects of vasodilation. Noting the word “priority” will direct you to the correct option, checking the blood pressure. Review nursing interventions for the administration of nitroglycerin if you had difficulty with this question.

Reference: Ignatavicius, D., & Workman, M. (2010). Medical-surgical nursing: Patient-centered collaborative care (6th ed., p. 856). St. Louis: Saunders.

Cognitive Ability: Applying

Client Needs: Physiological Integrity

Integrated Process: Nursing Process/Implementation

Content Area: Adult Health/Cardiovascular Awarded 0.0 points out of 1.0 possible points. 135.ID: 383702991 Ciprofloxacin hydrochloride (Cipro) is prescribed to a client with a urinary tract infection. The nurse, providing instruction about the medication, tells the client that it is best to take the medication: With milk With an antacid 2 hours after meals Correct With aluminum hydroxide Rationale: Ciprofloxacin hydrochloride is an anti-infective in the fluoroquinolone family. It may be taken without regard to meals, but the preferred dosing time is 2 hours after a meal. Milk may affect absorption. Antacids (here, aluminum hydroxide) may reduce absorption and should be administered 2 hours apart from the ciprofloxacin hydrochloride.

Test-Taking Strategy: Use the process of elimination. Eliminate the options that are comparable or alike in that they call for administration of the medication with another substance (milk, antacids). Review ciprofloxacin hydrochloride and how it is administered if you had difficulty with this question.

Reference: Hodgson, B., & Kizior, R. (2010). Saunders nursing drug handbook 2010 (p. 239). St. Louis: Saunders.

Cognitive Ability: Applying

Client Needs: Physiological Integrity

Integrated Process: Nursing Process/Implementation

Content Area: Pharmacology Awarded 0.0 points out of 1.0 possible points. 136.ID: 383711485 A nurse provides home care instructions to a client with coronary artery disease (CAD) who is being discharged from the hospital. Which statement by the client indicates a need for further instruction? "I need to carry my nitroglycerin with me at all times." "I need to check my pulse before, during, and after exercise." "I need to avoid foods with saturated fats and foods high in cholesterol." "I need to participate in aerobic and weightlifting exercise three times a week." Correct Rationale: The client with CAD should participate in a simple exercise program on a regular basis. The client may begin a simple walking program by walking 400 feet twice a day at a rate of 1 mph the first week after discharge and increasing the distance and rate as tolerated, usually weekly, until he or she can walk 2 miles at 3 to 4 mph. The client should always carry nitroglycerin and must comply with dietary restrictions, including avoiding foods with saturated fats and foods high in cholesterol. The nurse instructs the client to take a pulse reading before, halfway through, and after exercise. The client should avoid activities that involve straining, including weightlifting, push-ups and pull-ups, and straining during bowel movements.

Test-Taking Strategy: Use the process of elimination and note the strategic words "need for further instruction," which indicate a negative event query and the need to select the incorrect statement. Recall the pathophysiology associated with CAD and note the words "weightlifting exercise" in the correct option. Review home care instructions for the client with CAD if you had difficulty with this question.

Reference: Ignatavicius, D., & Workman, M. (2010). Medical-surgical nursing: Patient-centered collaborative care (6th ed., p. 871). St. Louis: Saunders.

Cognitive Ability: Evaluating

Client Needs: Physiological Integrity

Integrated Process: Teaching and Learning

Content Area: Adult Health/Cardiovascular Awarded 0.0 points out of 1.0 possible points. 137.ID: 383710062 A nurse provides information to a client who will be undergoing endoscopic retrograde cholangiopancreatography (ERCP). The nurse tells the client that: There is no need to fast (NPO status) before the procedure The gallbladder is easily removed during this procedure if gallstones are found The procedure is performed specifically to visualize the esophagus, stomach, and duodenum Dye may be injected during the procedure to permit visualization of the pancreatic and biliary ducts Correct Rationale: ERCP involves the oral insertion of an endoscope with a side-viewing tip and a cannula that can be maneuvered into the ampulla of Vater. Dye may be injected to outline the pancreatic and biliary ducts. The procedure may be combined with papillotomy to enlarge the sphincter and release gallstones. However, the gallbladder itself cannot be removed during this procedure. As with any endoscopic procedure, the client must remain NPO for 8 hours before the test.

Test-Taking Strategy: Use the process of elimination. Eliminate the option containing the closed-ended word "only." To select from the remaining options, focus on the name of the procedure. Note the relationship between the name and the words "pancreatic and biliary ducts" in the correct option. Review this endoscopic procedure if you had difficulty with this question.

Reference: Ignatavicius, D., & Workman, M. (2010). Medical-surgical nursing: Patient-centered collaborative care (6th ed., p. 1227). St. Louis: Saunders.

Cognitive Ability: Applying

Client Needs: Physiological Integrity

Integrated Process: Nursing Process/Implementation

Content Area: Adult Health/Gastrointestinal Awarded 0.0 points out of 1.0 possible points. 138.ID: 383713175 A client who has undergone knee-replacement surgery will be self-administering enoxaparin sodium (Lovenox) at home. The nurse teaches the client about the medication and tells the client to: Store the medication in the refrigerator Lie down to administer the subcutaneous injection Correct Inject the medication in the upper outer aspect of the arm Discard the medication if the solution appears pale yellow Rationale: Enoxaparin sodium is an anticoagulant that is administered by way of subcutaneous injection. It is injected into the abdominal wall. The solution, which appears clear and colorless to pale yellow, is stored at room temperature. The client is instructed to lie down to administer the injection and to introduce the entire length of the needle (½ inch) into a skin fold held between the thumb and forefinger.

Test-Taking Strategy: Note the strategic words “self-administering.” Recalling that enoxaparin sodium is an anticoagulant will assist you in answering correctly, because subcutaneous anticoagulants are administered in the abdomen. Review client instructions for the administration of enoxaparin sodium if you had difficulty with this question.

Reference: Skidmore-Roth, L. (2010). Mosby’s nursing drug reference (23rd ed., p. 430). St. Louis: Mosby.

Cognitive Ability: Applying

Client Needs: Physiological Integrity

Integrated Process: Teaching and Learning

Content Area: Pharmacology Awarded 0.0 points out of 1.0 possible points. 139.ID: 383703667 An intravenous dose of adenosine (Adenocard) is prescribed for a client to treat Wolff-Parkinson-White syndrome. Which piece of equipment does the nurse make a priority of obtaining before administering the medication? Pulse oximeter Cardiac monitor Correct Blood-pressure cuff Suction catheter and suction machine Rationale: Wolff-Parkinson-White syndrome is an abnormality of cardiac rhythm that is manifested as supraventricular tachycardia. Adenosine is an antidysrhythmic medication used to treat this dysrhythmia. It is administered intravenously. Therefore obtaining a cardiac monitor is the priority. A pulse oximeter and blood pressure cuff will each provide information about the client’s cardiovascular status, but neither is the priority. There is no information in the question to indicate that a suction catheter and suction machine are necessary.

Test-Taking Strategy: Use the process of elimination and note the strategic word "priority." Note the relationship between the name of the medication, Adenocard, and the correct option. Review care of the client receiving adenosine (Adenocard) if you had difficulty with this question.

Reference: Kee, J., Hayes, E., & McCuistion, L. (2009). Pharmacology: A nursing process approach (6th ed., p. 928). Philadelphia: Saunders.

Cognitive Ability: Applying

Client Needs: Physiological Integrity

Integrated Process: Nursing Process/Planning

Content Area: Critical Care Awarded 0.0 points out of 1.0 possible points. 140.ID: 383703619 A nurse provides information to a client with coronary artery disease (CAD) about smoking-cessation measures. Which statement by the client indicates a need for further information? "A community support group will help me quit." "I should drink a cup of coffee if I feel the urge to smoke." Correct "Relaxation exercises will help control my urge to smoke." "I can try chewing gum or sucking on hard candy if I feel the urge to smoke." Rationale: The nurse should reinforce education about the hazards of smoking and provide encouragement to clients who are interested in or willing to try smoking cessation. The client should avoid using coffee and other caffeine-containing products, which can cause restlessness and anxiety, increasing the urge to smoke. There are appropriate strategies to assist in smoking cessation. Among these are community support groups, relaxation exercises, and strategies such as chewing gum or sucking on hard candy if the urge to smoke arises.

Test-Taking Strategy: Note the strategic words "need for further information," which indicate a negative event query and the need to select the incorrect statement. Focusing on the subject and recalling the effects of caffeine on the body will direct you to the correct option. Review the strategies to assist in a smoking-cessation program if you had difficulty with this question.

Reference: Ignatavicius, D., & Workman, M. (2010). Medical-surgical nursing: Patient-centered collaborative care (6th ed., pp. 84, 85, 854). St. Louis: Saunders.

Cognitive Ability: Evaluating

Client Needs: Health Promotion and Maintenance

Integrated Process: Teaching and Learning

Content Area: Adult Health/Cardiovascular Awarded 0.0 points out of 1.0 possible points. 141.ID: 383708584 Captopril (Capoten) is prescribed for a hospitalized client with heart failure. Which action is a priority once the nurse has administered the first dose? Checking the client's apical heart rate Maintaining the client on bed rest for 3 hours Correct Monitoring the client for increased urine output Checking the client's breath sounds for decreased wheezing Rationale: Captopril is an angiotensin-converting enzyme (ACE) inhibitor. Excessive hypotension (first-dose syncope) may occur in the client with heart failure or in the client who is severely salt or volume depleted. The client is closely monitored for hypotension at the start of therapy and is maintained on bed rest for 3 hours after the initial dose. Checking the apical heart rate will provide information about the client’s cardiac status but is not an intervention specifically related to this medication. Increased urine output and decreased wheezing are expected if the client has received a diuretic.

Test-Taking Strategy: Use the process of elimination and focus on the name of the medication. Remember that medication names that end with -pril are ACE inhibitors and that these medications have an effect on blood pressure. This will direct you to the correct option. Review the effects of captopril and associated nursing interventions if you had difficulty with this question.

References: Ignatavicius, D., & Workman, M. (2010). Medical-surgical nursing: Patient-centered collaborative care (6th ed., p. 801). St. Louis: Saunders.
Lehne, R. (2010). Pharmacology for nursing care (7th ed., p. 477). St. Louis: Saunders.

Cognitive Ability: Applying

Client Needs: Physiological Integrity

Integrated Process: Nursing Process/Implementation

Content Area: Adult Health/Cardiovascular Awarded 0.0 points out of 1.0 possible points. 142.ID: 383706680 A client with heart failure suddenly experiences profound dyspnea, pallor, audible wheezing, and cyanosis, and the nurse suspects pulmonary edema. The nurse would first: Obtain a pulse oximetry reading Raise the head of the client's bed Correct Administer a dose of morphine sulfate Obtain a specimen for an arterial blood gas determination Rationale: Acute pulmonary edema is characterized by profound dyspnea, pallor, audible wheezing, and cyanosis. The nurse would first raise the head of the client’s bed and position the client to maximize chest expansion to ease the air hunger that the client is experiencing. An arterial blood gas or pulse oximetry reading will reveal the need for supplemental oxygen. Morphine sulfate may be administered because it blunts the sympathetic response and promotes peripheral vasodilation. On the basis of the options provided, however, the initial action is placing the client in a head-elevated position.

Test-Taking Strategy: Note the strategic word "first" and note that the question presents an emergency situation. Use the ABCs — airway, breathing, and circulation. The correct option is the action that will help alleviate the air hunger that the client is experiencing. Review immediate nursing actions for the client with pulmonary edema if you had difficulty with this question.

Reference: Ignatavicius, D., & Workman, M. (2010). Medical-surgical nursing: Patient-centered collaborative care (6th ed., pp. 770, 1620). St. Louis: Saunders.

Cognitive Ability: Applying

Client Needs: Physiological Integrity

Integrated Process: Nursing Process/Implementation

Content Area: Critical Care Awarded 0.0 points out of 1.0 possible points. 143.ID: 383703665 The nurse administers intravenous morphine sulfate to a client in pulmonary edema. For which intended effect of the medication does the nurse monitor the client? Relief of pain Relief of anxiety Correct Decreased urine output Increased blood pressure Rationale: Morphine sulfate reduces anxiety in the client in pulmonary edema. It blunts the sympathetic response and increases venous capacitance, thereby decreasing left atrial pressure. It also promotes peripheral vasodilation and causes blood to pool in the periphery. The client receiving morphine sulfate is monitored for signs and symptoms of respiratory depression and extreme decreases in blood pressure, especially when the medication is administered intravenously. Although morphine sulfate is an opioid analgesic and relieves pain, it is not administered to the client with pulmonary edema for its analgesic effect. Furosemide (Lasix) is administered to the client with pulmonary edema to increase urine output.

Test-Taking Strategy: Use the process of elimination, noting the strategic words "intended effect" and the client’s diagnosis. Thinking about the expected outcome of treatment for pulmonary edema and the effect of morphine sulfate will direct you to the correct option. Review the intended effects of morphine sulfate for the client with pulmonary edema if you had difficulty with this question.

Reference: Ignatavicius, D., & Workman, M. (2010). Medical-surgical nursing: Patient-centered collaborative care (6th ed., pp. 682-683, 776, 1620). St. Louis: Saunders.

Cognitive Ability: Evaluating

Client Needs: Physiological Integrity

Integrated Process: Nursing Process/Evaluation

Content Area: Critical Care Awarded 0.0 points out of 1.0 possible points. 144.ID: 383702944 A nurse is providing home care instructions to a client with coronary artery disease (CAD) who will be discharged home and will be taking 1 aspirin daily. The nurse tells the client: To stop the aspirin if nausea occurs To take the aspirin on an empty stomach That ringing in the ears is a sign of toxicity Correct That the aspirin is a short-term treatment and will probably be discontinued in 2 weeks Rationale: Aspirin, an antiplatelet agent, helps prevent blood clotting. A single low daily dose of aspirin may be a long-term component of the standard treatment regimen for the client with CAD. Gastric irritation may occur with the use of aspirin, and the client is advised to take the daily dose with food to prevent this. Ringing in the ears (tinnitus) is a sign of toxicity; if it occurs, the client should contact the physician. The client would not stop the medication if nausea occurs but would instead take the medication with food to relieve this discomfort.

Test-Taking Strategy: Use the process of elimination. Recalling the action and purpose of aspirin in the client with CAD will direct you to the correct option. Remember that aspirin causes gastric irritation and that tinnitus is a sign of toxicity. Review the purpose of aspirin in the client with a cardiac disorder if you had difficulty with this question.

Reference: Ignatavicius, D., & Workman, M. (2010). Medical-surgical nursing: Patient-centered collaborative care (6th ed., p. 858). St. Louis: Saunders.

Cognitive Ability: Applying

Client Needs: Physiological Integrity

Integrated Process: Teaching and Learning

Content Area: Adult Health/Cardiovascular Awarded 0.0 points out of 1.0 possible points. 145.ID: 383713112 A client receiving parenteral nutrition (PN) suddenly experiences chest pain and dyspnea, and the nurse suspects an air embolism. The nurse immediately places the client in a lateral Trendelenburg position, on the left side. What action does the nurse take next? Auscultating heart sounds Clamping the intravenous catheter Correct Checking the client's blood pressure Obtaining an arterial blood gas specimen Rationale: The signs of air embolism include chest pain, dyspnea, and hypoxia. Tachycardia and hypotension will also be present, and the client will experience anxiety. The nurse will also hear a loud churning sound on auscultation over the pericardium. The nurse immediately places the client in a lateral Trendelenburg position, on the left side. This position prevents air from flowing into the pulmonary veins. The nurse then clamps the intravenous catheter and notifies the physician. Although auscultating heart sounds, checking the client’s blood pressure, and obtaining an arterial blood gas specimen may be appropriate interventions, none is the next action to take after positioning the client.

Test-Taking Strategy: Use the process of elimination and note the strategic word "next" in the query of the question. Think about the consequences of an air embolism that travels to the pulmonary vessels. This will direct you to the correct option. Review care for the client with an air embolism if you had difficulty with this question.

Reference: Ignatavicius, D., & Workman, M. (2010). Medical-surgical nursing: Patient-centered collaborative care (6th ed., p. 231). St. Louis: Saunders.

Cognitive Ability: Applying

Client Needs: Physiological Integrity

Integrated Process: Nursing Process/Implementation

Content Area: Critical Care Awarded 0.0 points out of 1.0 possible points. 146.ID: 383711448 A nurse is teaching a client with left-side weakness how to walk with the use of a quad-cane. The nurse ensures that: The client places the cane on the left side The top of the cane is level with the client's waist 30-degree flexion of the client's elbow is maintained when the client is holding the cane Correct The client leans on the cane and places as much weight as possible on the cane when moving it forward Rationale: A cane may be used if a client needs minimal support for an affected leg. A straight cane offers the least support. A hemi-cane or quad-cane provides a broader base for the cane and therefore more support. The cane, which is placed on the unaffected side, should produce no more than 30 degrees of flexion of the elbow. The top of the cane should be level with the greater trochanter of the femur.

Test-Taking Strategy: Use the process of elimination. Keeping safety principles in mind will assist you in eliminating these options. To select from the remaining options, visualize the use of a cane. This will direct you to the correct option. Review teaching points for use of a cane if you had difficulty with this question.

Reference: Ignatavicius, D., & Workman, M. (2010). Medical-surgical nursing: Patient-centered collaborative care (6th ed., p. 1194). St. Louis: Saunders.

Cognitive Ability: Applying

Client Needs: Safe and Effective Care Environment

Integrated Process: Teaching and Learning

Content Area: Safety Awarded 0.0 points out of 1.0 possible points. 147.ID: 383702961 A nurse is preparing the room of a client in skeletal traction who will be admitted to the nursing unit. Which item for use by the client does the nurse identify as the most important? Telephone Television Trapeze bar Correct Bedside commode Rationale: A trapeze bar is a triangular device that hangs from a securely fastened overhead bar that is attached to the bed frame. It allows the client to pull up with the upper extremities to raise the trunk off the bed. It is a useful device for helping increase independence, maintain upper-body strength, and reducing the shearing action that results when the client slides across or up and down in the bed. The client in traction would not be allowed to get out of bed to use a bedside commode; rather, a bedpan or fracture pan would be used. Although a telephone and a television are helpful for diversion and maintaining social contact, they are not the most important items.

Test-Taking Strategy: Use the process of elimination. Eliminate the bedside commode first, recalling that the client in traction remains in bed. From the remaining options, use your knowledge of Maslow’s Hierarchy of Needs theory. Use of a telephone or television is a social and diversional activity, representative of psychosocial needs, so eliminate these options. The correct option satisfies a physiological need. Review care of the client in traction if you had difficulty with this question.

Reference: Black, J., & Hawks, J. (2009). Medical-surgical nursing: Clinical management for positive outcomes (8th ed., p. 525). St. Louis: Saunders.

Cognitive Ability: Applying

Client Needs: Physiological Integrity

Integrated Process: Nursing Process/Planning

Content Area: Adult Health/Musculoskeletal Awarded 0.0 points out of 1.0 possible points. 148.ID: 383702975 A nurse taking the vital signs of a client immediately after she has delivered a newborn notes that the client's heart rate is 110 beats/min. The nurse would first: Document the findings Offer the client oral fluids Recheck the heart rate in 1 hour Check the uterus and amount of lochia discharge Correct Rationale: After delivery, the normal heart rate ranges from 60 to 90 beats/min. Tachycardia may indicate excitement, fatigue, dehydration, hypovolemia, pain, or infection. If tachycardia is noted, additional assessments — blood pressure, location and firmness of the uterus, amount of lochia, estimated blood loss at delivery, and hemoglobin and hematocrit determinations — should be carried out. Although the nurse would document the findings, it is most appropriate for the nurse to assess the client to determine the cause of the tachycardia. Oral fluids are important if the client is dehydrated, but further assessment of the problem is required and dehydration would first need to be confirmed. Rechecking the heart rate in 1 hour will delay necessary interventions.

Test-Taking Strategy: Use the process of elimination and note the strategic word "first." Noting that the client is experiencing tachycardia and recalling the signs of shock will direct you to the correct option. Also, the correct option addresses the first step of the nursing process, assessment. Although rechecking the heart rate in 1 hour represents assessment, waiting 1 hour is incorrect. Review the expected findings in maternal vital signs after delivery if you had difficulty with this question.

Reference: McKinney, E., James, S., Murray, S., & Ashwill, J. (2009). Maternal-child nursing (3rd ed., pp. 465, 466). St. Louis: Elsevier.

Cognitive Ability: Applying

Client Needs: Physiological Integrity

Integrated Process: Nursing Process/Implementation

Content Area: Maternity/Postpartum Awarded 0.0 points out of 1.0 possible points. 149.ID: 383708504 A client is receiving an intravenous infusion of alteplase (tissue plasminogen activator, recombinant; tPA). For which adverse effect of the medication does the nurse monitor the client most closely? Bleeding Correct Hearing loss Decreased urine output Increased blood pressure Rationale: Alteplase is a thrombolytic agent used to dissolve existing thrombi. Bleeding is the most common adverse effect, and the nurse must monitor the client for obvious or occult signs of bleeding. Hearing loss and decreased urine output are not associated with the use of this medication. The medication may also cause a decrease in blood pressure and an allergic reaction, denoted by a rash or wheezing. 

Test-Taking Strategy: Note the strategic words "adverse effect." Also focus on the name of the medication, alteplase, and recall that medication names that end in the letters -ase identify thrombolytic agents. Recalling that thrombolytics cause bleeding will direct you to the correct option. Review alteplase and its adverse effects if you had difficulty with this question.

Reference: Gahart, B., & Nazareno, A. (2010). Intravenous medications (26th ed., p. 66). St. Louis: Mosby.

Cognitive Ability: Analyzing

Client Needs: Physiological Integrity

Integrated Process: Nursing Process/Assessment

Content Area: Pharmacology Awarded 0.0 points out of 1.0 possible points. 150.ID: 383708596 View video. The nurse is performing a sterile change of an abdominal dressing. Once the dressing has been removed and discarded in a waterproof bag, which action should the nurse take next?
  Assessing the wound Correct Donning sterile gloves Cleansing the wound Setting up the sterile field Rationale: View video. When performing a dressing change, the nurse dons clean gloves and removes the old dressing; checks the dressing for drainage, noting the amount, color, and odor if drainage is present; and discards the gloves and dressing. The nurse next assesses the wound for size, redness, swelling, and amount, color, odor, and type of drainage, if drainage is present. Next the nurse washes the hands and sets up the sterile field; dons sterile gloves; cleanses the wound with solution as prescribed, moving from the least to the most contaminated area; and redresses the wound. If a drain is present, the nurse applies additional layers of gauze as needed.

Test-Taking Strategy: Focus on the subject, a sterile dressing change. Visualize this procedure to answer the question after noting the strategic word “next.” Use the steps of the nursing process and note that the correct option is the only one that addresses the first step of the nursing process, assessment. Review the procedure for performing a dressing change of you had difficulty with this question.

Reference: Potter, P., & Perry, A. (2009). Fundamentals of nursing (7th ed., p. 1315). St. Louis: Mosby.

Cognitive Ability: Applying

Client Needs: Safe and Effective Care Environment

Integrated Process: Nursing Process/Implementation

Content Area: Infection Control Awarded 0.0 points out of 1.0 possible points. 151.ID: 383706038 A nurse is providing morning care to a client who has undergone surgery to repair a fractured left hip. Which item is most important for the nurse to use in turning the client from side to side to change the bed linens? Trapeze bar Sliding board Adduction device Abduction device Correct Rationale: After surgery to repair a fractured hip, an abduction device is used to maintain the affected extremity in good alignment and prevent dislocation. Some surgeons only allow the client to turn directly onto one side or the other; therefore the nurse checks the postoperative prescription. A trapeze bar may also be used, but it is not the priority item for repositioning the client. A sliding board is used to transfer a client from a bed to a stretcher or vice versa.

Test-Taking Strategy: Use the process of elimination, focusing on the subject, repositioning. Note the options that provide opposite actions. This will assist you in determining that one of these options may be correct. Remember that an abductor splint is used to maintain correct alignment and prevent dislodgment in a client who has undergone repair of a fractured hip. Review care of the client after hip surgery if you had difficulty with this question.

Reference: Ignatavicius, D., & Workman, M. (2010). Medical-surgical nursing: Patient-centered collaborative care (6th ed., p. 1197). St. Louis: Saunders.

Cognitive Ability: Applying

Client Needs: Safe and Effective Care Environment

Integrated Process: Nursing Process/Implementation

Content Area: Adult Health/Musculoskeletal Awarded 0.0 points out of 1.0 possible points. 152.ID: 383710526 A nurse provides dietary instructions to a client with osteoporosis who has sustained a fracture about foods that will promote healing. The nurse tells the client that it is best to consume foods that are high in: Fats Vitamin C Correct Carbohydrates Concentrated sugar Rationale: Dietary therapy for the client with osteoporosis includes foods high in calcium. If a fracture has occurred, the nurse encourages the client to eat foods high in vitamin C, protein, and iron, because these nutrients will promote healing. Although fats and carbohydrates should be included in the daily diet, they are not specifically related to the healing process. Foods containing concentrated sugar do not promote healing.

Test-Taking Strategy: Focus on the subject, foods that promote healing. Noting that the client has sustained a fracture will assist you in answering this question. Remember that vitamin C will promote healing. Review the foods that promote healing if you had difficulty with this question.

Reference: Ignatavicius, D., & Workman, M. (2010). Medical-surgical nursing: Patient-centered collaborative care (6th ed., pp. 1157, 1195). St. Louis: Saunders.

Cognitive Ability: Applying

Client Needs: Physiological Integrity

Integrated Process: Teaching and Learning

Content Area: Adult Health/Musculoskeletal Awarded 0.0 points out of 1.0 possible points. 153.ID: 383710520 A nurse in a physician's office is talking to a client who underwent mastectomy of the right breast 2 weeks ago. The client says to the nurse, "I hate looking at this incision. I feel that I'm not even myself anymore." The nurse interprets this statement to mean that the client is experiencing which problem? Inability to cope Distorted body image Correct Inability to care for self Inability to maintain health Rationale: Distorted body image is characterized by negative verbalizations or feelings about a body part. This is a common response after mastectomy. The nurse supports the client and helps her work through these feelings. There is no information in the question to indicate that inability to care for self, inability to cope, or inability to maintain health is a problem.

Test-Taking Strategy: Use the process of elimination. In this question, focusing on the client’s statement will direct you to the correct option. Review care of the client after mastectomy if you had difficulty with this question.

Reference: Black, J., & Hawks, J. (2009). Medical-surgical nursing: Clinical management for positive outcomes (8th ed., p. 965). St. Louis: Saunders.

Cognitive Ability: Evaluating

Client Needs: Psychosocial Integrity

Integrated Process: Nursing Process/Evaluation

Content Area: Adult Health/Oncology Awarded 0.0 points out of 1.0 possible points. 154.ID: 383703639 A nurse discovers that a client receiving heparin sodium by way of continuous intravenous (IV) infusion has removed the IV tubing from the infusion pump to change his hospital gown. After assessing the client and placing the tubing back in the infusion pump, which medication does the nurse check for in the medication room in case a heparin overdose has occurred? Protamine sulfate Correct Enoxaparin (Lovenox) Phytonadione (vitamin K) Aminocaproic acid (Amicar) Rationale: If the IV tubing is removed from an infusion pump and the tubing is not clamped, the client will receive a bolus of the solution of the medication contained in the solution. Heparin is an anticoagulant, and the client who receives a bolus dose of heparin is at risk for bleeding. The nurse would notify the physician. A blood sample for partial thromboplastin time (PTT) would be drawn and the results of testing evaluated. If the PTT is too high, the infusion may be stopped for a time, or a dose of protamine sulfate, the antidote for heparin, may be prescribed. Enoxaparin is an anticoagulant. Phytonadione is the antidote for warfarin sodium (Coumadin). Aminocaproic acid is an antifibrinolytic, inhibiting clot breakdown.

Test-Taking Strategy: Use the process of elimination and focus on the data in the question. Note that the cient is receiving heparin sodium and remember that this medication is an anticoagulant. Recalling that the antidote for heparin is protamine sulfate will direct you to the correct option. Review heparin sodium and the medications identified in the options if you had difficulty with this question.

Reference: Gahart, B., & Nazareno, A. (2010). Intravenous medications (26th ed., p. 682). St. Louis: Mosby.

Cognitive Ability: Applying

Client Needs: Physiological Integrity

Integrated Process: Nursing Process/Implementation

Content Area: Pharmacology Awarded 0.0 points out of 1.0 possible points. 155.ID: 383703663 The mother of a newborn found to have a congenital diaphragmatic hernia asks the nurse to explain the diagnosis. The nurse tells the mother that in this condition: The esophagus terminates before it reaches the stomach Gastric contents are regurgitated back into the esophagus Abdominal contents herniate through an opening of the diaphragm Correct A portion of the stomach protrudes through the esophageal hiatus of the diaphragm Rationale: In a congenital diaphragmatic hernia, abdominal contents herniate through an opening in the diaphragm. In gastroesophageal reflux disease, gastric contents are regurgitated back into the esophagus. Esophageal atresia is a condition in which the esophagus terminates before it reaches the stomach. A hiatal hernia is a condition in which a portion of the stomach protrudes through the esophageal hiatus of the diaphragm.

Test-Taking Strategy: Use the process of elimination. Note the relationship between the word "diaphragmatic" in the name of the diagnosis and "diaphragm" in the correct option. Review the characteristics of a congenital diaphragmatic hernia if you had difficulty with this question.

Reference: Lowdermilk, D., Perry, S., & Cashion, K. (2010). Maternity nursing (8th ed., p. 794). St. Louis: Mosby.

Cognitive Ability: Understanding

Client Needs: Physiological Integrity

Integrated Process: Nursing Process/Implementation

Content Area: Maternity/Postpartum Awarded 0.0 points out of 1.0 possible points. 156.ID: 383710522 A nurse is performing an assessment of a newborn with a diagnosis of esophageal atresia (EA) and tracheoesophageal fistula (TEF). Which findings does the nurse expect to note in the infant? Select all that apply. Drooling Correct Wheezing Hiccuping Short periods of apnea Excessive oral secretions Correct Bowel sounds over the chest Rationale: EA and TEF, the most life-threatening anomalies of the esophagus, often occur together, although they may occur singly. EA is a congenital anomaly in which the esophagus ends in a blind pouch or narrows into a thin cord, thereby failing to form a continuous passageway to the stomach. TEF is an abnormal connection between the esophagus and trachea. EA with or without TEF results in excessive oral secretions, drooling, and feeding intolerance. When fed, the infant may swallow but will then cough and gag and return the fluid through the nose and mouth. Bowel sounds over the chest is a clinical manifestation associated with congenital diaphragmatic hernia. Hiccuping and spitting up after a meal are clinical manifestations of gastroesophageal reflux. Coughing, wheezing, and short periods of apnea are clinical manifestations of hiatal hernia.

Test-Taking Strategy: Use the process of elimination. Focusing on the words "atresia" and "fistula" will direct you to the correct options. If you had difficulty with this question, review the clinical manifestations of EA and TEF.

Reference: McKinney, E., James, S., Murray, S., & Ashwill, J. (2009). Maternal-child nursing (3rd ed., p. 1097). St. Louis: Elsevier.

Cognitive Ability: Analyzing

Client Needs: Physiological Integrity

Integrated Process: Nursing Process/Assessment

Content Area: Newborn Awarded 0.0 points out of 1.0 possible points. 157.ID: 383711493 A nurse is reviewing the medical record of an infant in whom hypertrophic pyloric stenosis (HPS) is suspected. Which characteristics associated with the disorder does the nurse expects see documented in the infant’s medical record? Select all that apply. Weight loss Correct Facial edema Metabolic acidosis Projectile vomiting Correct Distended upper abdomen Correct Rationale: HPS occurs when the circular muscle of the pylorus becomes thickened, causing constriction of the pylorus and obstruction of the gastric outlet. Clinical manifestations include projectile vomiting, a hungry infant who eagerly accepts a second feeding after the vomiting episode, weight loss, signs of dehydration, and a distended upper abdomen. A readily palpable olive-shaped mass in the epigastrium just to the right of the umbilicus is noted, and gastric peristaltic waves, moving from left to right across the epigastrium, are visible. Laboratory findings include metabolic alkalosis, a result of the vomiting that occurs in this disorder. Facial edema and metabolic acidosis do not occur in this disorder.

Test-Taking Strategy: Focus on the name of the diagnosis and think about the pathophysiology associated with the disorder. Noting the word “stenosis” in the name of the condition will direct you to the correct option. Review the clinical manifestations of HPS if you had difficulty with this question.

Reference: McKinney, E., James, S., Murray, S., & Ashwill, J. (2009). Maternal-child nursing (3rd ed., p. 1120). St. Louis: Elsevier.

Cognitive Ability: Analyzing

Client Needs: Physiological Integrity

Integrated Process: Nursing Process/Assessment

Content Area: Child Health/Gastrointestinal Awarded 0.0 points out of 1.0 possible points. 158.ID: 383703641 A client with a history of angina pectoris tells the nurse that the chest pain usually occurs with moderate to prolonged exertion and is generally relieved by nitroglycerin or rest. Which type of angina does the nurse recognize in the client’s description? Stable Correct Variant Unstable Crescendo Rationale: Stable angina is chest discomfort that occurs with moderate to prolonged exertion in a pattern that is familiar to the client. It is usually relieved by nitroglycerin and rest. Unstable angina is a broad spectrum of disorders, including new-onset angina, variant (Prinzmetal) angina, preinfarction angina, and crescendo angina.Unstable angina is chest pain or discomfort that occurs at rest or with minimal exertion and causes marked limitation of activity. An increase in the number of attacks and an increase in the intensity of pain are characteristic of unstable angina. The pain may last longer than 15 minutes or may be poorly relieved by rest or nitroglycerin.

Test-Taking Strategy: Use the process of elimination. Noting that the question indicates that the presence of chest pain is usually relieved by nitroglycerin or rest will direct you to the correct option. Also eliminate options that are comparable or alike; variant and crescendo angina are both forms of unstable angina. Review the characteristics of the various types of angina if you had difficulty with this question.

Reference: Ignatavicius, D., & Workman, M. (2010). Medical-surgical nursing: Patient-centered collaborative care (6th ed., p. 848). St. Louis: Saunders.

Cognitive Ability: Understanding

Client Needs: Physiological Integrity

Integrated Process: Nursing Process/Assessment

Content Area: Adult Health/Cardiovascular Awarded 0.0 points out of 1.0 possible points. 159.ID: 383706082 Methylergonovine (Methergine) is prescribed for a client to control postpartum bleeding. Which action does the nurse take before administering the medication? Checking the episiotomy site Palpating the client's bladder Checking the client's blood pressure Correct Ensuring that the uterus is contracted Rationale: Methylergonovine is an oxytocic that stimulates contraction of the uterus and causes arterial vasoconstriction. It is used for the prevention and treatment of postpartum and postabortal hemorrhage caused by uterine atony or subinvolution. Because the medication causes arterial vasoconstriction and hypertension, the nurse checks the client’s blood pressure before administering the medication. There is no information to indicate that the client has had an episiotomy. Although the nurse may palpate the client’s bladder, this action is unrelated to the use of the medication.

Test-Taking Strategy: Use the process of elimination. There is no information in the question that indicates that the client had an episiotomy, so eliminate the option of checking the episiotomy site. Recalling the action of the medication will assist you in eliminating the remaining incorrect options. Review the procedure for administering methylergonovine if you had difficulty with this question.

Reference: Perry, S., Hockenberry, M., Lowdermilk, D., & Wilson, D. (2010). Maternal-child nursing care (4th ed., pp. 580, 581). St. Louis: Mosby.

Cognitive Ability: Applying

Client Needs: Physiological Integrity

Integrated Process: Nursing Process/Implementation

Content Area: Pharmacology Awarded 0.0 points out of 1.0 possible points. 160.ID: 383706068 A nurse is teaching a client with angina pectoris who is being discharged from the hospital about managing chest pain at home. Which statement by the client indicates a need for further teaching? "I need to keep fresh nitroglycerin available in case I need it." "I need to check the expiration date on the nitroglycerin bottle." "If I have any chest pain, I need to stop what I am doing and sit or lie down." "If I get chest pain, I should put 3 nitroglycerin tablets under my tongue and then go to the emergency department if that doesn’t work." Correct Rationale: The client is instructed to place 1 nitroglycerin tablet under the tongue and allow it to dissolve. The client waits 5 minutes for relief. A total of 3 nitroglycerin tablets are taken (the client waits 5 minutes after each for relief). If no relief is obtained after the administration of 3 nitroglycerin tablets (each 5 minutes apart), the client should call for transportation to the emergency department. The client needs to obtain instructions from the physician about the procedure for administing nitroglycerin, because some physicians advise the client to take 1 nitroglycerin and then, if no relief is obtained, call for transportation to the emergency department. The remaining statements reflect accurate understanding of the nurse’s teaching.

Test-Taking Strategy: Use the process of elimination, noting the strategic words "need for further teaching," which indicate a negative event query and the need to select the incorrect client statement. Noting the words “put 3 nitroglycerin tablets under my tongue” will assist in directing you to this option as incorrect. Review teaching points for the client with angina pectoris if you had difficulty with this question.

Reference: Ignatavicius, D., & Workman, M. (2010). Medical-surgical nursing: Patient-centered collaborative care (6th ed., p. 856). St. Louis: Saunders.

Cognitive Ability: Evaluating

Client Needs: Physiological Integrity

Integrated Process: Teaching and Learning

Content Area: Adult Health/Cardiovascular Awarded 0.0 points out of 1.0 possible points. 161.ID: 383707949 A nurse develops a list of home care instructions for a client who is wearing a halo fixation device after sustaining a cervical fracture. Which instructions should the nurse include? Select all that apply.

  Use a straw to drink. Correct Avoid sexual activity while the vest is in place. Apply powder under the vest to prevent irritation. Use caution when leaning forward or backward. Correct Wear snug clothing to prevent the device from shifting. Do not drive, because full range of vision is impaired with the device. Correct Rationale: A halo fixation (stabilization) device is used to prevent the head and neck from moving after a neck injury. Straws are used to drink, and meat and other foods are cut into small pieces to facilitate swallowing. The halo fixation device is not removed. Sexual activity does not have to be avoided; the client is instructed to use a position of comfort. Powders and lotions are used sparingly or not at all to prevent buildup of moisture and subsequent skin breakdown. The weight of the halo device alters balance; therefore the client should use caution when leaning forward or backward. The client is instructed to wear loose clothing with a large neck. The client is not to drive, because full range of vision is impaired with the device. The halo fixation device should not shift; if it does, the physician must be notified. The client is taught to sleep with the head supported with a small pillow to prevent unnecessary pressure and discomfort.

Test-Taking Strategy: Read each option carefully. Thinking about the purpose of the halo fixation device and visualizing its appearance will assist you in identifying the correct home care instructions. Review client instructions for the halo fixation device if you had difficulty with this question.

Reference: Ignatavicius, D., & Workman, M. (2010). Medical-surgical nursing: Patient-centered collaborative care (6th ed., p. 996). St. Louis: Saunders.

Cognitive Ability: Applying

Client Needs: Safe and Effective Care Environment

Integrated Process: Teaching and Learning

Content Area: Adult Health/Neurological Awarded 0.0 points out of 1.0 possible points. 162.ID: 383710057 A nurse is assessing a client who is experiencing chest pain. Which of the following observations indicates to the nurse that the pain is most likely a result of angina? The pain is relieved by rest and nitroglycerin. Correct The pain is relieved by the administration of an antacid. The pain is relieved by the administration of an antiinflammatory medication. The pain is relieved with an upright sitting position and the administration of an analgesic. Rationale: Anginal pain may radiate to the left shoulder, arm, neck, or jaw. It is often precipitated by exertion or stress, has few associated symptoms, and is relieved by rest and nitroglycerin. Antacids provide relief from chest pain (squeezing heartburn pain) caused by esophageal or gastric disorders. An upright sitting position, the administration of an analgesic, or the administration of an antiinflammatory medication often relieves chest pain (sudden, sharp, and stabbing) caused by pericarditis.

Test-Taking Strategy: Use the process of elimination and focus on the subject, anginal pain. Think about the pathophysiology associated with angina and recall that rest and nitroglycerin relieve pain caused by angina. This will direct you to the correct option. Review the clinical manifestations and characteristics of angina if you had difficulty with this question.

Reference: Ignatavicius, D., & Workman, M. (2010). Medical-surgical nursing: Patient-centered collaborative care (6th ed., pp. 853, 854, 856). St. Louis: Saunders.

Cognitive Ability: Analyzing

Client Needs: Physiological Integrity

Integrated Process: Nursing Process/Assessment

Content Area: Adult Health/Cardiovascular Awarded 0.0 points out of 1.0 possible points. 163.ID: 383706640 A nurse has provided nutrition instructions to a mother of an infant. Which statement by the mother indicates to the nurse that the mother requires further instruction? "It’s best to use cow's milk, as long as it’s whole milk and not skim." Correct "When I start feeding solid foods, I might need to add water to the food." "When the baby starts to take juices, I shouldn’t warm the juice, because that will destroy the vitamin C." "The baby will get the right nutrition if I feed breast milk or store-bought formula that’s been fortified with iron.” Rationale: Cow’s milk (whole, skim, 1%, 2%) is not recommended in the first 12 months of life. It contains too little iron, its high renal solute load and unmodified derivatives can put small infants at risk for dehydration, and the tough, hard curd is difficult for infants to digest. In addition, skim milk and reduced-fat milk deprive the infant of needed calories and essential fatty acids. Breast milk or commercially prepared iron-fortified formula should be used to feed the infant and provides adequate nutrition throughout the infancy period. The remaining options are correct statements about feeding an infant.

Test-Taking Strategy: Use the process of elimination and note the strategic words "mother requires further instruction," which indicate a negative event query and the need to select the incorrect client statement. Reading each option carefully and recalling the disadvantages of cow’s milk will direct you to the correct option. Review nutrition requirements for infants if you had difficulty with this question.

Reference: McKinney, E., James, S., Murray, S., & Ashwill, J. (2009). Maternal-child nursing (3rd ed., pp. 88, 90). St. Louis: Elsevier.

Cognitive Ability: Evaluating

Client Needs: Health Promotion and Maintenance

Integrated Process: Teaching and Learning

Content Area: Nutrition Awarded 0.0 points out of 1.0 possible points. 164.ID: 383703671 A pediatric nurse is caring for a hospitalized toddler. Which of the following activities does the nurse deem the most appropriate for the toddler? Singing games Watching videos Simple board games Large building blocks Correct Rationale: The toddler is developing the use of motor skills and enjoys manipulating small objects such as blocks, push-pull toys, and toy people, cars, and animals. Therefore large building blocks are appropriate for a client of this age. Singing games, simple board games, and videos are appropriate for the preschooler.

Test-Taking Strategy: Note the age group of the child and think about the related developmental stage to answer the question. Recalling that play activities should meet the need of the toddler’s increased use of motor skills will help direct you to the correct option. Review age-related activities and toys for the toddler if you had difficulty with this question.

Reference: McKinney, E., James, S., Murray, S., & Ashwill, J. (2009). Maternal-child nursing (3rd ed., p. 108). St. Louis: Elsevier.

Cognitive Ability: Applying

Client Needs: Health Promotion and Maintenance

Integrated Process: Nursing Process/Planning

Content Area: Developmental Stages Awarded 0.0 points out of 1.0 possible points. 165.ID: 383709206 A nurse in a physician's office is reviewing the medical record of a child with a diagnosis of lactose intolerance. Which of the following findings does the nurse expect to see documented in the child's record? Fatty stools Episodes of foul-smelling ribbonlike stools Episodes of profuse watery diarrhea and vomiting Episodes of cramping abdominal pain and excessive flatus Correct Rationale: Manifestations of lactose intolerance include diarrhea that is frothy (but not fatty), abdominal distention, cramping abdominal pain, and excessive flatus. The presence of fatty stools may indicate a problem with bile flow. Foul-smelling ribbonlike stool is a clinical manifestation of Hirschsprung disease. Profuse watery diarrhea and vomiting is one clinical manifestation of celiac disease.

Test-Taking Strategy: Focus on the diagnosis. Think about the pathophysiology associated with lactose intolerance to identify its associated clinical manifestations and answer correctly. Review lactose intolerance if you had difficulty with this question.

Reference: McKinney, E., James, S., Murray, S., & Ashwill, J. (2009). Maternal-child nursing (3rd ed., p. 1127). St. Louis: Elsevier.

Cognitive Ability: Understanding

Client Needs: Physiological Integrity

Integrated Process: Nursing Process/Assessment

Content Area: Child Health/Gastrointestinal Awarded 0.0 points out of 1.0 possible points. 166.ID: 383712470 A nurse is providing dietary instructions to the mother of a child with celiac disease. The nurse tells the mother that it is acceptable to give the child: Boiled rice Correct Cooked pasta Warm oatmeal Baked macaroni and cheese Rationale: Celiac disease is a disorder in which the affected person has an intolerance to wheat, rye, barley, and oats. Dietary management is the mainstay of treatment for the child with celiac disease. Products containing gluten (e.g., wheat, rye, barley, oats, hydrolyzed vegetable protein) should be eliminated from the child’s diet. This includes most pasta and baked products, as well as many breakfast cereals. Corn, rice, and millet may be substituted as grains. Vitamin supplements, especially fat-soluble vitamins and folate, may be needed in the early period of treatment to correct deficiencies.

Test-Taking Strategy: Use the process of elimination. Eliminate the options that are comparable or alike in that they are both pasta products. To select from the remaining options, recall that wheat, rye, barley, and oats must be eliminated from the diet; this will direct you to the correct option. Review the diet for the child with celiac disease if you had difficulty with this question.

Reference: McKinney, E., James, S., Murray, S., & Ashwill, J. (2009). Maternal-child nursing (3rd ed., p. 1130). St. Louis: Elsevier.

Cognitive Ability: Applying

Client Needs: Physiological Integrity

Integrated Process: Teaching and Learning

Content Area: Child Health/Gastrointestinal Awarded 0.0 points out of 1.0 possible points. 167.ID: 383713110 A nurse admitting a newborn to the nursery notes that the physician has documented that the newborn has a gastroschisis. The nurse performs an assessment, expecting to note that the viscera are: Inside the abdominal cavity and under the skin Inside the abdominal cavity and under the dermis Outside the abdominal cavity, not covered with a sac Correct Outside the abdominal cavity but inside a translucent sac covered with peritoneum and amniotic membrane Rationale: Gastroschisis is a defect of the abdominal wall in which the viscera are outside the abdominal cavity and not covered with a sac. An umbilical hernia is usually located inside the abdominal cavity and under the dermis or under the skin. An omphalocele is located outside the abdominal cavity and inside a translucent sac covered with peritoneum and amniotic membrane.

Test-Taking Strategy: Use the process of elimination. Eliminate the options that are comparable or alike; the dermis is a layer of the skin. To select correctly from the remaining options, it is necessary to know that gastroschisis is a defect of the abdominal wall in which the viscera are outside the abdominal cavity and not covered by a sac. Review the characteristics of gastroschisis if you had difficulty with this question.

Reference: McKinney, E., James, S., Murray, S., & Ashwill, J. (2009). Maternal-child nursing (3rd ed., p. 1101). St. Louis: Elsevier.

Cognitive Ability: Understanding

Client Needs: Physiological Integrity

Integrated Process: Nursing Process/Assessment

Content Area: Newborn Awarded 0.0 points out of 1.0 possible points. 168.ID: 383707998 A nurse is monitoring a child with intussusception for signs of peritonitis. For which of the following findings, indicative of this complication, does the nurse notify the physician? Increased alertness Increased heart rate Correct A sausage-shaped abdominal mass Diarrhea and the passage of bloody mucous stool Rationale: Intussusception is an invagination of a section of the intestine into the distal bowel. It is the most common cause of bowel obstruction in children ages 3 months to 6 years. The classic signs of intussusception are a sausage-shaped abdominal mass and the passage of bloody ("currant jelly") stool and diarrhea. The nurse monitors the child closely for signs of sepsis, peritonitis, and shock. Possible indicators of peritonitis include fever, increased heart rate, changes in the level of consciousness or in blood pressure, and respiratory distress.

Test-Taking Strategy: Use the process of elimination. Recalling the classic signs of intussusception will assist you in eliminating the options that identify these signs (i.e., sausage-shaped abdominal mass and the passage of bloody [currant jelly] stool and diarrhea). From the remaining options, think about the signs of peritonitis to direct you to the correct option. Review the characteristics of intussusception and the signs of peritonitis if you had difficulty with this question.

Reference: McKinney, E., James, S., Murray, S., & Ashwill, J. (2009). Maternal-child nursing (3rd ed., p. 1124). St. Louis: Elsevier.

Cognitive Ability: Analyzing

Client Needs: Physiological Integrity

Integrated Process: Nursing Process/Implementation

Content Area: Child Health/Gastrointestinal Awarded 0.0 points out of 1.0 possible points. 169.ID: 383713179 The nurse, auscultating the breath sounds of a client, hears these sounds. What are they?
  Rhonchi Crackles Wheezes Correct Vesicular Rationale: Wheezes are continuous squeaky, musical sounds, the result of air rushing through narrowed airways. They are associated with inflammation, bronchospasm, edema, secretions, or pulmonary vessel engorgement. Rhonchi are continuous low-pitched, coarse snoring sounds associated with thick, tenacious secretions; sputum production; or obstruction by a foreign body. Crackles are discontinuous popping sounds caused by air moving into previously deflated airways (they sound like air is being rolled between the fingers near the ear). Vesicular sounds are rustling sounds heard over the peripheral lung fields. They are a normal finding.

Test-Taking Strategy: Listen to the sound and note that it is a continuous squeaky, musical sound. Think about the characteristics of each breath sound identified in the options to direct you to the correct one. Review the characteristics of each breath sound identified in the options if you had difficulty with this question.

Reference: Ignatavicius, D., & Workman, M. (2010). Medical-surgical nursing: Patient-centered collaborative care (6th ed., p. 563). St. Louis: Saunders.

Cognitive Ability: Analyzing

Client Needs: Physiological Integrity

Integrated Process: Nursing Process/Assessment

Content Area: Adult Health/Respiratory Awarded 0.0 points out of 1.0 possible points. 170.ID: 383703631 A registered nurse is planning client assisgnments for the day. There is a licensed practical nurse and a nursing assistant on the team. Which client is the appropriate choice for the nursing assistant? A client with hemophilia who needs assistance with shaving A client with pneumonia who requires frequent oropharyngeal suctioning A client with rheumatoid arthritis who needs assistance with feeding and ambulation Correct A client with heart failure who needs daily weights and monitoring of intake and output Rationale: When a nurse delegates aspects of a client’s care to another staff member, the nurse assigning the task is responsible for ensuring that each task is appropriately assigned on the basis of the educational level and competency of the staff member. A client with hemophilia is at risk for bleeding and should be cared for by a licensed nurse. Suctioning, an invasive treatment, must be performed by a licensed nurse. Although a nursing assistant may weigh a client and monitor intake and output, this client has heart failure and should be monitored by a licensed nurse who will be able to assess breath sounds and detect changes in the client’s status. Feeding a client, turning and repositioning a client, assisting with hygiene measures, and ambulating a client are noninvasive interventions and that may be assigned to a nursing assistant. Therefore the most appropriate client for the nursing assistant is the client with rheumatoid arthritis who needs assistance with feeding and ambulation.

Test-Taking Strategy: Use the process of elimination, focusing on the subject, assignment to a nursing assistant. Think about the activities that the nursing assistant can perform and recall that they cannot perform invasive treatments or care for clients who require monitoring for complications of their disorder. Review the principles of delegating tasks if you had difficulty with this question.

References: Huber, D. (2010). Leadership and nursing care management (4th ed., pp. 242, 243). St. Louis: Saunders.
Potter, P., & Perry, A. (2009). Fundamentals of nursing (7th ed., pp. 400-402). St. Louis: Mosby.

Cognitive Ability: Applying

Client Needs: Safe and Effective Care Environment

Integrated Process: Nursing Process/Implementation

Content Area: Delegating/Prioritizing Awarded 0.0 points out of 1.0 possible points. 171.ID: 383706623 A nurse is monitoring a client with bronchogenic carcinoma for signs of superior vena cava syndrome. For which early sign of this oncological emergency does the nurse assess the client? Dyspnea Cyanosis Hypotension Stokes sign Correct Rationale: In superior vena cava syndrome, the superior vena cava is compressed or obstructed by tumor growth. Early signs and symptoms, which generally occur in the early morning, include edema of the face, especially around the eyes (periorbital edema), and complaints of tightness of a shirt or blouse collar (Stokes sign). As the compression worsens, the client experiences edema of the hands and arms, dyspnea, erythema of the upper body, and epistaxis (nosebleeds). Late (and life-threatening) signs and symptoms include hemorrhage, cyanosis, mental status changes resulting from lack of blood to the brain, decreased cardiac output, and hypotension (low blood pressure). Death may result if the compression is not relieved.

Test-Taking Strategy: Use the process of elimination. Noting the strategic word "early" will assist you in answering correctly. Also remember that dyspnea, cyanosis, and hypotension are late signs. If you are unfamiliar with superior vena cava syndrome, review this oncological emergency.

Reference: Ignatavicius, D., & Workman, M. (2010). Medical-surgical nursing: Patient-centered collaborative care (6th ed., pp. 436, 437). St. Louis: Saunders.

Cognitive Ability: Analyzing

Client Needs: Physiological Integrity

Integrated Process: Nursing Process/Assessment

Content Area: Adult Health/Oncology Awarded 0.0 points out of 1.0 possible points. 172.ID: 383704500 A nurse is caring for a client who has undergone transsphenoidal hypophysectomy to remove a microadenoma of the pituitary gland. Which of these findings would be of greatest concern to the nurse? Urinary specific gravity is low Correct Blood pressure is 138/80 mm Hg. The client complains of a dry mouth. The client frequently performs deep-breathing exercises. Rationale: After transsphenoidal hypophysectomy, the client is monitored for transient diabetes insipidus. In a client with diabetes insipidus, the urinary specific gravity is low and urine output is excessive. A blood pressure of 138/80 mm Hg is not cause for concern. The client will have nasal packing and a mustache dressing and may complain of dry mouth because of the necessity for mouth breathing. The nurse would provide frequent oral rinses and apply petroleum jelly to dry lips. The client should perform frequent deep-breathing exercises (coughing is contraindicated) to help prevent pulmonary complications.

Test-Taking Strategy: Use the process of elimination and consider the anatomical location of the surgical procedure. Noting the words "greatest concern" and thinking about the complications of this surgical procedure will direct you to the correct option. Review expected findings and the complications associated with transsphenoidal hypophysectomy if you had difficulty with this question

Reference: Ignatavicius, D., & Workman, M. (2010). Medical-surgical nursing: Patient-centered collaborative care (6th ed., p. 1433). St. Louis: Saunders.

Cognitive Ability: Analyzing

Client Needs: Physiological Integrity

Integrated Process: Nursing Process/Analysis

Content Area: Adult Health/Endocrine Awarded 0.0 points out of 1.0 possible points. 173.ID: 383703683 The nurse notes the presence of drainage on the mustache dressing of a client who has undergone transsphenoidal hypophysectomy. The initial nursing action is to: Contact the surgeon Change the dressing Document the findings Check the drainage for glucose Correct Rationale: After hypophysectomy, the nurse assesses the client’s nasal drainage for quantity, quality, and the presence of glucose, which. indicates that the fluid is cerebrospinal fluid (CSF). Therefore the initial nursing action is to check the drainage for glucose. If glucose is present in the drainage, the surgeon must be notified. The nurse would not change the dressing unless a specific prescription to do so has been given. The nurse would document the findings, but this would not be the initial action.

Test-Taking Strategy: Note the strategic word "initial." Think about the anatomical location of the surgery and focus on the subject, drainage on the dressing. Remembering that CSF leakage is a complication of this type of surgery and recalling the characteristics of this type of fluid will direct you to the initial nursing action. Review the complications of hypophysectomy and the actions to take if drainage is noted if you had difficulty with this question.

Reference: Ignatavicius, D., & Workman, M. (2010). Medical-surgical nursing: Patient-centered collaborative care (6th ed., p. 1431). St. Louis: Saunders.

Cognitive Ability: Applying

Client Needs: Physiological Integrity

Integrated Process: Nursing Process/Implementation

Content Area: Adult Health/Endocrine Awarded 0.0 points out of 1.0 possible points. 174.ID: 383706676 A nurse is monitoring a client who has undergone subtotal thyroidectomy for signs of postoperative complications. Which of the following findings would be a matter of concern for the nurse as an indication of hypocalcemia? The client's temperature is 100.6˚ F. The client's voice is hoarse and weak. The client's heart rate is 92 beats/min. The client complains of a tingling sensation around the mouth. Correct Rationale: The parathyroid glands may be damaged or their blood supply impaired during thyroid surgery. Hypocalcemia and tetany result when the level of parathyroid hormone decreases. The nurse assesses the client for symptoms of tingling around the mouth or of the toes and fingers and muscle twitching, signs of calcium deficiency. Calcium gluconate or calcium chloride for intravenous administration must be available in an emergency situation (severe hypocalcemia). Hoarseness and a weak voice may result if the laryngeal nerve is damaged during surgery; however, the client is reassured that the hoarseness is usually temporary. Tachycardia and a fever are symptoms of thyroid storm or thyroid crisis, a complication of thyroid surgery. Postoperative thyroid storm is rare because clients receive antithyroid medications, beta-blockers, corticosteroids, and iodides before thyroid surgery.

Test-Taking Strategy: Focus on the subject, the presence of hypocalcemia. It is necessary to recall the signs and symptoms of hypoglycemia to answer correctly. Thinking about the purpose and action of calcium in the body will also assist in answering. Review the signs and symptoms of hypocalcemia if you had difficulty with this question.

Reference: Ignatavicius, D., & Workman, M. (2010). Medical-surgical nursing: Patient-centered collaborative care (6th ed., p. 1454). St. Louis: Saunders.

Cognitive Ability: Analyzing

Client Needs: Physiological Integrity

Integrated Process: Nursing Process/Assessment

Content Area: Adult Health/Endocrine Awarded 0.0 points out of 1.0 possible points. 175.ID: 383705065 A nurse is monitoring a client who was brought to the emergency department in an unresponsive state and is now being treated for hyperglycemic hyperosmolar nonketotic syndrome (HHNS). Which of the following findings indicates to the nurse that fluid replacement is inadequate? Increased urine output Potassium level of 3.6 mEq/L Blood pressure of 128/80 mm Hg Level of consciousness remains unchanged Correct Rationale: The initial objective for fluid replacement in HHNS is to increase circulating blood volume. Lack of any improvement in the level of consciousness may indicate an inadequate rate of fluid replacement or a reduction in plasma osmolarity. Increased urine output indicates an improvement in the client’s status. Potassium level of 3.6 mEq/L and a blood pressure of 128/80 mm Hg are normal findings.

Test-Taking Strategy: Note the strategic words "fluid replacement is inadequate," which indicate a negative event query and the need to select the unwanted outcome. Eliminate the options that are comparable or alike in that they are normal findings. To select from the remaining options, note the word “unchanged” in the correct option. Review care of the client with HHNS and expected outcomes of treatment if you had difficulty with this question.

Reference: Black, J., & Hawks, J. (2009). Medical-surgical nursing: Clinical management for positive outcomes (8th ed., pp. 1093, 1094). St. Louis: Saunders.

Cognitive Ability: Evaluating

Client Needs: Physiological Integrity

Integrated Process: Nursing Process/Evaluation

Content Area: Adult Health/Endocrine Awarded 0.0 points out of 1.0 possible points. 176.ID: 383706619 A nurse is reviewing the laboratory results of a client in the emergency department with diabetic ketoacidosis (DKA). Which laboratory result would the nurse expect to note? Creatinine 1.0 mg/dL Serum bicarbonate of 12 mEq/L Correct Blood urea nitrogen (BUN) of 15 mg/dL Negative results on urinary ketone testing Rationale: In DKA, the serum bicarbonate concentration is less than 15 mEq/L. The BUN and creatinine levels may be increased because of dehydration, and urinary ketones are present. Additionally, the serum glucose is greater than 300 mg/dL and the serum pH is less than 7.35. A BUN reading of 15 mg/dL, a creatinine level of 1.0 mg/dL, and a serum bicarbonate concentration of 12 mEq/L are normal findings.

Test-Taking Strategy: Focus on the subject, the laboratory findings in DKA. First eliminate the option that includes the word “negative.” Next eliminate the options that are comparable or alike in that they represent normal laboratory values. Review laboratory findings in DKA if you had difficulty with this question.

Reference: Swearingen, P. (2008). All-in-one care planning resource: Medical-surgical, pediatric, maternity & psychiatric nursing care plans (2nd ed., pp. 387, 388). St. Louis: Mosby.

Cognitive Ability: Analyzing

Client Needs: Physiological Integrity

Integrated Process: Nursing Process/Assessment

Content Area: Adult Health/Endocrine Awarded 0.0 points out of 1.0 possible points. 177.ID: 383710513 Propylthiouracil (PTU) has been prescribed for a client with Graves disease, and the nurse provides instructions to the client about the medication. For which of the following occurrences does the nurse tells the client to contact the physician? Fatigue Diaphoresis Sore throat Correct Heat intolerance Rationale: Graves disease (hyperthyroidism) occurs as a result of excessive thyroid hormone secretion. Propylthiouracil is an antithyroid medication that blocks thyroid hormone production. One adverse effect is agranulocytosis, and the client is instructed to report signs of infection such as a sore throat. Fatigue may or may not occur in this disorder; however, it is not an adverse effect of the medication. One manifestation of Graves disease is heat intolerance, and the client may experience diaphoresis even when the environmental temperature is comfortable for others.

Test-Taking Strategy: Focus on the client’s diagnosis and think about the manifestations that occur in this disorder. Next note that the question asks about the manifestation requiring physician notification. Sore throat is a sign of infection. Review the manifestations of Graves disease and the adverse effects of this medication if you had difficulty with this question.

References: Ignatavicius, D., & Workman, M. (2010). Medical-surgical nursing: Patient-centered collaborative care (6th ed., p. 1453). St. Louis: Saunders.
Lehne, R. (2010). Pharmacology for nursing care (7th ed., p. 699). St. Louis: Saunders.

Cognitive Ability: Analyzing

Client Needs: Physiological Integrity

Integrated Process: Teaching and Learning

Content Area: Adult Health/Endocrine Awarded 0.0 points out of 1.0 possible points. 178.ID: 383709215 A nurse is providing information to a client with diabetes insipidus who will be taking desmopressin acetate (DDAVP) by way of the nasal route. For which of the following occurrences does the nurse tell the client to contact the physician? Abdominal cramps Stuffy or runny nose Headache and nausea Correct Decreased urine output Rationale: Diabetes insipidus is a disorder of water metabolism caused by a deficiency of antidiuretic hormone (ADH). Desmopressin, a synthetic form of antidiuretic hormone, causes increased resorption of water and a resultant decrease in urine output (an expected outcome). One adverse effect of the medication is water intoxication. Early signs of water intoxication include headache, nausea, shortness of breath, drowsiness, and listlessness. The physician is notified if these signs occur. Abdominal cramping is a side effect, not an adverse effect, of the parenteral form of the medication. A runny or stuffy nose is a side effect, not an adverse effect, of the medication.

Test-Taking Strategy: Use the process of elimination. Recalling the pathophysiology of diabetes insipidus and the action of desmopressin will assist you in determining that water intoxication is an adverse effect. From this point, recalling the signs of water intoxication will direct you to the correct option. Review the adverse effects of DDAVP if you had difficulty with this question.

Reference: Edmunds, M. (2010). Introduction to clinical pharmacology (6th ed., p. 343). St. Louis: Mosby.

Cognitive Ability: Analyzing

Client Needs: Physiological Integrity

Integrated Process: Teaching and Learning

Content Area: Adult Health/Endocrine Awarded 0.0 points out of 1.0 possible points. 179.ID: 383702948 A client with diabetes mellitus calls the clinic nurse and reports that she has been vomiting during the night and now has diarrhea. Which question does the nurse make a priority of asking the client? "Do you have a fever?" "Did you eat any breakfast?" "Are you urinating frequently?" "Have you tested your blood glucose?" Correct Rationale: The priority is learning whether the client has tested her blood glucose level and what the level is. During illness, the client should follow the “sick-day rules” for diabetics. The blood glucose level should be checked every 4 hours. Urine is tested for ketones if the blood glucose level is greater than 240 mg/dL. The client should take the prescribed insulin or oral antidiabetic medication and should drink 8 to 12 oz of sugar-free liquids every hour when awake. The client should notify the physician if she becomes ill. The remaining options are questions that may be asked, but they are not the priority.

Test-Taking Strategy: Use the process of elimination and recall the sick-day rules for the client with diabetes mellitus. Note the relationship between the client’s diagnosis and the correct option. Review the sick-day rules for clients with diabetes mellitus if you had difficulty with this question.

Reference: Ignatavicius, D., & Workman, M. (2010). Medical-surgical nursing: Patient-centered collaborative care (6th ed., p. 1512). St. Louis: Saunders.

Cognitive Ability: Applying

Client Needs: Physiological Integrity

Integrated Process: Nursing Process/Assessment

Content Area: Adult Health/Endocrine Awarded 0.0 points out of 1.0 possible points. 180.ID: 383708540 A nurse is providing information to a client who will be self-administering regular insulin about storage of the insulin. The nurse tells the client: That placing the vial near heat or in sunlight will not affect the insulin To freeze unopened vials and remove a vial from the freezer 24 hours before opening it That insulin stored at room temperature causes more discomfort on injection than does cold insulin That the vial in current use may be kept at room temperature for as long as 1 month without significant loss of activity Correct Rationale: Insulin preparations are stable at room temperature for as long as 1 month without significant loss of chemical activity. Direct sunlight and extreme heat must be avoided. Insulin should not be frozen. If the insulin is frozen, the insulin should be discarded and the nurse should obtain another vial. Insulin stored at room temperature causes less discomfort on injection than does cold insulin.

Test-Taking Strategy: Knowledge of the principles for storing insulin is needed to answer this question. Eliminate the option that includes the word “heat” and the option that contains the word “freezer.” Careful reading of the remaining options will assist you in answering correctly. Review insulin storage principles if you are unfamiliar with them.

Reference: Lehne, R. (2010). Pharmacology for nursing care (7th ed., pp. 670, 685, 686). St. Louis: Saunders.

Cognitive Ability: Applying

Client Needs: Physiological Integrity

Integrated Process: Teaching and Learning

Content Area: Adult Health/Endocrine Awarded 0.0 points out of 1.0 possible points. 181.ID: 383704568 A nurse reviews the medical record of a client with histoplasmosis. Which clinical manifestation of this infection does the nurse expect to see documented? Neurological deficits Cardiac dysrhythmias Gastrointestinal disturbances Flulike pulmonary symptoms Correct Rationale: Histoplasmosis is a fungal infection of the lungs. The client typically experiences a flulike pulmonary illness with cough, chest pain, dyspnea, headache, fever, arthralgia, anorexia, erythema nodosum, hepatomegaly, and splenomegaly. Neurological disturbances, gastrointestinal disturbances, and cardiac dysrhythmias are not associated with this infection.

Test-Taking Strategy: Use the process of elimination. Recalling that histoplasmosis is a fungal infection of the lungs will direct you to the correct option. Review the clinical manifestations of with histoplasmosis if you had difficulty with this question.

Reference: Ignatavicius, D., & Workman, M. (2010). Medical-surgical nursing: Patient-centered collaborative care (6th ed., p. 371). St. Louis: Saunders.

Cognitive Ability: Analyzing

Client Needs: Physiological Integrity

Integrated Process: Nursing Process/Assessment

Content Area: Adult Health/Respiratory Awarded 0.0 points out of 1.0 possible points. 182.ID: 383704517 A client with a medical history of diabetes mellitus is found to have sarcoidosis, and oral prednisone is prescribed. The nurse provides instructions to the client about the medication and tells the client to: Eat foods that are high in sodium Decrease the daily dose of insulin Eat foods that are low in potassium Closely monitor the blood glucose level Correct Rationale: Sarcoidosis is a multisystem granulomatous disorder of unknown cause that can affect virtually any organ. Corticosteroids are the primary pharmacological therapy for sarcoidosis. When taken over an extensive period of time, prednisone may cause increased blood glucose and decreased potassium. It may also cause increased blood pressure and sodium retention. The client with diabetes mellitus would be instructed to closely monitor the blood glucose level. The physician may prescribe an increased dose of insulin while the client is taking this medication.

Test-Taking Strategy: Use the process of elimination. Recalling the side effects of prednisone will assist you in answering this question, but also be sure to note the relationship between the client’s medical history and the correct option. Review the side effects of prednisone if you had difficulty with this question.

Reference: Lehne, R. (2010). Pharmacology for nursing care (7th ed., p. 855). St. Louis: Saunders.

Cognitive Ability: Applying

Client Needs: Physiological Integrity

Integrated Process: Teaching and Learning

Content Area: Adult Health/Respiratory Awarded 0.0 points out of 1.0 possible points. 183.ID: 383711481 A client with tuberculosis will be taking pyrazinamide (Pyrazinamide), and the nurse provides instructions about the adverse effects of the medication. For which of the following occurrences does the nurse tell the client to contact the physician? Headache Yellow skin Correct Difficulty sleeping Nasal congestion Rationale: Pyrazinamide is an antitubercular medication that is given in conjunction with other antitubercular medications. Adverse effects include hepatotoxicity, thrombocytopenia, and anemia. The nurse instructs the client to contact the physician if he experiences jaundice (yellow skin or eyes), unusual tiredness, fever, loss of appetite, or hot, painful, or swollen joints. Headache, nasal congestion, and difficulty sleeping are not associated with the use of this medication.

Test-Taking Strategy: Focus on the subject, an adverse effect of this medication. Recalling that this medication is hepatotoxic will direct you to the correct option. Yellow skin is a sign of hepatotoxicity. Review the adverse effects of this medication if you had difficulty with this question.

Reference: Lehne, R. (2010). Pharmacology for nursing care (7th ed., p. 1045). St. Louis: Saunders.

Cognitive Ability: Analyzing

Client Needs: Physiological Integrity

Integrated Process: Teaching and Learning

Content Area: Pharmacology Awarded 0.0 points out of 1.0 possible points. 184.ID: 383702946 A client who was involved in a high-speed motor vehicle crash is brought to the emergency department. Which of the following findings indicates to the nurse that the client has sustained flail chest? Asymmetrical chest movement Correct Complaints of mild chest discomfort Increased breath sounds on auscultation Deep respirations, 18 breaths/min Rationale: Flail chest is a thoracic injury resulting in paradoxical motion of the chest-wall segment. This causes the mediastinal structures to swing back and forth with respiration. The nurse specifically notes paradoxical breathing (asymmetrical chest movement); oscillation of the mediastinum; increasing dyspnea; rapid, shallow respirations; accessory muscle breathing; restlessness; decreased breath sounds on auscultation; cyanosis; and anxiety related to difficulty breathing. The client will also complain of severe chest pain.

Test-Taking Strategy: Use the process of elimination. Eliminate the options that include phrasing that indicates a normal or near-normal finding: “mild,” “increased breath sounds,” and “18 breaths/min.” Once these options have been eliminated, remember the specific details of the client’s diagnosis to confirm the correct option. Review the clinical manifestations of flail chest if you had difficulty with this question.

Reference: Black, J., & Hawks, J. (2009). Medical-surgical nursing: Clinical management for positive outcomes (8th ed., pp. 1659, 1660). St. Louis: Saunders.

Cognitive Ability: Analyzing

Client Needs: Physiological Integrity

Integrated Process: Nursing Process/Assessment

Content Area: Critical Care Awarded 0.0 points out of 1.0 possible points. 185.ID: 383708594 A client is found to have hypoxemic respiratory failure. Which finding does the nurse expect to note on review of the results of the client's arterial blood gas analysis? Pao2 of 73 mm Hg, Paco2 of 62 mm Hg Pao2 of 58 mm Hg, Paco2 of 35 mm Hg Pao2 of 60 mm Hg, Paco2 of 45 mm Hg Pao2 of 49 mm Hg, Paco2 of 32 mm Hg Correct Rationale: The normal Pao2 is 80 to 100 mm Hg and the normal Paco2 is 35 to 45 mm Hg. Respiratory failure may be classified, according to the underlying pathophysiology, as hypoxemic respiratory failure or hypoxemic-hypercapnic respiratory failure. Hypoxemic respiratory failure is characterized by a low Pao2 (less than 55 mm Hg) and a normal or low Paco2. Hypoxemic-hypercapnic respiratory failure is characterized by a low Pao2 (less than 55 mm Hg) and an increased Paco2 (greater than 50 mm Hg). A Pao2 of 49 mm Hg and Paco2 of 32 mm Hg is the only option that characterizes hypoxemic respiratory failure.

Test-Taking Strategy: Use the process of elimination and focus on the subject, hypoxemic respiratory failure. Note that the correct option identifies the lowest Pao2 level. Review blood gas findings in the client with hypoxemic respiratory failure if you had difficulty with this question.

Reference: Lehne, R. (2010). Pharmacology for nursing care (7th ed., p. 685). St. Louis: Saunders.

Cognitive Ability: Analyzing

Client Needs: Physiological Integrity

Integrated Process: Nursing Process/Analysis

Content Area: Critical Care Awarded 0.0 points out of 1.0 possible points. 186.ID: 383707958 A client with acute gouty arthritis is being started on medication therapy with indomethacin (Indocin). The nurse, providing medication instructions, and tells the client to take the medication: At bedtime With food Correct 1 hour before meals On an empty stomach Rationale: Indomethacin is a nonsteroidal antiinflammatory medication that produces analgesic and antiinflammatory effects by inhibiting prostaglandin synthesis. Adverse effects include ulceration of the esophagus, stomach, duodenum, and small intestine. Therefore the client is instructed to take the medication with food.

Test-Taking Strategy: Use the process of elimination. Recall the classification of the medication to answer correctly. Also note that the incorrect options are comparable or alike in that they indicate that the medication should be taken on an empty stomach. Review client teaching points for indomethacin if you had difficulty with this question.

Reference: Lehne, R. (2010). Pharmacology for nursing care (7th ed., pp. 837, 844). St. Louis: Saunders.

Cognitive Ability: Applying

Client Needs: Physiological Integrity

Integrated Process: Teaching and Learning

Content Area: Pharmacology Awarded 0.0 points out of 1.0 possible points. 187.ID: 383705037 An emergency department nurse is monitoring a client who sustained a severe inhalation burn injury during a fire in which the client was trapped in an enclosed space. The nurse auscultates the client's trachea and notes that the previously heard wheezing sounds have disappeared. The nurse most appropriately: Continues monitoring the client Notifies the emergency department physician Correct Documents the client's improvement in the medical record Removes the oxygen mask and fits the client with a nasal cannula Rationale: A client with an inhalation burn injury is at risk for respiratory complications. Upper-airway edema and inhalation injury are most notable in the trachea and main stem bronchi. Auscultation of these areas reveals wheezes, which are a sign of obstruction. A client with a severe inhalation injury may sustain such progressive obstruction that within a short time he or she cannot force air through the narrowed airways. As a result, the wheezing sounds disappear. This finding indicates impending airway obstruction and demands immediate intubation. The nurse would notify the physician immediately. Continuing to monitor the client, documenting the client's improvement in the medical record, and removing the oxygen mask and fitting the client with a nasal cannula are all incorrect and would delay necessary interventions.

Test-Taking Strategy: Focus on the data in the question and think about the pathophysiology associated with an inhalation burn injury. Noting the strategic word "severe" in the question and the words “wheezing sounds have disappeared” will help direct you to the correct option. Review nursing care of the client with an inhalation burn injury if you had difficulty with this question.

Reference: Ignatavicius, D., & Workman, M. (2010). Medical-surgical nursing: Patient-centered collaborative care (6th ed., p. 529). St. Louis: Saunders.

Cognitive Ability: Applying

Client Needs: Physiological Integrity

Integrated Process: Nursing Process/Implementation

Content Area: Critical Care Awarded 0.0 points out of 1.0 possible points. 188.ID: 383703645 A nurse reviews arterial blood gas values and notes a pH of 7.50 and a Pco2 of 30 mm Hg. The nurse interprets these values as indicative of: Metabolic acidosis Metabolic alkalosis Respiratory acidosis Respiratory alkalosis Correct Rationale: The normal pH is 7.35 to 7.45. The normal Pco2 is 35 to 45 mm Hg. In respiratory alkalosis, the pH will be higher than normal and the Pco2 will be low. The only option that reflects these conditions is respiratory alkalosis.

Test-Taking Strategy: Use the process of elimination. Remember that when an alkalotic condition exists, the pH will be high. Next, recall that in a respiratory condition, the Pco2 will move in the opposite direction from the pH. The only option that represents these conditions is the correct option. Review the process of arterial blood gas analysis if you had difficulty with this question.

Reference: Ignatavicius, D., & Workman, M. (2010). Medical-surgical nursing: Patient-centered collaborative care (6th ed., pp. 208, 210, 211). St. Louis: Saunders.

Cognitive Ability: Analyzing

Client Needs: Physiological Integrity

Integrated Process: Nursing Process/Assessment

Content Area: Acid-Base Awarded 0.0 points out of 1.0 possible points. 189.ID: 383702997 A nurse provides information to a client with chronic obstructive pulmonary disease (COPD) about methods of alleviating shortness of breath while the client is eating. Which statement by the client indicates a need for further instruction? "I should rest before I eat." "I should use my bronchodilator 30 minutes before I eat." "Pursed-lip breathing will help relieve my shortness of breath." "I should eat three meals a day, and the biggest meal should be at suppertime." Correct Rationale: Shortness of breath is the most common problem related to eating for a client with COPD. The client can ease dyspnea by resting before meals. A bronchodilator used 30 minutes before a meal may be helpful if the meal-related dyspnea is a result of bronchospasm or secretions. The biggest meal of the day is planned for the time when the client is hungriest and most rested. Most clients are increasingly tired at the end of the day. Four to six small meals per day are preferable to three larger ones. Pursed-lip or abdominal breathing may alleviate dyspnea.

Test-Taking Strategy: Focus on the subject, alleviating shortness of breath during eating, and note the strategic words "need for further instruction," which indicate a negative event query and the need to select the incorrect client statement. Using your knowledge of the principles of alleviating dyspnea will direct you to the correct option. Also note the words “biggest meal should be at suppertime” in the correct option. Review the care of the client with COPD if you had difficulty with this question.

Reference: Ignatavicius, D., & Workman, M. (2010). Medical-surgical nursing: Patient-centered collaborative care (6th ed., p. 633). St. Louis: Saunders.

Cognitive Ability: Evaluating

Client Needs: Physiological Integrity

Integrated Process: Teaching and Learning

Content Area: Adult Health/Respiratory Awarded 0.0 points out of 1.0 possible points. 190.ID: 383712487 A postoperative client with deep-vein thrombosis is at risk for pulmonary embolism. For which characteristic sign or symptom of this complication does the nurse monitor the client? Pleuritic chest pain Correct Slowed heart rate Chills and a high fever Decreased respiratory rate Rationale: Pulmonary embolism, whch results in blockage of the main artery of the lung or one of its branches, occurs when an object, such as a clot, or substance travels from elsewhere in the body through the bloodstream (embolism). The characteristic signs or symptoms of pulmonary embolism are dyspnea, tachypnea, tachycardia, and pleuritic chest pain (sharp, stabbing pain on inspiration). The client may become apprehensive and restless. A low-grade fever may occur. Chills and a high fever are characteristics of an infection.

Test-Taking Strategy: Focus on the subject, pulmonary embolism. First eliminate the option that describes characteristics of an infection (i.e., chills and fever). To select from the remaining options, recall the pathophysiology of pulmonary embolism, which will assist you in answering correctly. Review the signs and symptoms of pulmonary embolism if you had difficulty with this question.

Reference: Ignatavicius, D., & Workman, M. (2010). Medical-surgical nursing: Patient-centered collaborative care (6th ed., p. 679). St. Louis: Saunders.

Cognitive Ability: Analyzing

Client Needs: Physiological Integrity

Integrated Process: Nursing Process/Assessment

Content Area: Adult Health/Cardiovascular Awarded 0.0 points out of 1.0 possible points. 191.ID: 383710589 A nurse caring for a client 24 hours after a radical neck dissection notes the presence of serosanguineous drainage in the portable wound suction device attached to the surgical site. On the basis of this finding, the nurse should: Contact the physician Document the findings Correct Ask the physician to remove the drains Increase the pressure on the wound suction device Rationale: Immediately after radical neck dissection, the client will have a wound drain in the neck, attached to portable suction. The nurse places the client in the semi-Fowler position to minimize postoperative edema and monitors the neck drainage for volume and color. Sanguineous and serosanguineous drainage is expected in the 72 hours after surgery. Once drainage has stopped, the wound drains are removed. Because the findings detailed in the question are expected, the nurse would document them.

Test-Taking Strategy: Use the process of elimination. Eliminate the options that are comparable or alike and indicate contacting the physician. Next recall that the nurse would not increase pressure on the wound suction device without a prescription to do so and eliminate this option. Also note the words "24 hours after" in the question and recall that serosanguineous drainage is normal at this time. Review expected findings after radical neck dissection if you had difficulty with this question.

Reference: Black, J., & Hawks, J. (2009). Medical-surgical nursing: Clinical management for positive outcomes (8th ed., p. 1561). St. Louis: Saunders.

Cognitive Ability: Applying

Client Needs: Physiological Integrity

Integrated Process: Nursing Process/Implementation

Content Area: Adult Health/Respiratory Awarded 0.0 points out of 1.0 possible points. 192.ID: 383703625 Although previously well controlled with glyburide (Diabeta), a client’s fasting blood glucose has been running 180 to 200 mg/dL. On reviewing the client's record, which medication, recently added to the client's regimen, does the nurse recognize as a possible contributor to the hyperglycemia? Phenelzine (Nardil) Atenolol (Tenormin) Allopurinol (Zyloprim) Lithium carbonate (Lithobid) Correct Rationale: Glyburide is a hypoglycemic medication. If the client takes a corticosteroid, thiazide diuretic, or lithium carbonate concurrently, the effect of the glyburide is diminished. Lithium carbonate, an antimanic medication, may increase the blood glucose level. Phenelzine is a monoamine oxidase inhibitor. Atenolol is a beta-blocker. Allopurinol is a xanthine oxidase inhibitor. These medications may amplify the effects of an oral hypoglycemic.

Test-Taking Strategy: The subject of the question is hyperglycemia. This question requires that you know the classifications of the medications in the options and correlate them with their potential hypoglycemic or hyperglycemic effects. It is necessary to recall that lithium carbonate may increase the blood glucose level. Review the medications in the options and their effects on blood glucose level if you are unfamiliar with them.

Reference: Edmunds, M. (2010). Introduction to clinical pharmacology (6th ed., p. 339). St. Louis: Mosby.

Cognitive Ability: Analyzing

Client Needs: Physiological Integrity

Integrated Process: Nursing Process/Analysis

Content Area: Adult Health/Endocrine Awarded 0.0 points out of 1.0 possible points. 193.ID: 383704525 A nurse is performing an assessment of a client with suspected pheochromocytoma. Which clinical manifestation does the nurse expect to note? Weight gain Flushed face Client complaint of diarrhea A blood pressure higher than the normal range Correct Rationale: Pheochromocytoma is a catecholamine-producing tumor arising from cells of the adrenal medulla and sympathetic ganglia. These tumors release excessive amounts of catecholamines, mainly norepinephrine, with associated symptoms that include the "five P’s": pressure (paroxysmal increases in blood pressure), palpitations, pallor, perspiration (profuse and generalized), and pain (paroxysmal pulsatile headaches, chest, and abdominal pain). Other clinical signs include weight loss, constipation, tremors, hypertensive retinopathy, hyperglycemia, and hypercalcemia.

Test-Taking Strategy: Use the process of elimination. Recalling that pheochromocytoma is a catecholamine-producing tumor arising from cells of the adrenal medulla and recalling the effects of catecholamines will direct you to the correct option. Review the clinical manifestations of pheochromocytoma if you had difficulty with this question.

Reference: Black, J., & Hawks, J. (2009). Medical-surgical nursing: Clinical management for positive outcomes (8th ed., p. 1052). St. Louis: Saunders.

Cognitive Ability: Analyzing

Client Needs: Physiological Integrity

Integrated Process: Nursing Process/Assessment

Content Area: Adult Health/Endocrine Awarded 0.0 points out of 1.0 possible points. 194.ID: 383712485 The nurse is the first responder at the scene of a bus crash. After a quick assessment of the victims, which one does the nurse care for first? A victim with a twisted ankle and leg bruises A victim with an open fracture of the arm that is bleeding profusely Correct A victim who is anxiously moving among the victims, searching for her husband A victim who is unresponsive, with severe swelling and bruising around the eyes, and is not breathing Rationale: In a disaster situation, the nurse must triage victims on the basis of severity of injury and potential for recovery. Victims with life-threatening injuries that are readily corrected are classified as emergent and are the first priority (in this case, the victim with an open fracture of the arm that is bleeding profusely). Victims with injuries that do not require immediate treatment but that will need to be treated within 1 to 2 hours are classified as urgent and are the second priority (here, the victim with a twisted ankle and leg bruises). Victims with no injuries, those whose condition is noncritical, and victims who are ambulatory are classified as delayed (nonurgent) and are the third priority (in this case, the victim who is anxiously walking about among the victims, searching for her husband). The victim who is unresponsive and not breathing, with severely swollen and bruised eyes, is most likely dead after having sustained a severe head injury and, in this situation, is not the priority. The nurse does not have the resources necessary to save this client, whereas it may be possible for the nurse to save the client who is bleeding profusely by applying pressure to the bleeding site.

Test Taking Strategy: Use your knowledge of the principles of prioritizing to answer the question. Noting the description of the victim who is likely dead will assist you in eliminating this option. To select from the remaining options, use the ABCs — airway, breathing, and circulation — to find the correct option. Bleeding involves the circulation. Also remember that in a disaster situation you must think about survivability and which victim can be saved. Review triage systems if you had difficulty with this question.

Reference: Ignatavicius, D., & Workman, M. (2010). Medical-surgical nursing: Patient-centered collaborative care (6th ed., pp. 160, 161). St. Louis: Saunders.

Cognitive Ability: Applying

Client Needs: Safe and Effective Care Environment

Integrated Process: Nursing Process/Implementation

Content Area: Delegating/Prioritizing Awarded 0.0 points out of 1.0 possible points. 195.ID: 383705057 A client with suspected HIV infection has positive results on enzyme-linked immunosorbent assay (ELISA) and Western blot tests. The plasma HIV RNA level is assessed, and the result is reported as 8000 copies/mL. The nurse interprets the results of the HIV RNA test as indicating that the client: Is at low risk for AIDS Correct Is at high risk for AIDS Is at risk for HIV infection Requires further testing to confirm the presence of HIV Rationale: The test most commonly used to screen clients for HIV infection is the ELISA. A positive ELISA result must be confirmed with the Western blot technique. The plasma HIV RNA level is an indication of the amount of virus in the person’s serum, which is a reflection of active viral replication, or viral load. The steeper the rate of increase in plasma HIV RNA, the greater the risk of disease progression. A plasma HIV RNA level of less than 10,000 copies/mL is considered indicative of low risk for the development of AIDS. Levels between 10,000 and 100,000 copies/mL represent a doubled risk for AIDS, and a result of more than 100,000 copies/mL indicates a high risk for AIDS.

Test-Taking Strategy: Use the process of elimination. Eliminate the options that are comparable or alike in that they indicate that the client is at risk for HIV infection or AIDS. To select from the remaining options, focus on the data in the question and recall that 8000 copies/mL indicates a low risk. Review the HIV RNA test if you had difficulty with this question.

References: Ignatavicius, D., & Workman, M. (2010). Medical-surgical nursing: Patient-centered collaborative care (6th ed., pp. 373, 374). St. Louis: Saunders.
Pagana, K., & Pagana, T. (2009). Mosby’s diagnostic and laboratory test reference (9th ed., pp. 524-527). St. Louis: Mosby.

Cognitive Ability: Understanding

Client Needs: Physiological Integrity

Integrated Process: Nursing Process/Assessment

Content Area: Laboratory Values Awarded 0.0 points out of 1.0 possible points. 196.ID: 383703675 A nurse developing a plan of care for a client with HIV infection identifies several concerns. List them in order of priority, from highest to lowest. Incorrect Despair Possible infection Fatigue Decreased nutrition The correct order is: Possible infection Decreased nutrition Fatigue Despair Rationale: HIV is a retrovirus that infects T-lymphocytes, disabling the body’s ability to fight infection. Therefore infection is a major complication of HIV, and it is the top priority. The nutritional status of the client may range from normal early in the disease process to extreme cachexia and wasting syndrome with progressive disease. For this reason, decreased nutrition is the second priority. Fatigue is one result of the disease process, and energy management is important. Of the options presented, fatigue is the third priority. Although despair might be an appropriate concern for the client, it is a psychosocial issue and would therefore be the fourth priority in this situation.

Test-Taking Strategy: Use your knowledge of Maslow’s Hierarchy of Needs theory. First list despair, the psychosocial need, as the fourth priority. To order the remaining options, focus on the client’s diagnosis and note that infection is the most life-threatening, making it the highest-priority diagnosis. Recalling that poor nutritional status can lead to cachexia and wasting syndrome with progressive disease will help you determine that decreased nutrition is the second-priority diagnosis, leaving fatigue as the third priority. Review priority concerns in the care of the client with HIV if you had difficulty with this question.

References: Black, J., & Hawks, J. (2009). Medical-surgical nursing: Clinical management for positive outcomes (8th ed., pp. 2105, 2107). St. Louis: Saunders.
Ignatavicius, D., & Workman, M. (2010). Medical-surgical nursing: Patient-centered collaborative care (6th ed., p. 364). St. Louis: Saunders.

Cognitive Ability: Analyzing

Client Needs: Safe and Effective Care Environment

Integrated Process: Nursing Process/Planning

Content Area: Adult Health/Immune Awarded 0.0 points out of 1.0 possible points. 197.ID: 383710518 A client is found to have iron-deficiency anemia, and ferrous sulfate (Feosol) is prescribed. The nurse tells the client that it is best to take the medication with: Milk Apple juice Orange juice Correct Scrambled eggs Rationale: Ferrous sulfate is an iron preparation, and the client is instructed to take the medication with orange juice or another vitamin C–containing product to increase absorption of the iron. Milk and eggs inhibit the absorption of iron. Orange juice is higher in vitamin C than apple juice.

Test-Taking Strategy: Use the process of elimination. Recalling that vitamin C increases the absorption of iron will assist you in eliminating the food items that do not contain vitamin C. To select from the remaining options, note the strategic word "best," which will direct you to the correct option. Review client teaching points for the administration of ferrous sulfate if you had difficulty with this question.

Reference: Lehne, R. (2010). Pharmacology for nursing care (7th ed., pp. 636, 644). St. Louis: Saunders.

Cognitive Ability: Applying

Client Needs: Physiological Integrity

Integrated Process: Teaching and Learning

Content Area: Pharmacology Awarded 0.0 points out of 1.0 possible points. 198.ID: 383705086 A nurse is monitoring a client with pheochromocytoma who is receiving an intravenous (IV) infusion of phentolamine. Which vital sign does the nurse monitor most closely during the infusion? Apical pulse Respirations Temperature Blood pressure Correct Rationale: Pheochromocytoma is a catecholamine-producing tumor arising from cells of the adrenal medulla and sympathetic ganglia. These tumors release excessive amounts of catecholamines, mainly norepinephrine. Most affected clients present with hypertensive crises that may be treated with phentolamine (Regitine). This medication, a short-acting alpha-adrenergic blocker, and is given by way of IV bolus or infusion for a hypertensive crisis. The nurse checks the client’s blood pressure every 15 minutes during infusion of the medication.

Test-Taking Strategy: Focus on the client’s diagnosis and think about its associated pathophysiology. Recalling that hypertensive crisis is a concern with this diagnosis will direct you to the correct option. Review the pathophysiology of pheochromocytoma and the action of phentolamine if you had difficulty with this question.

Reference: Gahart, B., & Nazareno, A. (2010). Intravenous medications (26th ed., p. 1072). St. Louis: Mosby.

Cognitive Ability: Applying

Client Needs: Physiological Integrity

Integrated Process: Nursing Process/Implementation

Content Area: Pharmacology Awarded 0.0 points out of 1.0 possible points. 199.ID: 383711489 Oral prednisone 5 mg/day has been prescribed for a client with a chronic respiratory disorder, and the nurse provides instructions to the client about the medication. The nurse tells the client to: Take the medication on an empty stomach Take half of the daily dose if weight gain occurs Stop taking the medication if the ankles begin to swell Call the physician if a fever, sore throat, or muscle aches develop Correct Rationale: Prednisone is a glucocorticoid that prevents or diminishes tissue response to an inflammatory process. The client taking this medication should notify the physician if fever, sore throat, muscle aches, or sudden weight gain or swelling occurs. The medication is taken with food or milk because it can cause gastrointestinal irritation. The client is instructed not to change the dose and to not stop taking the medication without contacting the physician. The medication must be tapered gradually under medical supervision.

Test-Taking Strategy: Use the process of elimination. General principles related to medication administration will assist you in eliminating the oprions that involve adjusting the dosage or stopping the medication. To select from the remaining options, recall the action and adverse effects of this medication and remember that the medication alters the inflammatory response; this will direct you to the correct option. Review client teaching points related to prednisone if you had difficulty with this question.

Reference: Lehne, R. (2010). Pharmacology for nursing care (7th ed., p. 855). St. Louis: Saunders.

Cognitive Ability: Applying

Client Needs: Physiological Integrity

Integrated Process: Teaching and Learning

Content Area: Pharmacology Awarded 0.0 points out of 1.0 possible points. 200.ID: 383705035 As a nurse prepares to administer medications to an assigned client, the client asks, "Why don't you just leave me alone?" What is the best response by the nurse? "Don't yell at me." "These medications will help you feel much better." "Why do you want to be left alone? I’m here to help you." "I can see that you’re upset. Would you like to talk about it?" Correct Rationale: The nurse addresses the client’s feelings by responding, "I can see that you’re upset. Would you like to talk about it?" The client may not want to talk about his or her feelings, but the nurse has provided the opportunity to the client and is maintaining an open channel of communication. "Don't yell at me" is confrontational and nonfacilitative and imposes control. "These medications will help you feel much better" avoids the client’s feelings. "Why do you want to be left alone? I’m here to help you" is not a facilitative response and may place the client, who may not be able to express his or her feelings, on the defensive.

Test-Taking Strategy: Use your knowledge of therapeutic communication techniques and focus on the client’s statement. Remember the need to address the client’s feelings and maintain an open channel of communication. This will direct you to the correct option. Review therapeutic communication techniques if you had difficulty with this question.

Reference: Potter, P., & Perry, A. (2009). Fundamentals of nursing (7th ed., pp. 352-357). St. Louis: Mosby.

Cognitive Ability: Applying

Client Needs: Psychosocial Integrity

Integrated Process: Communication and Documentation

Content Area: Mental Health Awarded 0.0 points out of 1.0 possible points. 201.ID: 383703635 A nurse is assessing a client with hepatitis for signs of jaundice. Which area does the nurse check, knowing that it will provide the best data regarding the presence of jaundice? Lips Soles Palms Mucous membranes Correct Rationale: Assessment of the skin, sclera, and mucous membranes provides the best data regarding the presence of jaundice. The color of the lips provides data regarding the presence of cyanosis. Although assessment of the skin provides adequate data regarding jaundice, the soles and palms are not the best areas of skin for assessment.

Test-Taking Strategy: Use the process of elimination. Eliminate the options that are comparable or alike (i.e., soles and palms). To select from the remaining options, note the strategic word "best" in the question and recall that the lips are assessed for the presence of cyanosis. Review assessment techniques used in the detection of jaundice if you had difficulty with this question.

Reference: Ignatavicius, D., & Workman, M. (2010). Medical-surgical nursing: Patient-centered collaborative care (6th ed., p. 1359). St. Louis: Saunders.

Cognitive Ability: Applying

Client Needs: Health Promotion and Maintenance

Integrated Process: Nursing Process/Assessment

Content Area: Adult Health/Gastrointestinal Awarded 0.0 points out of 1.0 possible points. 202.ID: 383713181 A mother calls the emergency department and tells the nurse that her 3-year-old child drank ammonia from a bottle while the mother was cleaning house. The nurse tells the mother to immediately: Induce vomiting Call the child's physician Bring the child to the emergency department Encourage the child to drink water or milk in small amounts Correct Rationale: Administering water or milk can dilute the toxic effects of acid or alkali ingestion. These substances, when ingested, may cause burning of tissue along the gastrointestinal tract. Because these caustic substances continue to cause damage until they are neutralized, induction of emesis is contraindicated. Although calling the child’s physician and bringing the child to the emergency department may each be necessary, they are not the actions to be taken immediately, because they would delay necessary treatment.

Test-Taking Strategy: Use the process of elimination. Eliminate the options that are comparable or alike in that they delay necessary treatment. To select from the remaining options, focus on the substance that the child drank. Knowing that ammonia is a corrosive substance will direct you to the correct option. Review immediate treatment measures for poisoning if you had difficulty with this question.

Reference: McKinney, E., James, S., Murray, S., & Ashwill, J. (2009). Maternal-child nursing (3rd ed., p. 870). St. Louis: Elsevier.

Cognitive Ability: Applying

Client Needs: Physiological Integrity

Integrated Process: Nursing Process/Implementation

Content Area: Critical Care Awarded 0.0 points out of 1.0 possible points. 203.ID: 383703651 Calcium disodium edetate (EDTA) and British antilewisite (BAL, dimercaprol) is prescribed for a child with lead poisoning. What does the nurse ask the child's mother before administering the medications? "Can your child swallow pills?" "Has your child been running a fever?" "Does your child have an allergy to peanuts?" Correct "How long has your child been exposed to the lead?" Rationale: BAL may be used in conjunction with EDTA to treat lead poisoning. BAL is administered by way of deep intramuscular injection. EDTA is administered by way of the intravenous or intramuscular route. BAL must not be used in the presence of a glucose-6-phosphate dehydrogenase deficiency (G6PD) or peanut allergy, nor should it be given in conjunction with iron. Therefore the nurse must ask about allergy to peanuts. The assessment questions noted in the remaining options are unrelated to the administration of this medication.

Test-Taking Strategy: Knowledge regarding the contraindications of BAL is required to answer this question. It is necessary to know that BAL must not be used in the presence of a G6PD or peanut allergy. Review the medications used to treat lead poisoning if you had difficulty with this question.

Reference: Perry, S., Hockenberry, M., Lowdermilk, D., & Wilson, D. (2010). Maternal-child nursing care (4th ed., p. 1436). St. Louis: Mosby.

Cognitive Ability: Analyzing

Client Needs: Physiological Integrity

Integrated Process: Nursing Process/Assessment

Content Area: Pharmacology Awarded 0.0 points out of 1.0 possible points. 204.ID: 383706070 A child is brought to the emergency department by ambulance after swallowing several capsules of acetaminophen (Tylenol). Which medication does the nurse prepare, anticipating that it will be prescribed to treat the child? Protamine sulfate Succimer (Chemet) Phytonadione (vitamin K) Acetylcysteine (Mucomyst) Correct Rationale: Acetylcysteine is the antidote for acetaminophen overdose. It is administered orally with juice or cola or through a nasogastric tube. Protamine sulfate is the antidote to heparin. Succimer is used in the treatment of lead poisoning. Vitamin K is the antidote to warfarin sodium (Coumadin).

Test-Taking Strategy: Knowledge regarding the antidote for acetaminophen overdose is required to answer this question. Think about the antidotes used for certain medication and use the process of elimination. Remember that acetylcysteine is the antidote for acetaminophen overdose. Review the common medication antidotes if you had difficulty with this question.

Reference: McKinney, E., James, S., Murray, S., & Ashwill, J. (2009). Maternal-child nursing (3rd ed., p. 871). St. Louis: Elsevier.

Cognitive Ability: Understanding

Client Needs: Physiological Integrity

Integrated Process: Nursing Process/Planning

Content Area: Pharmacology Awarded 0.0 points out of 1.0 possible points. 205.ID: 383703687 A female client with rheumatoid arthritis is taking 3.6 g of acetylsalicylic acid (aspirin) daily in a divided dose. At the physician's office, the client tells the nurse that she has been experiencing ringing in the ears over the past few days. The nurse tells the client that: This is expected and nothing to be concerned about It is important to drink at least 10 glasses of water a day to prevent ringing in the ears This is a sign of toxicity, so the aspirin will be discontinued and replaced with a nonsteroidal antiinflammatory medication The physician will probably withhold the aspirin until the symptoms have subsided, then resume the aspirin at a lower dosage Correct Rationale: Salicylism is a syndrome that begins to develop when the aspirin level climbs just slightly above the therapeutic level. Overt signs include tinnitus (ringing in the ears), sweating, headache, and dizziness. If salicylism develops, aspirin is withheld until the symptoms have subsided; therapy should then be resumed with a small reduction in dosage. Tinnitus is not an expected finding. Drinking water will not prevent tinnitus. A nonsteroidal antiinflammatory medication will not be prescribed, because these medications’ chemical properties are similar to those of aspirin.

Test-Taking Strategy: Use the process of elimination. Recalling that tinnitus denotes salicylism will direct you to the correct option. Remember that when a toxic effect occurs, the medication will probably be withheld until the symptoms subside. Review salicylate toxicity if you had difficulty with this question.

Reference: Lehne, R. (2010). Pharmacology for nursing care (7th ed., p. 845). St. Louis: Saunders.

Cognitive Ability: Applying

Client Needs: Physiological Integrity

Integrated Process: Nursing Process/Implementation

Content Area: Adult Health/Immune Awarded 0.0 points out of 1.0 possible points. 206.ID: 383712468 A nurse is caring for a client who sustained burn injuries on the anterior lower legs and anterior thorax. What percentage of the client’s body, according to the Rule of Nines, has been affected? 36% Correct 42% 45% 31.5% Rationale: According to the Rule of Nines, the anterior lower legs equal 18% and the anterior thorax equals 18%, for a total of 36%.

Test-Taking Strategy: Knowledge regarding the rule of nines is required to answer this question. Remember that 9 (head), 18 (arms), 36 (thorax), 36 (legs), and 1 (perineum) equal 99. Review the rule of nines if you had difficulty with this question.

Reference: Ignatavicius, D., & Workman, M. (2010). Medical-surgical nursing: Patient-centered collaborative care (6th ed., p. 531). St. Louis: Saunders.

Cognitive Ability: Understanding

Client Needs: Physiological Integrity

Integrated Process: Nursing Process/Assessment

Content Area: Adult Health/Integumentary Awarded 0.0 points out of 1.0 possible points. 207.ID: 383706625 A client is brought to the emergency department after sustaining smoke inhalation injury during a fire in the client's home. The nurse plans to first: Check for a patent IV line Provide emotional support to the client Provide the client with 100% oxygen by mask Correct Administer intravenous (IV) fluids as prescribed Rationale: When smoke is inhaled, carbon monoxide binds with hemoglobin, displacing oxygen. A high carboxyhemoglobin level impairs tissue oxygenation, resulting in tissue asphyxia. Providing the client with 100% oxygen by mask reverses this condition. The nurse would next ensure that the client has a patent IV line and then administer fluids as prescribed. The nurse would also maintain body temperature, provide wound care as needed, and provide comfort and emotional support.

Test-Taking Strategy: Note the strategic word “first.” Use your knowledge of Maslow’s Hierarchy of Needs theory to eliminate the psychosocial option, providing emotional support. To select from the remaining options, use the ABCs — airway, breathing, and circulation. This will direct you to the correct option. Review care of the client with an inhalation injury if you had difficulty with this question.

Reference: Copstead, L., & Banasik, J. (2010). Pathophysiology (4th ed., pp. 1271, 1272). St. Louis: Mosby.

Cognitive Ability: Applying

Client Needs: Physiological Integrity

Integrated Process: Nursing Process/Planning

Content Area: Critical Care Awarded 0.0 points out of 1.0 possible points. 208.ID: 383702985 A client with emphysema is receiving theophylline (Theo-24). While providing dietary instructions, the nurse tells the client that it is acceptable to consume: Cola Coffee Hot cocoa Apple juice Correct Rationale: Theophylline is a methylxanthine bronchodilator. Caffeine is a methylxanthine with pharmacologic properties like those of theophylline. For this reason, caffeine can intensify the adverse effects of theophylline on the central nervous system and heart. In addition, caffeine competes with theophylline for drug-metabolizing enzymes, thereby causing the theophylline level to increase. Because of these interactions, individuals taking theophylline should avoid caffeine-containing beverages such as cola, coffee, tea, and cocoa, as well as other caffeine-containing products.

Test-Taking Strategy: Use the process of elimination. Eliminate the options that are comparable or alike in that they are beverages that contain caffeine. Review client teaching points for theophylline if you had difficulty with this question.

Reference: Lehne, R. (2010). Pharmacology for nursing care (7th ed., p. 897). St. Louis: Saunders.

Cognitive Ability: Applying

Client Needs: Physiological Integrity

Integrated Process: Teaching and Learning

Content Area: Pharmacology Awarded 0.0 points out of 1.0 possible points. 209.ID: 383705055 Testing of the plasma theophylline level in a client who is receiving a continuous intravenous infusion of theophylline reveals a level of 20 mcg/mL. The nurse interprets this result as: Below the therapeutic range In excess of the therapeutic range At the top of the therapeutic range Correct In the middle of the therapeutic range Rationale: The normal therapeutic range for theophylline is 10 to 20 mcg/mL. The client’s value, 20 mcg/mL, is at the top of the therapeutic range.

Test-Taking Strategy: Knowledge regarding the therapeutic theophylline range will assist you in answering this question. Recalling that the level is between 10 to 20 mcg/mL will direct you to the direct option. Review the therapeutic theophylline level if you had difficulty with this question.

References: Gahart, B., & Nazareno, A. (2010). Intravenous medications (26th ed., p. 83). St. Louis: Mosby.
Lehne, R. (2010). Pharmacology for nursing care (7th ed., p. 897). St. Louis: Saunders.

Cognitive Ability: Understanding

Client Needs: Physiological Integrity

Integrated Process: Nursing Process/Evaluation

Content Area: Pharmacology Awarded 0.0 points out of 1.0 possible points. 210.ID: 383702967 Fluticasone propionate (Advair) and albuterol (Ventolin HFA), administered by inhalation twice daily, are prescribed for a client with asthma. The nurse, providing information to the client about administration of the medication, tells the client to use the: Fluticasone propionate immediately after inhaling the albuterol Albuterol immediately after inhaling the fluticasone propionate Fluticasone propionate several minutes before inhaling the albuterol Albuterol several minutes before inhaling the fluticasone propionate Correct Rationale: Albuterol is a bronchodilator, and fluticasone propionate is a corticosteroid. In a client receiving a bronchodilator by way of inhalation concomitantly with corticosteroid therapy, the client is instructed to use the bronchodilator several minutes before corticosteroid inhalation to enhance penetration of the corticosteroid into the bronchial tree. Therefore the other options are incorrect.

Test-Taking Strategy: Focus on the names of the medications and their actions. Recall that albuterol is a bronchodilator and fluticasone propionate is a corticosteroid. (Medications that end with the letters -sone are corticosteroids). It makes sense that bronchodilation would be best before inhalation of a corticosteroid. Review the procedure for administering an inhaled bronchodilator and corticosteroid if you had difficulty with this question.

Reference: Lehne, R. (2010). Pharmacology for nursing care (7th ed., p. 904). St. Louis: Saunders.

Cognitive Ability: Applying

Client Needs: Physiological Integrity

Integrated Process: Teaching and Learning

Content Area: Pharmacology Awarded 0.0 points out of 1.0 possible points. 211.ID: 383702987 A nurse assessing the wound of a client with a stage 3 pressure ulcer and notes that the wound bed is pale. The nurse interprets this finding as a possible indication that: The wound is healthy The wound is improving Necrotic tissue is present The client's hemoglobin level is low Correct Rationale: The color of a wound provides information about vascular supply, infection, the presence of healthy or necrotic tissue, and nutritional status. A pale wound bed may reflect a low hemoglobin value. Healthy full-thickness wounds have a granular beefy-red appearance. Necrotic tissue is white, yellow, gray, or black. Pseudomonas infection can produce greenish drainage in a wound bed.

Test-Taking Strategy: Use the process of elimination. Eliminate the options that are comparable or alike (wound is healthy and wound is improving). To select from the remaining options, focus on the word "pale" in the question. This will direct you to the correct option. Review the characteristics of pressure ulcers if you had difficulty with this question.

References: Ignatavicius, D., & Workman, M. (2010). Medical-surgical nursing: Patient-centered collaborative care (6th ed., pp. 489, 493). St. Louis: Saunders.
Perry, A., & Potter, P. (2010). Clinical nursing skills & techniques (7th ed., p. 471). St. Louis: Mosby.

Cognitive Ability: Evaluating

Client Needs: Physiological Integrity

Integrated Process: Nursing Process/Evaluation

Content Area: Adult Health/Integumentary Awarded 0.0 points out of 1.0 possible points. 212.ID: 383708555 A client calls the emergency department and tells the nurse that he may have come in contact with poison ivy while trimming bushes in his yard. The nurse tells the client to immediately: Contact the physician Report to the emergency department for treatment Get into the shower and rinse the skin for at least 15 minutes Correct Go to the drugstore, purchase an over-the-counter topical corticosteroid, and rub it into the exposed skin Rationale: Persons walking or working in areas where poison ivy grows should protect the skin by wearing appropriate clothing. If contact with poison ivy is suspected, symptoms may be averted by immediately rinsing the skin for 15 minutes with running water to remove the resin before skin penetration occurs. The client is also instructed to remove clothing carefully to avoid skin contact. Although a topical over-the-counter corticosteroid may relieve some of the discomfort of the poison ivy rash, this is not the action that needs to be taken immediately. Contacting the physician and coming to the emergency department for treatment are unnecessary.

Test-Taking Strategy: Note the strategic word “immediately.” Eliminate the options that are comparable or alike (contacting the physician and reporting to the emergency department). Next note that the question involves an individual who may have been exposed to poison ivy; showering will help remove the resin. Review the immediate treatment measures for poison ivy if you had difficulty with this question.

Reference: Monahan, F., Sands, J., Marek, J., Neighbors, M., & Green, C. (2007). Phipps' medical-surgical nursing: Health and illness perspectives (8th ed., p. 1884). St. Louis: Mosby.

Cognitive Ability: Applying

Client Needs: Safe and Effective Care Environment

Integrated Process: Nursing Process/Implementation

Content Area: Adult Health/Integumentary Awarded 0.0 points out of 1.0 possible points. 213.ID: 383712464 A nurse provides skin care instructions to a client with acne vulgaris. Which statement by the client indicates a need for further instruction? "I should use oil-based cosmetics." Correct "I shouldn't leave make-up on overnight." "I should avoid rubbing my face vigorously." "I should wash my face two or three times a day with a mild cleanser." Rationale: The client with acne is instructed to use water-based cosmetics and to avoid exposure to skin products that contain oil, because products that are oily may cause flare-ups. The statements in the other options are correct.

Test-Taking Strategy: Use the process of elimination and note the strategic words "need for further instruction," which indicate a negative event quety and the need to select the incorrect client statement. Knowledge regarding the basic principles of skin care and the pathophysiology of acne vulgaris will direct you to the correct option. Also note the words “oil-based” in the correct option. Review teaching points for the client with acne if you had difficulty with this question.

Reference: Copstead, L., & Banasik, J. (2010). Pathophysiology (4th ed., p. 1235). St. Louis: Mosby.

Cognitive Ability: Evaluating

Client Needs: Health Promotion and Maintenance

Integrated Process: Teaching and Learning

Content Area: Adult Health/Integumentary Awarded 0.0 points out of 1.0 possible points. 214.ID: 383710053 Oral candidiasis (thrush) develops in a client infected with HIV, and the nurse provides instruction to the client about measures to relieve the discomfort. Which statement by the client indicates a need for further instruction? "I should avoid spicy foods." "I should eat foods with a soft texture." "I should use a soft-bristled toothbrush." "I should put ice in my drinks to help soothe the discomfort." Correct Rationale: Oral candidiasis lesions make it difficult for the client to tolerate temperature extremes, so it is best for the client to consume foods and fluids that are tepid or room temperature rather than iced or hot. The other options are useful measures for alleviating the discomfort associated with this disorder.

Test-Taking Strategy: Use the process of elimination and note the strategic words "need for further instruction," which indicate a negative event query and the need to select the incorrect client statement. Eliminate the options that are comparable or alike in that they address the use of soft items. Recalling it makes sense to avoid spicy foods when mouth lesions are present will assist you in eliminating the last incorrect option. Review self-care instructions for the client with oral candidiasis if you had difficulty with this question.

Reference: Black, J., & Hawks, J. (2009). Medical-surgical nursing: Clinical management for positive outcomes (8th ed., pp. 597, 598). St. Louis: Saunders.

Cognitive Ability: Evaluating

Client Needs: Physiological Integrity

Integrated Process: Teaching and Learning

Content Area: Adult Health/Immune Awarded 0.0 points out of 1.0 possible points. 215.ID: 383710516 A client with HIV infection who has been found to have histoplasmosis is being treated with intravenous amphotericin B (Fungizone). Which parameter does the nurse check to detect the most common adverse effect of this medication? Temperature Blood pressure Peripheral pulses Intake and output Correct Rationale: Amphotericin B, an antifungal medication, is highly toxic, and infusion reactions and renal damage occur, to varying degrees, in all clients. As a means of detecting renal injury, tests of kidney function should be performed weekly and intake and output should be monitored closely. Other adverse effects include delirium, hypotension, hypertension, wheezing, and hypoxia. The remaining options are not associated with an adverse effect of the medication.

Test-Taking Strategy: Note the strategic words "most common adverse effect." Focus on the name of the medication. Recalling that this medication is nephrotoxic will direct you to the correct option. Review the adverse effects of amphotericin B if you had difficulty with this question.

Reference: Gahart, B., & Nazareno, A. (2010). Intravenous medications (26th ed., p. 102). St. Louis: Mosby.

Cognitive Ability: Analyzing

Client Needs: Physiological Integrity

Integrated Process: Nursing Process/Assessment

Content Area: Adult Health/Immune Awarded 0.0 points out of 1.0 possible points. 216.ID: 383713108 A hospitalized client scheduled for surgery is told by the physician that she is extremely anemic and will need a blood transfusion. The client, a Jehovah's Witness, tells the nurse that she is refusing the transfusion. What is the most appropriate initial nursing action? Supporting the client's decision to refuse the transfusion Correct Teaching the client ways to increase dietary intake of iron Telling the client about the importance of the blood transfusion Telling the client that if she refuses the blood transfusion, the surgery will have to be canceled Rationale: Religious beliefs can influence individual and family responses to health and illness, and religion can be a source of support and comfort for a client. Awareness of a client’s religious beliefs enables the nurse to appropriately support the client’s decisions regarding care. Initially the nurse would most appropriately be supportive of a Jehovah’s Witness’ refusal to receive a blood transfusion. The nurse might need to instruct the client in how to increase dietary intake of iron, but this would not be the initial action. Telling the client about the importance of blood transfusions and telling her that the surgery will need to be canceled if she refuses the blood transfusion are both inappropriate.

Test-Taking Strategy: Use the process of elimination and note the strategic word "initial." Recalling that the nurse needs to support the client’s decisions regarding healthcare issues will direct you to the correct option. Review cultural considerations as they relate to nursing care if you had difficulty with this question.

Reference: Perry, A., & Potter, P. (2010). Clinical nursing skills & techniques (7th ed., p. 787). St. Louis: Mosby.

Cognitive Ability: Applying

Client Needs: Safe and Effective Care Environment

Integrated Process: Nursing Process/Implementation

Content Area: Cultural Diversity Awarded 0.0 points out of 1.0 possible points. 217.ID: 383706621 A nurse is performing an assessment of a client with Ménière disease. Which question does the nurse ask to elicit data about the manifestations of this disease? "Do you have headaches?" "Have you had any loss of appetite?" "Do you have episodes of dizziness?" Correct "Have you been having any diarrhea?" Rationale: Ménière disease or syndrome, also called idiopathic endolymphatic hydrops, occurs when the normal fluid and electrolyte balance of the inner ear is disrupted. The classic triad consists of episodic true vertigo (dizziness), sensorineural hearing loss, and tinnitus. Some clients experience vertigo associated with nausea, vomiting, and ataxia. Diarrhea, headaches, and loss of appetite are not associated with this disease.

Test-Taking Strategy: Focus on the client’s diagnosis and recall the pathophysiology of this disease. Recalling that the normal fluid and electrolyte balance of the inner ear is disrupted in this disorder will direct you to the correct option. Review the manifestations of Ménière disease if you had difficulty with this question.

Reference: Ignatavicius, D., & Workman, M. (2010). Medical-surgical nursing: Patient-centered collaborative care (6th ed., p. 1127). St. Louis: Saunders.

Cognitive Ability: Analyzing

Client Needs: Physiological Integrity

Integrated Process: Nursing Process/Assessment

Content Area: Adult Health/Ear Awarded 0.0 points out of 1.0 possible points. 218.ID: 383704521 A client with chronic back pain asks a nurse about the use of complementary and alternative therapies to treat the pain. The nurse would initially: Identify the client's treatment goals Correct Share current research outcomes with the client Offer options that may be beneficial to the client Tell the client that the physician does not believe in these therapies Rationale: If a client asks a nurse about complementary and alternative therapies, the nurse should initially identify the client’s treatment goals, such as symptom management. The nurse should respect the client’s request and inquiry about these therapies. Offering options that may be beneficial to the client are appropriate but would not be the initial nursing action. Telling the client that the physician does not believe in these therapies is a violation of the client’s right to be informed about treatment options.

Test-Taking Strategy: Note the strategic word “initially.” First eliminate the option that is nontherapeutic and violates the client’s rights. To select from the remaining options, use the steps of the nursing process. The correct option is the only one that addresses the process of assessment. Review the nurse’s role in regard to complementary and alternative therapies if you had difficulty with this question.

References: Ignatavicius, D., & Workman, M. (2010). Medical-surgical nursing: Patient-centered collaborative care (6th ed., pp. 36, 56-58). St. Louis: Saunders.

Cognitive Ability: Applying

Client Needs: Safe and Effective Care Environment

Integrated Process: Nursing Process/Implementation

Content Area: Complementary and Alternative Therapies Awarded 0.0 points out of 1.0 possible points. 219.ID: 383704547 A client has been scheduled for an electronystagmography (ENG), and the nurse provides instructions to the client about the test. Which statement by the client tells the nurse that the client understands the instructions? "I need to not drink coffee before the test." Correct "I’ll need to receive sedation before the test." "I won’t be able to eat for 24 hours after the test." "I can eat a light breakfast on the morning of the test." Rationale: An ENG is a test that is sensitive in detecting both central and peripheral disease of the vestibular system in the ear. It detects nystagmus (involuntary eye movements), which can be recorded. After electrodes are taped to the skin near the eyes, procedures are performed to stimulate nystagmus. The client must fast for several hours (but not for 24 hours) before the test and should avoid caffeine-containing products for 24 to 48 hours before the test. No sedation is used for the ENG. Fluids are given to the client after the test but are introduced carefully to prevent nausea.

Test-Taking Strategy: Use the process of elimination and eliminate the options that identify opposite actions (eating and not eating before the test). To select from the remaining options, think about the purpose of the test, the procedure for performing it, and factors that might affect the results. Remember that caffeine-containing products should be avoided before this test. Review the procedure for performing an ENG if you had difficulty with this question.

Reference: Pagana, K., & Pagana, T. (2009). Mosby’s diagnostic and laboratory test reference (9th ed., p. 382). St. Louis: Mosby.

Cognitive Ability: Evaluating

Client Needs: Physiological Integrity

Integrated Process: Nursing Process/Evaluation

Content Area: Adult Health/Ear Awarded 0.0 points out of 1.0 possible points. 220.ID: 383702952 An emergency department nurse has a physician's prescription to irrigate a client's ears. List in order of priority the steps that the nurse should take in performing this procedure. Incorrect Document the completion of the procedure and how the client tolerated it. Fill an irrigating syringe with warm water. Warm tap water to body temperature. Use an otoscope to ensure that the tympanic membrane is intact. Insert the irrigating solution by directing the solution toward the wall of the ear canal. The correct order is: Use an otoscope to ensure that the tympanic membrane is intact. Warm tap water to body temperature. Fill an irrigating syringe with warm water. Insert the irrigating solution by directing the solution toward the wall of the ear canal. Document the completion of the procedure and how the client tolerated it. Rationale: Irrigation of the ear is not performed if the client has a perforated tympanic membrane or otitis media. Therefore the nurse would first use an otoscope to ensure that the tympanic membrane is intact. Once intactness of the tympanic membrane has been verified, the nurse would warm the irrigation solution to body temperature, fill an irrigating syringe with the solution, and instill the solution by directing it toward the wall of the ear canal. Finally the nurse would document that the procedure was performed and record how the client tolerated it.

Test-Taking Strategy: Focus on the subject, the steps involved in ear irrigation. Visualize this procedure to answer correctly. Remember that the tympanic membrane must be intact before irrigation is performed. Also recall that the nurse would document the procedure once it had been performed. Review the procedure for ear irrigation if you had difficulty with this question.

Reference: Proehl, J. (2009). Emergency nursing procedures (4th ed., pp. 792, 793). Philadelphia: Saunders.

Cognitive Ability: Applying

Client Needs: Physiological Integrity

Integrated Process: Nursing Process/Implementation

Content Area: Adult Health/Ear Awarded 0.0 points out of 1.0 possible points. 221.ID: 383713148 A community health nurse is preparing a poster for a health fair that will include information about the ways to prevent ear infection or ear trauma. Which prevention measures does the nurse include on the poster? Select all that apply. Always sneeze with the mouth closed. Occlude one nostril when blowing the nose. Keep the volume of headphones at the lowest setting. Correct Avoid environmental conditions involving rapid changes in air pressure. Correct Clean the external ear and canal daily in the shower or while washing the hair. Correct Be cautious when using cotton-tipped applicators to clean the external ear canal. Rationale: The client is instructed to wash the external ear and canal daily in the shower or while washing his or her hair. The client should never use a small object such as a cotton-tipped applicator to clean the external ear canal. The client is instructed to blow the nose gently and is told not to occlude a nostril when blowing the nose. The client should also wear sound protection around loud or continuous noises, avoid activities with a high risk for ear trauma, keep the volume of headphones at the lowest setting, and avoid environmental conditions involving rapid changes in air pressure.

Test-Taking Strategy: Visualize each of the actions in the options. Recall of the general principles of ear care and thinking about the effect of each action on the ear will help direct you to the correct options. Review measures for preventing ear infection or trauma if you had difficulty with this question.

Reference: Ignatavicius, D., & Workman, M. (2010). Medical-surgical nursing: Patient-centered collaborative care (6th ed., p. 1134). St. Louis: Saunders.

Cognitive Ability: Applying

Client Needs: Health Promotion and Maintenance

Integrated Process: Teaching and Learning

Content Area: Adult Health/Ear Awarded 0.0 points out of 1.0 possible points. 222.ID: 383708517 A nurse assigns a nursing assistant to care for a client who is hearing impaired and provides instructions to the nursing assistant about the effective methods for communicating with the client. Which statement by the nursing assistant indicates that further instruction is needed? "I should speak slowly and clearly to the client." "I should stand directly in front of the client when I’m talking." "I should make sure that the room is well lit when I’m talking to the client." "I should raise the volume of my voice and stand on the client's affected side when I’m talking to him." Correct Rationale: When communicating with a hearing-impaired client, the nurse should position herself or himself directly in front of the client so that the client may read the nurse’s lips. The room should be well lit, and the nurse should get the client’s attention before speaking. The nurse should move closer to the ear with better hearing and speak slowly and clearly. The nurse should speak in a normal tone of voice and keep his or her hands and other objects away from the mouth while talking to the client.

Test-Taking Strategy: Note the words "need for further instruction,” which indicate a negative event query and the need to select the incorrect statement. Focus on the subject, communicating with a hearing-impaired client. Use your knowledge of the basic principles of communication to select the correct option. Review the methods of effective communication with a hearing-impaired client if you had difficulty with this question.

Reference: Ignatavicius, D., & Workman, M. (2010). Medical-surgical nursing: Patient-centered collaborative care (6th ed., p. 1135). St. Louis: Saunders.

Cognitive Ability: Evaluating

Client Needs: Physiological Integrity

Integrated Process: Teaching and Learning

Content Area: Leadership and Management Awarded 0.0 points out of 1.0 possible points. 223.ID: 383706084 A Tensilon test is performed on a client with suspected myasthenia gravis. Which finding constitutes a positive result? A decrease in muscle strength No change in muscle strength An increase in muscle strength Correct The presence of tremors in previously weakened muscles Rationale: Myasthenia gravis is a disease that affects the myoneural junction. It is believed to result from an autoimmune response that destroys a variable number of acetylcholine receptors at the myoneural junction. A positive result on Tensilon testing is considered diagnostic. In this test, edrophonium (Tensilon), a short-acting anticholinesterase, is administered intravenously. A client with myasthenia gravis experiences a brief but significant increase in muscle strength in previously weakened muscles in response to the medication, and this response is considered a positive result. A decrease or a lack of change in muscle strength could indicate a cholinergic crisis. The presence of tremors in previously weakened muscles is unrelated to this test.

Test-Taking Strategy: Think about the pathophysiology of myasthenia gravis. Next, recall the action and effect of edrophonium, which will direct you to the correct option. Review myasthenia gravis and the Tensilon test if you had difficulty with this question.

Reference: Ignatavicius, D., & Workman, M. (2010). Medical-surgical nursing: Patient-centered collaborative care (6th ed., p. 1017). St. Louis: Saunders.

Cognitive Ability: Understanding

Client Needs: Physiological Integrity

Integrated Process: Nursing Process/Assessment

Content Area: Adult Health/Neurological Awarded 0.0 points out of 1.0 possible points. 224.ID: 383706086 A client with myasthenia gravis who has been taking pyridostigmine bromide (Mestinon) for the treatment of the disorder comes to the emergency department complaining of severe muscle weakness, and cholinergic crisis is diagnosed. Which medication does the nurse prepare for immediate use in treating the crisis? Atropine sulfate Correct Carisoprodol (Soma) Cyanocobalamin (vitamin B12) Cyclobenzaprine hydrochloride (Flexeril) Rationale: Pyridostigmine bromide is an anticholinesterase (cholinergic muscle stimulant) that prevents the destruction of acetylcholine, thus enhancing impulse transmission across the myoneural junction. Overmedication with pyridostigmine bromide can produce cholinergic crisis, manifested by increasingly severe muscle weakness. The weakness first appears in the muscles involved in chewing and swallowing and is followed by weakness of the shoulder girdle and upper extremities, respiratory muscles, and pelvic girdle and leg muscles. Treatment includes withdrawal of all cholinergic medications and immediate administration of atropine sulfate. Carisoprodol and cyclobenzaprine hydrochloride are skeletal muscle relaxants. Cyanocobalamin is used to treat vitamin B12 deficiencies such as pernicious anemia.

Test-Taking Strategy: Use the process of elimination. Eliminate the options that are comparable or alike in that they are muscle relaxants. To select from the remaining options, focus on the information in the question to direct you to the correct option. Also recall that cyanocobalamin is used to treat vitamin B12 deficiency. Review cholinergic crisis in the client with myasthenia gravis if you had difficulty with this question.

Reference: Lehne, R. (2010). Pharmacology for nursing care (7th ed., pp. 135, 138). St. Louis: Saunders.

Cognitive Ability: Understanding

Client Needs: Physiological Integrity

Integrated Process: Nursing Process/Planning

Content Area: Adult Health/Neurological Awarded 0.0 points out of 1.0 possible points. 225.ID: 383702993 A nurse provides information about activity and exercise to the wife of a client with Parkinson's disease. Which statement by the spouse indicates a need for further instruction? "He needs to have a broad base of support when ambulating." "He needs to avoid staying in one position for a prolonged period." "I should encourage him to keep his hands hanging at his side when he walks." Correct "I should help him perform range-of-motion exercises of his joints three times a day." Rationale: Parkinson’s disease is a chronic degenerative disorder that primarily affects the neurons of the basal ganglia. The client’s muscles become stiff and rigid and require increased effort to move. The client is taught to hold the hands clasped behind the back when walking to keep the spine erect and counter the problems that may result when the arms hang stiffly at the sides. The statements in the other options indicate correct understanding of measures regarding activity and exercise for the client with Parkinson’s disease.

Test-Taking Strategy: Use the process of elimination and note the strategic words "need for further instruction," which indicate a negative event query and the need to select the incorrect client statement. Think about the characteristics associated with Parkinson’s disease and visualize each of the options. This will direct you to the correct one. Review instructions regarding activity and exercise for the client with Parkinson’s disease if you had difficulty with this question.

References: Black, J., & Hawks, J. (2009). Medical-surgical nursing: Clinical management for positive outcomes (8th ed., pp. 1905, 1906). St. Louis: Saunders.
Ignatavicius, D., & Workman, M. (2010). Medical-surgical nursing: Patient-centered collaborative care (6th ed., p. 968). St. Louis: Saunders.

Cognitive Ability: Evaluating

Client Needs: Physiological Integrity

Integrated Process: Teaching and Learning

Content Area: Adult Health/Neurological Awarded 0.0 points out of 1.0 possible points. 226.ID: 383704589 A nurse is caring for a client who has had a stroke and is experiencing hemianopsia. Which of the following measures does the nurse take in the care of the client? Approaching the client from the side of nonintact vision Teaching the client to move the head from side to side (scan) when eating Correct Placing objects needed for self-care within the client's nonintact visual field Positioning the client in the room so that his nonintact visual field faces the door Rationale: Hemianopsia is the loss of vision in a portion of the visual field. Approaching the client from the side of nonintact vision, placing objects needed for self-care within the client’s nonintact visual field, and positioning the client so that the damaged part of the visual field faces the door are incorrect because the client depends on and needs to use the intact portion of the visual field for sensory input. The client is taught to move the head from side to side (scan) to compensate for a diminished visual field. Scanning is also important when the client is eating.

Test-Taking Strategy: Focus on the client’s diagnosis and recall that hemianopsia is the loss of vision in a portion of the visual field. Note that the incorrect options are comparable or alike in that they involve the damaged portion of the client’s visual field. Review care of the client with hemianopsia if you had difficulty with this question.

Reference: Black, J., & Hawks, J. (2009). Medical-surgical nursing: Clinical management for positive outcomes (8th ed., pp. 1849, 1864, 1865). St. Louis: Saunders.

Cognitive Ability: Applying

Client Needs: Safe and Effective Care Environment

Integrated Process: Nursing Process/Implementation

Content Area: Adult Health/Neurological Awarded 0.0 points out of 1.0 possible points. 227.ID: 383705063 A nurse has provided information about exercise to a client with a diagnosis of degenerative joint disease (osteoarthritis). Which of the following types of exercise does the nurse tell the client to avoid? High-impact exercise Correct Swimming and water exercise Daily range-of-motion exercises Regular exercise with warm-up and cool-down sessions Rationale: Osteoarthritis is a joint disorder marked by joint pain that is worse after exercise or when the affected person puts weight on the joint and is relieved by rest. Vigorous or high-impact exercise may exacerbate symptoms may be damaging to articulating surfaces within joints, and should be avoided by clients with osteoarthritis. The other options are helpful activities/exercises in promoting joint mobility.

Test-Taking Strategy: Use the process of elimination and note the strategic word "avoid." Evaluate each of the options in relation to their helpfulness to the client with impaired joint mobility and joint pain. This will direct you to the correct option. Review care of the client with osteoarthritis if you had difficulty with this question.

Reference: Black, J., & Hawks, J. (2009). Medical-surgical nursing: Clinical management for positive outcomes (8th ed., p. 480). St. Louis: Saunders.

Cognitive Ability: Applying

Client Needs: Physiological Integrity

Integrated Process: Teaching and Learning

Content Area: Adult Health/Musculoskeletal Awarded 0.0 points out of 1.0 possible points. 228.ID: 383713173 A client with schizophrenia says to the nurse, "I decided not to take my medication because it can't help. I am the only one who can help me." Which nursing response is therapeutic in this situation? "Only you can help?" "You decided not to take your medication?" "If you can make that observation, you probably don't need your medication any longer." "Your doctor wants you to continue this medication because it’s helping you. Do you recall needing to be hospitalized because you stopped your medication?" Correct Rationale: Noncompliance with antipsychotic medication is one of the reasons clients with schizophrenia have relapses. The nurse should give a response to the schizophrenic client that will help the client identify the causes of relapse. In asking, “Only you can help?” the nurse is employing restating, which can be therapeutic but is not useful in this client’s situation. "You decided not to take your medication?" is another example of restating. In stating, "If you can make this observation, you probably don't need your medication any longer," the nurse is using an illogical, judgmental, and biased response that is not therapeutic.

Test-Taking Strategy: Focus on the client’s diagnosis to identify the therapeutic response. Note that the correct option is therapeutic and helps the client identify the cause of his relapse. Review care of the client with schizophrenia if you had difficulty with this question.

References: Stuart, G., & Laraia, M. (2009). Principles and practice of psychiatric nursing (9th ed., pp. 27-31). St. Louis: Mosby.
Varcarolis, E., & Halter, M. (2009). Essentials of psychiatric mental health nursing: A communication approach to evidence-based care (p. 293). St. Louis: Saunders.

Cognitive Ability: Applying

Client Needs: Psychosocial Integrity

Integrated Process: Communication and Documentation

Content Area: Mental Health Awarded 0.0 points out of 1.0 possible points. 229.ID: 383704539 A nurse prepares to administer digoxin (Lanoxin) to a client with congestive heart failure. Which vital sign must be checked before the medication is administered? Temperature Respirations Apical pulse Correct Blood pressure Rationale: Digoxin is a cardiac glycoside used to treat congestive heart failure and control the ventricular rate in clients with atrial fibrillation. The nurse would count the client’s apical heart rate for 60 seconds before giving the medication. If the heart rate is slower than 60 beats/min, the medication is withheld and the physician notified. Excessive slowing of the heart rate is one clinical sign of digoxin toxicity. Although the nurse might check other vital signs, the apical pulse is the most important in this context.

Test-Taking Strategy: Use the process of elimination and note the strategic words “must be checked.” Focus on the client’s diagnosis and note the relationship between the diagnosis and the correct option. Review nursing interventions for the administration of digoxin if you had difficulty with this question.

Reference: Kee, J., Hayes, E., & McCuistion, L. (2009). Pharmacology: A nursing process approach (6th ed., p. 627). Philadelphia: Saunders.

Cognitive Ability: Applying

Client Needs: Physiological Integrity

Integrated Process: Nursing Process/Implementation

Content Area: Pharmacology Awarded 0.0 points out of 1.0 possible points. 230.ID: 383706058 A nurse preparing to administer digoxin (Lanoxin) to a client calls the laboratory for the result of the digoxin assay performed on a specimen that was drawn at 6 a.m. The laboratory reports that the result was 2.4 ng/mL. On the basis of this result, the nurse would: Contact the physician Correct Administer the digoxin Wait for the physician to make rounds and report the result Check the client's apical heart rate and administer the digoxin if the rate is faster than 60 beats/min Rationale: The therapeutic serum level for digoxin is 0.5 to 2.0 ng/mL. A digoxin concentration greater than 2.0 ng/mL indicates toxicity, and requires the nurse to contact the physician. The remaining options are inappropriate because they could delay necessary and immediate intervention, resulting in harm to the client.

Test-Taking Strategy: Use the process of elimination. Eliminate the options that are comparable or alike in that they involve administration of the digoxin. To select from the remaining options, recall the therapeutic range of serum values for digoxin, which will direct you to the correct option. Review the therapeutic level of digoxin and the associated nursing actions if you had difficulty with this question.

Reference: Lehne, R. (2010). Pharmacology for nursing care (7th ed., pp. 525, 532). St. Louis: Saunders.

Cognitive Ability: Applying

Client Needs: Physiological Integrity

Integrated Process: Nursing Process/Implementation

Content Area: Pharmacology Awarded 0.0 points out of 1.0 possible points. 231.ID: 383706078 A nurse transcribing the prescriptions of a client admitted to the nursing unit notes that metformin (Glucophage) 850 mg/day has been prescribed. The nurse makes a note in the client's medication record that the medication should be administered: At noon With supper With the morning meal Correct With the midafternoon snack Rationale: Metformin is an antihyperglycemic medication used as an adjunct to diet in the management of type 2 diabetes mellitus. When it is prescribed for daily use, it is administered with the morning meal. Giving the medication at noon, with supper, or with a midafternoon snack is incorrect because the medication is needed to maintain a controlled and consistent blood glucose level. If the prescribed metformin is an extended-release form, the physician may prescribe a daily dose of the medication with the evening meal, because this timing may enhance absorption as a result of slower nighttime gastrointestinal transit time.

Test-Taking Strategy: Focus on the medication’s name, Glucophage. This should indicate to you that the medication is used to treat diabetes mellitus. Recalling the pathophysiology of diabetes mellitus and remembering that medication used to treat diabetes mellitus is most often administered in the morning will direct you to the correct option. Review the timing of administration of metformin if you had difficulty with this question.

Reference: Lehne, R. (2010). Pharmacology for nursing care (7th ed., pp. 675, 687). St. Louis: Saunders.

Cognitive Ability: Applying

Client Needs: Physiological Integrity

Integrated Process: Nursing Process/Implementation

Content Area: Pharmacology Awarded 0.0 points out of 1.0 possible points. 232.ID: 383708580 The wife of a client with diabetes mellitus calls the nurse and reports that her husband's blood glucose level is 60 mg/dL and that her husband is awake but groggy. The nurse tells the client's wife to immediately: Call the physician Administer glucagon hydrochloride (Glucagon) Call an ambulance to bring her husband to the emergency department Place some honey in her husband's mouth, between his gums and cheek Correct Rationale: The client and his wife spouse should be educated about the signs of hypoglycemia (blood glucose level of 60 mg/dL or lower). If a client experiences hypoglycemia and is awake but groggy, corn syrup, honey, or cake icing may be placed in the client’s mouth, between the gums and cheek. Once the sugar has been absorbed through the oral mucosa, the client can usually be aroused sufficiently to take a glass of juice, milk, or sugar-sweetened coffee or tea. There is no reason at this time to call the physician or to call an ambulance to bring the client to the emergency department. Glucagon hydrochloride is used to treat hypoglycemic coma.

Test-Taking Strategy: Use the process of elimination. Eliminate the options that are comparable or alike (calling a physician and taking the client to the emergency department). To select from the remaining options, note the words "awake but groggy" in the question, which will direct you to the correct option. Review the treatment for hypoglycemia if you had difficulty with this question.

Reference: Ignatavicius, D., & Workman, M. (2010). Medical-surgical nursing: Patient-centered collaborative care (6th ed., p. 1508). St. Louis: Saunders.

Cognitive Ability: Applying

Client Needs: Physiological Integrity

Integrated Process: Nursing Process/Implementation

Content Area: Adult Health/Endocrine Awarded 0.0 points out of 1.0 possible points. 233.ID: 383706667 A client with type 1 diabetes mellitus is instructed by the physician to obtain glucagon hydrochloride (Glucagon) for emergency home use. The nurse provides information to the client's wife about the medication. Which statement by the client’s wife indicates that she understands the information? "I need to store this medication in the freezer." "I know that this is used to treat episodes of high blood sugar." "I can give this medication instead of insulin if his insulin runs out." "I need to give this if he has signs of low blood sugar and goes into a coma." Correct Rationale: Glucagon is used to treat hypoglycemia resulting from insulin overdose. If hypoglycemic coma develops, glucagon is administered promptly. The family of the client is instructed in how to administer the medication. In an unconscious client, arousal usually occurs within 20 minutes of injection. Once consciousness has been produced, oral carbohydrates should be given. Insulin should never be stored in the freezer. Glucagon is not used to treat hyperglycemia or used in place of insulin; also, the client should be instructed to have additional vials of insulin on hand at home so that he will not run out. Glucagon is stored at room temperature.

Test-Taking Strategy: Examination of the medication name, glucagon, will assist you in determining that the medication contains some form of glucose. Note the relationship of the name of the medication and the information in the correct option. Review the action of glucagon if you had difficulty with this question.

Reference: Black, J., & Hawks, J. (2009). Medical-surgical nursing: Clinical management for positive outcomes (8th ed., pp. 1095, 1096). St. Louis: Saunders.

Cognitive Ability: Evaluating

Client Needs: Physiological Integrity

Integrated Process: Nursing Process/Evaluation

Content Area: Pharmacology Awarded 0.0 points out of 1.0 possible points. 234.ID: 383703615 A client with hypoparathyroidism is taking calcium gluconate to treat hypocalcemia. The client calls the clinic nurse and complains of becoming constipated since starting the medication. The nurse tells the client to: Stop the medication Contact the physician immediately Increase intake of high-fiber foods Correct Add a half-ounce of mineral oil to the daily diet Rationale: Calcium gluconate is a calcium supplement, and constipation is a side effect of calcium gluconate. It is not necessary for the client to contact the physician immediately if constipation occurs. Rather, the nurse would suggest interventions to alleviate and prevent constipation (e.g., increased fluid intake, high-fiber diet, exercise). Clients taking calcium supplements should be instructed to avoid the use of mineral oil as a laxative because it reduces vitamin D absorption, and vitamin D is needed to assist in the absorption of calcium. The client should not stop taking the medication.

Test-Taking Strategy: Use the process of elimination and focus on the subject, constipation. Recalling the basic principles of alleviating and preventing constipation will direct you to the correct option. Review the measures to alleviate constipation if you had difficulty with this question.

Reference: Lehne, R. (2010). Pharmacology for nursing care (7th ed., pp. 884, 885). St. Louis: Saunders.

Cognitive Ability: Applying

Client Needs: Physiological Integrity

Integrated Process: Teaching and Learning

Content Area: Pharmacology Awarded 0.0 points out of 1.0 possible points. 235.ID: 383704523 A client has a physician's appointment to get a prescription for sildenafil (Viagra). The nurse obtains the health history from the client. Which finding indicates that the medication is contraindicated? The client has type 2 diabetes mellitus. The client has a history of renal calculi. The client is taking glargine (Lantus) insulin. The client takes isosorbide dinitrate (Isordil). Correct Rationale: Sildenafil is used to treat male erectile dysfunction. It is contraindicated in the client who is using sodium nitroprusside or organic nitrates in any form because both sildenafil and nitrates promote hypotension. When these medications are combined, life-threatening hypotension may result. Type 2 diabetes mellitus, a history of renal calculi, and the use of glargine (Lantus) insulin are not contraindications to the use of sildenafil.

Test-Taking Strategy: Use the process of elimination and recall that sildenafil is contraindicated with the use of nitrates. Next read each option carefully and note the name of the medication, isosorbide dinitrate. Recalling that this medication is a nitrate should direct you to the correct option. Review contraindications to the use of sildenafil if you had difficulty with this question.

Reference: Hodgson, B., & Kizior, R. (2010). Saunders nursing drug handbook 2010 (p. 1034). St. Louis: Saunders.

Cognitive Ability: Analyzing

Client Needs: Physiological Integrity

Integrated Process: Nursing Process/Assessment

Content Area: Pharmacology Awarded 0.0 points out of 1.0 possible points. 236.ID: 383708586 Vasopressin (Pitressin) is prescribed to a client with diabetes insipidus. For which sign, indicative of an adverse effect of the medication, does the nurse monitor the client? Chest pain Correct Constipation Loss of appetite Decreased urine output Rationale: Because of its powerful vasoconstrictive action, vasopressin can cause adverse cardiovascular effects. By constricting arteries of the heart, vasopressin can cause angina pectoris and even myocardial infarction, especially if the medication is administered to a client with existing coronary artery disease. Vasopressin may also cause gangrene by decreasing blood flow in the periphery. Chest pain is a sign of an adverse effect, and if it occurs the physician is notified. Decreased urine output is an expected effect. Neither constipation nor loss of appetite is associated with this medication.

Test-Taking Strategy: Use the process of elimination. Recalling the pathophysiology of diabetes insipidus and the associated treatment will assist you in eliminating the option of decreased urine output, because this is an expected outcome. To select from the remaining options, note the relationship between the name of the medication, vasopressin, and chest pain in the correct option. Review the cautions associated with the administration of vasopressin if you had difficulty with this question.

Reference: Lehne, R. (2010). Pharmacology for nursing care (7th ed., p. 709). St. Louis: Saunders.

Cognitive Ability: Analyzing

Client Needs: Physiological Integrity

Integrated Process: Nursing Process/Assessment

Content Area: Pharmacology Awarded 0.0 points out of 1.0 possible points. 237.ID: 383703605 Desmopressin (DDAVP) is prescribed to a client with diabetes insipidus. Which parameter does the nurse tell the client that it is important to monitor while she is taking the medication? Appetite Pulse rate Bowel pattern Intake and output Correct Rationale: Diabetes insipidus is a disorder of water metabolism caused by a deficiency of antidiuretic hormone. Desmopressin promotes renal conservation of water. The hormone accomplishes this by acting on the collecting ducts of the kidney to increase their permeability to water, resulting in increased water reabsorption. The client is instructed to monitor and record daily intake and output of fluid. If the dose prescribed is adequate, urine volume should rapidly drop to normal. Appetite, pulse rate, and changes in bowel pattern are not associated with the use of this medication.

Test-Taking Strategy: Recalling the pathophysiology and clinical manifestations of diabetes insipidus will help you select the correct option. Review diabetes insipidus and the action of desmopressin if you had difficulty answering this question.

Reference: Skidmore-Roth, L. (2010). Mosby’s nursing drug reference (23rd ed., p. 359). St. Louis: Mosby.

Cognitive Ability: Applying

Client Needs: Physiological Integrity

Integrated Process: Teaching and Learning

Content Area: Pharmacology Awarded 0.0 points out of 1.0 possible points. 238.ID: 383706682 Warfarin sodium (Coumadin) is prescribed for a hospitalized client. While transcribing the physician's prescription, the nurse notes that the client is taking levothyroxine (Synthroid) to treat hypothyroidism. The nurse calls the physician to confirm the prescription for warfarin sodium because: Warfarin sodium amplifies the effect of levothyroxine Levothyroxine amplifies the effect of warfarin sodium Correct Warfarin sodium is contraindicated with the use of levothyroxine A severe allergic reaction may occur if warfarin sodium is administered concurrently with levothyroxine Rationale: Levothyroxine accelerates the degradation of vitamin K–dependent clotting factors. As a result, the effects of warfarin sodium are enhanced. If warfarin sodium administration is instituted in a client who takes levothyroxine, the dose of warfarin sodium should be reduced. Warfarin sodium is not contraindicated in the client who is using levothyroxine. Concurrent administration does not cause an allergic reaction.

Test-Taking Strategy: Knowledge regarding the medication interactions that may occur with levothyroxine is required to answer the question. Remember that levothyroxine accelerates the degradation of vitamin K–dependent clotting factors. This will direct you to the correct option. Review considerations in the administration of levothyroxine and warfarin sodium if you had difficulty with this question.

Reference: Lehne, R. (2010). Pharmacology for nursing care (7th ed., p. 694). St. Louis: Saunders.

Cognitive Ability: Understanding

Client Needs: Safe and Effective Care Environment

Integrated Process: Nursing Process/Implementation

Content Area: Pharmacology Awarded 0.0 points out of 1.0 possible points. 239.ID: 383706062 Iodine solution (Lugol solution) is prescribed to a client who is scheduled for subtotal thyroidectomy. The client calls the nurse at the clinic and complains of a burning sensation in the mouth and soreness of the gums and teeth. The nurse most appropriately tells the client: To contact the physician Correct That these are expected side effects of the medication That these discomforts will resolve with continued therapy To stop the medication for the next 24 hours and then continue as prescribed Rationale: Iodine can produce toxicity (iodism). The client should be instructed about the symptoms of iodism, which includes a brassy taste and burning sensation in the mouth, soreness of the gums and teeth, frontal headache, coryza, salivation, and skin eruptions. The client should be instructed to notify the physician if any of these symptoms occurs. The client’s symptoms are not expected side effects of the medication and will not resolve with continued therapy; in fact, continued therapy will worsen the client’s condition. Stopping the medication for 24 hours and then continuing it as prescribed will also worsen the client’s condition.

Test-Taking Strategy: Use the process of elimination. Eliminate the options that are comparable or alike in that they indicate that the client’s symptoms are expected or will resolve. Remembering that the nurse may not legally alter a medication prescription without a physician’s prescription to do so will assist you in answering correctly. Review the adverse effects of iodine solution and the signs of iodism if you had difficulty with this question.

Reference: Lehne, R. (2010). Pharmacology for nursing care (7th ed., pp. 697, 700). St. Louis: Saunders.

Cognitive Ability: Applying

Client Needs: Physiological Integrity

Integrated Process: Nursing Process/Implementation

Content Area: Pharmacology Awarded 0.0 points out of 1.0 possible points. 240.ID: 383712472 Levothyroxine (Synthroid) is prescribed to a client with hypothyroidism. One week after beginning the medication, the client calls the physician's office and tells the nurse that the medication has not helped. The nurse most appropriately tells the client that: The medication will need to be changed The full therapeutic effect may take 4 weeks Correct An additional medication will have to be added to the regimen The blood levels of the client’s thyroid hormones will need to be rechecked Rationale: Levothyroxine is used in the treatment of hypothyroidism. Although therapy with levothyroxine may begin with small doses that are gradually increased, the most appropriate response is to inform the client that full therapeutic effect may take 1 month. For this reason, changing the medication and the addition of a medication are not necessary. Although the blood levels of thyroid hormones are checked during therapy with levothyroxine, obtaining these levels after 1 week of therapy will not provide useful information.

Test-Taking Strategy: Use the process of elimination. Eliminate the options that are comparable or alike (the medication needs to be changed or an additional medication is needed). To select from the remaining options, note the words “1 week after beginning the medication” in the question and recall that about 1 month is required for the plasma level of levothyroxine to plateau. Review information on levothyroxine if you had difficulty with this question.

Reference: Lehne, R. (2010). Pharmacology for nursing care (7th ed., p. 699). St. Louis: Saunders.

Cognitive Ability: Applying

Client Needs: Physiological Integrity

Integrated Process: Teaching and Learning

Content Area: Pharmacology Awarded 0.0 points out of 1.0 possible points. 241.ID: 383713183 A nurse provides instructions to a client who will be taking levothyroxine (Synthroid) for hypothyroidism. The nurse tells the client that it is best to take the medication: With milk At bedtime With an antacid In the morning, before breakfast Correct Rationale: Levothyroxine, a synthetic preparation of thyroxine, a naturally occurring thyroid hormone, is used to treat clients who require hormone-replacement therapy. The client is instructed to take the medication on an empty stomach, preferably in the morning, before breakfast.

Test-Taking Strategy: Use the process of elimination. Eliminate the options that are comparable or alike (milk, antacid). To select from the remaining options, note the strategic word "best" in the query and recall the action and purpose of levothyroxine, which will direct you to the correct option. Review client teaching points for levothyroxine if you had difficulty with this question.

Reference: Lehne, R. (2010). Pharmacology for nursing care (7th ed., p. 698). St. Louis: Saunders.

Cognitive Ability: Applying

Client Needs: Physiological Integrity

Integrated Process: Teaching and Learning

Content Area: Pharmacology Awarded 0.0 points out of 1.0 possible points. 242.ID: 383706615 A client is brought to the emergency department by ambulance, and diabetic ketoacidosis is suspected. Blood samples are taken, and the nurse obtains supplies that will be needed to treat the client. Which type of insulin does the nurse take from the medication supply room for intravenous (IV) administration? NPH (Humulin N) Lente (Humulin L) Regular (Humulin R) Correct NPH/regular 50%/50% (Humulin 50/50) Rationale: Ketoacidosis, the most severe manifestation of insulin deficiency, is a life-threatening emergency. IV insulin is a primary component of treatment. Regular insulin, which has a rapid onset and short duration of action, is the only insulin that may be given IV. Because regular insulin forms a true solution, it is safe for IV use. NPH, Lente, NPH/regular 50%/50% are intermediate-acting insulins.

Test-Taking Strategy: Use the process of elimination. Eliminate the options that are comparable or alike in that they represent intermediate-acting insulin. Also remember that regular insulin is the only type of insulin that may be administered IV. Review the treatment of diabetic ketoacidosis if you had difficulty with this question.

Reference: Black, J., & Hawks, J. (2009). Medical-surgical nursing: Clinical management for positive outcomes (8th ed., p. 1092). St. Louis: Saunders.

Cognitive Ability: Applying

Client Needs: Physiological Integrity

Integrated Process: Nursing Process/Implementation

Content Area: Adult Health/Endocrine Awarded 0.0 points out of 1.0 possible points. 243.ID: 383711491 A home care nurse prefills syringes containing NPH (Humulin N) and regular (Humulin R) insulin for a client with diabetes mellitus who will be administering his own insulin but has difficulty seeing and accurately preparing doses. The nurse places the medication in the client's refrigerator with the syringes: Lying flat In a horizontal position In a vertical position with the needles pointing up Correct In a vertical position with the needles pointing down Rationale: Mixtures of insulin in prefilled syringes may be stored in a refrigerator, where they will be stable for at least 1 to 2 weeks. The syringes should be stored vertically, with the needles pointing up to prevent clogging of the needle with the insulin. Before administration of the medication, the syringe should be agitated gently to resuspend the insulin.

Test-Taking Strategy: Use the process of elimination. Eliminate the options that are comparable or alike (lying flat, horizontal). Next visualize each of the remaining options and think about the effect of having the needles point up or down. This will help direct you to the correct option. Review the principles of prefilling insulin syringes if you had difficulty with this question.

Reference: Lehne, R. (2010). Pharmacology for nursing care (7th ed., p. 670). St. Louis: Saunders.

Cognitive Ability: Applying

Client Needs: Physiological Integrity

Integrated Process: Nursing Process/Implementation

Content Area: Pharmacology Awarded 0.0 points out of 1.0 possible points. 244.ID: 383713106 Tolbutamide (Orinase) is prescribed to a client whose type 2 diabetes mellitus has not been controlled with diet and exercise alone. The nurse provides instructions to the client about the medication. Which statement by the client indicates a need for further instruction? "I need to maintain my exercise program." "I need to stay on my calorie-restricted diet." "I can take the medication with food if it upsets my stomach." "I can have a beer or glass of wine as long as I stay within my daily dietary restrictions." Correct Rationale: Tolbutamide, a first-generation sulfonylurea, is used as an adjunct to calorie restriction and exercise to maintain glycemic control in clients with type 2 diabetes mellitus. When alcohol is combined with tolbutamide, a disulfiram-like reaction, including flushing, palpitations, and nausea, may occur. Alcohol also potentiates the hypoglycemic effects of tolbutamide. The client must be told about the dangers of alcohol consumption. The statements in the remaining options are correct.

Test-Taking Strategy: Note the strategic words "needs for further instruction," which indicate a negative event query and the need to select the incorrect client statement. Use general principles of medication administration to direct you to the correct option. Remember, it is not acceptable for a client to consume alcohol when taking medication unless permission to do so has been obtained from the health care provider. Review client teaching points for tolbutamide if you had difficulty with this question.

Reference: Lehne, R. (2010). Pharmacology for nursing care (7th ed., p. 687). St. Louis: Saunders.

Cognitive Ability: Evaluating

Client Needs: Physiological Integrity

Integrated Process: Teaching and Learning

Content Area: Adult Health/Endocrine Awarded 0.0 points out of 1.0 possible points. 245.ID: 383703609 A nurse is transcribing a physician's prescription for oral prednisone 5 mg/day that was written in the chart of a client with type 2 diabetes mellitus who is already taking an oral hypoglycemic medication. The nurse contacts the physician to ask about the prescription because: Prednisone can lower the blood glucose level Prednisone can increase the blood glucose level Correct Prednisone is contraindicated with the use of oral hypoglycemic medications For prednisone to be effective in a client taking an oral hypoglycemic agent, a higher dosage of prednisone is required Rationale: Prednisone is a glucocorticoid. Glucocorticoids can increase the blood glucose level. Prednisone is not contraindicated in the client taking an oral hypoglycemic agent; however, diabetic clients may require increased dosages of insulin or oral hypoglycemic medications during glucocorticoid therapy.

Test-Taking Strategy: Focus on the data in the question and note that prednisone has been prescribed for the client. Recalling that glucocorticoids can increase the blood glucose level will help you answer the question. Review the side effects of prednisone if you had difficulty with this question

Reference: Lehne, R. (2010). Pharmacology for nursing care (7th ed., p. 855). St. Louis: Saunders.

Cognitive Ability: Understanding

Client Needs: Safe and Effective Care Environment

Integrated Process: Nursing Process/Implementation

Content Area: Pharmacology Awarded 0.0 points out of 1.0 possible points. 246.ID: 383713104 Oral prednisone 10 mg/day is prescribed for a client with an acute exacerbation of rheumatoid arthritis. The nurse, providing information to the client about the medication, tells the client that it is best to take it: At bedtime With lunch In the evening, after 9 p.m. In the morning, before 9:00 a.m. Correct Rationale: Prednisone, a glucocorticoid, may be prescribed to reduce inflammation and pain in acute exacerbations of rheumatoid arthritis. Glucocorticoids should be administered before 9 a.m. This schedule helps minimize adrenal insufficiency and mimics the burst of glucocorticoids released naturally by the adrenals each morning. Therefore the time frames in the remaining options are incorrect.

Test-Taking Strategy: Use the process of elimination. Eliminate the options that are comparable or alike (bedtime and in the evening after 9 p.m.). From the remaining options, note the strategic word "best" in the query of the question and recall the pathophysiology of adrenal function. Review client teaching points for prednisone if you had difficulty with this question.

Reference: Lehne, R. (2010). Pharmacology for nursing care (7th ed., p. 854). St. Louis: Saunders.

Cognitive Ability: Applying

Client Needs: Physiological Integrity

Integrated Process: Teaching and Learning

Content Area: Pharmacology Awarded 0.0 points out of 1.0 possible points. 247.ID: 383709204 A child with growth hormone deficiency will be receiving somatropin (Humatrope). The nurse provides information to the mother about the medication. Which of the following laboratory values does the nurse tell the mother will require monitoring? Creatinine Hemoglobin Blood urea nitrogen (BUN) Thyroid-stimulating hormone (TSH) Correct Rationale: Somatropin is a growth hormone. One adverse reaction to somatropin is hypothyroidism. Thyroid function is assessed before treatment and periodically thereafter. Creatinine and BUN are used to evaluate renal function, and hemoglobin reflects hematologic activity.

Test-Taking Strategy: Use the process of elimination. Eliminate the options that are comparable or alike in that they are indicators of renal function. To select from the remaining options, focus on the child’s diagnosis and note the relationship between the diagnosis and the correct option. Review the adverse effects of somatropin if you had difficulty with this question.

Reference: Lehne, R. (2010). Pharmacology for nursing care (7th ed., p. 708). St. Louis: Saunders.

Cognitive Ability: Applying

Client Needs: Physiological Integrity

Integrated Process: Teaching and Learning

Content Area: Child Health/Endocrine Awarded 0.0 points out of 1.0 possible points. 248.ID: 383708553 Laboratory studies are performed on a client with suspected sickle cell disease, and electrophoresis reveals a large percentage of hemoglobin S (HbS). Which additional laboratory finding will the nurse expect to note that is a characteristic of this disease? Low reticulocyte count Low total bilirubin level Increased hematocrit count Increased white blood cell (WBC) count Correct Rationale: Sickle cell disease is a genetic disorder that results in chronic anemia, pain, disability, organ damage, increased risk for infection, and early death. In this disorder the red blood cells assume a sickle shape, become rigid, and clump together. The main laboratory finding associated with sickle cell disease is the large percentage of HbS present on electrophoresis. The reticulocyte count is increased, indicating anemia of long duration, and the total bilirubin level is increased. The hematocrit level is low and decreases during crisis, because the bone marrow fails to produce cells during stressful periods. The WBC count is usually higher than normal in clients with sickle cell disease. It is believed that this increase is related to chronic inflammation resulting from tissue hypoxia and ischemia.

Test-Taking Strategy: Specific knowledge regarding the laboratory findings associated with sickle cell disease is required to answer this question. Focusing on the pathophysiology of this disease will assist you in answering this question. Remember that the WBC count is usually higher than normal in clients with sickle cell disease. Review these laboratory findings if you had difficulty with this question.

Reference: Ignatavicius, D., & Workman, M. (2010). Medical-surgical nursing: Patient-centered collaborative care (6th ed., pp. 894, 895). St. Louis: Saunders.

Cognitive Ability: Analyzing

Client Needs: Physiological Integrity

Integrated Process: Nursing Process/Assessment

Content Area: Adult Health/Cardiovascular Awarded 0.0 points out of 1.0 possible points. 249.ID: 383703647 A client being seen in the clinic complains of fatigue and weakness. Laboratory studies are performed because the physician suspects iron-deficiency anemia. Which finding indicative of this type of anemia does the nurse expect to find on reviewing the laboratory results? An increased RBC count An increased hematocrit level An increased hemoglobin level Microcytic red blood cells (RBCs) Correct Rationale: In iron-deficiency anemia, laboratory testing will reveal low hemoglobin and hematocrit levels. In iron-deficiency anemia, iron stores are depleted first, followed by hemoglobin stores. As a result, the RBC count is low and the RBCs are microcytic (small).

Test-Taking Strategy: Use the process of elimination. Focusing on the diagnosis, iron-deficiency anemia, will assist you in eliminating the options that indicate an increased level. Review the characteristics of iron-deficiency anemia if you had difficulty with this question.

Reference: Ignatavicius, D., & Workman, M. (2010). Medical-surgical nursing: Patient-centered collaborative care (6th ed., p. 893). St. Louis: Saunders.

Cognitive Ability: Analyzing

Client Needs: Physiological Integrity

Integrated Process: Nursing Process/Assessment

Content Area: Adult Health/Cardiovascular Awarded 0.0 points out of 1.0 possible points. 250.ID: 383708563 Alendronate (Fosamax) is prescribed for a client with postmenopausal osteoporosis. The nurse provides information on the medication to the client. When does the nurse tell the client to take the alendronate? At bedtime With orange juice, to enhance absorption at night Every morning before breakfast, with a full glass of water Correct Every morning after breakfast, after which the client should lie down for 30 minutes Rationale: Alendronate is a medication used to treat postmenopausal osteoporesis, glucocorticoid-induced osteoporosis, and Paget’s disease of bone. Proper administration is necessary to maximize bioavailability and minimize the risk of esophagitis. The medication should be taken in the morning before breakfast on an empty stomach to maximize its bioavailability. No food, including orange juice or coffee, should be consumed for at least 30 minutes after alendronate is taken. To minimize the risk of esophagitis, the client should take the medication with a full glass of water and remain upright (seated or standing) for at least 30 minutes. Therefore taking the medication at bedtime, with orange juice to help with absorption, and every morning after breakfast, followed by a 30-minute period of lying down, are all incorrect.

Test-Taking Strategy: Use the process of elimination, focusing on the name of the medication. Recalling the adverse effects of the medication and remembering that the medication can cause esophagitis will direct you to the correct option. Review client teaching points for alendronate if you had difficulty with this question.

Reference: Lehne, R. (2010). Pharmacology for nursing care (7th ed., p. 886). St. Louis: Saunders.

Cognitive Ability: Applying

Client Needs: Physiological Integrity

Integrated Process: Teaching and Learning

Content Area: Pharmacology Awarded 0.0 points out of 1.0 possible points. 251.ID: 383703629 Calcium carbonate (Os-Cal 500) is prescribed for a client with mild hypocalcemia. What food does the nurse instruct the client to avoid consuming while taking this medication? Fish Milk Spinach Correct Watermelon Rationale: Oral calcium salts are used to treat mild hypocalcemia and to supplement dietary calcium. The client is instructed to take oral calcium with a large glass of water with or after a meal to promote absorption. The client is also instructed to avoid taking calcium with foods that can suppress calcium absorption. Such foods include spinach, Swiss chard, beets, bran, and whole-wheat cereals. The client does not need to avoid fish, milk, or watermelon.

Test-Taking Strategy: Specific knowledge regarding the foods that suppress calcium absorption is needed to answer this question. Remember that spinach is one of the foods that can suppress calcium absorption. Review client teaching points for calcium carbonate if you had difficulty with this question.

References: Black, J., & Hawks, J. (2009). Medical-surgical nursing: Clinical management for positive outcomes (8th ed., p. 160). St. Louis: Saunders.
Lehne, R. (2010). Pharmacology for nursing care (7th ed., p. 885). St. Louis: Saunders.

Cognitive Ability: Applying

Client Needs: Physiological Integrity

Integrated Process: Teaching and Learning

Content Area: Pharmacology Awarded 0.0 points out of 1.0 possible points. 252.ID: 383705078 An emergency department nurse is caring for a client in hypovolemic shock, a result of external hemorrhage caused by a gunshot wound. Which nursing interventions should the nurse take? Select all that apply. Maintaining the client in a high Fowler’s position Checking the client's vital signs every hour until stable Ensuring that direct pressure is applied to the external hemorrhage site Correct Ensuring a patent airway and supplying oxygen to the client as prescribed Correct Inserting an intravenous (IV) catheter and administering fluids as prescribed Correct Ensuring that the call bell is in place for the client's use when the nurse is out of the room Rationale: When caring for a client in hypovolemic shock, the nurse must first ensure a patent airway and supply oxygen to the client. The nurse would insert an IV catheter if one is not already present and administer fluids as prescribed. The nurse would elevate the client’s feet, keeping his or her head flat or elevated to a 30-degree angle. Direct pressure is applied to the site of external bleeding. The nurse would take the client’s vital signs every 5 minutes until they were stable. The nurse would not leave the client alone.

Test-Taking Strategy: Focus on the client’s diagnosis and type of injury. Use the ABCs (airway, breathing, and circulation). Thinking about the pathophysiology of hypovolemic shock and noting that the situation described is a life-threatening one will direct you to the correct options. Review care of the client in hypovolemic shock if you had difficulty with this question.

Reference: Ignatavicius, D., & Workman, M. (2010). Medical-surgical nursing: Patient-centered collaborative care (6th ed., p. 836). St. Louis: Saunders.

Cognitive Ability: Applying

Client Needs: Physiological Integrity

Integrated Process: Nursing Process/Implementation

Content Area: Critical Care Awarded 0.0 points out of 1.0 possible points. 253.ID: 383706658 A child with a diagnosis of Wilms’ tumor is being admitted to the pediatric unit. The nurse prepares the room for the child and places a sign at the child's bedside that tells staff to avoid: Palpating the abdomen Correct Taking temperatures rectally Turning the child to the right side Measuring blood pressure in the right arm Rationale: Wilms’ tumor, or nephroblastoma, is the most common renal tumor in children. Arising from the renal parenchyma of the kidney, this tumor grows very rapidly. It may be unilateral and localized or bilateral and sometimes involves metastasis to other organs. The tumor mass should not be palpated because of the risk that the protective capsule will rupture. Excessive manipulation can result in seeding of the tumor and the spread of cancerous cells. The nurse would place a sign on the bed warning against palpation of the child’s abdomen. Taking temperatures rectally, turning the child to the right side, and measuring blood pressure in the right arm are interventions that do not need to be avoided.

Test-Taking Strategy: Focus on the child’s diagnosis and its pathophysiology. Recalling the location of Wilms' tumor and remembering that it can metastasize will direct you to the correct option. Review care of the child with Wilms' tumor if you had difficulty with this question.

Reference: McKinney, E., James, S., Murray, S., & Ashwill, J. (2009). Maternal-child nursing (3rd ed., p. 1333). St. Louis: Elsevier.

Cognitive Ability: Applying

Client Needs: Physiological Integrity

Integrated Process: Nursing Process/Planning

Content Area: Child Health/Renal Awarded 0.0 points out of 1.0 possible points. 254.ID: 383713102 A client with multiple sclerosis has been started on baclofen (Lioresal) for muscle spasms. The client calls the physician's office 1 week after beginning the medication and tells the nurse that she feels extremely drowsy. The nurse most appropriately tells the client: That she will need to be seen by the physician That the medication will need to be discontinued That drowsiness usually diminishes with continued therapy Correct To stop the medication for 2 days and then resume it at the prescribed dosage Rationale: Baclofen (Lioresal) is a centrally acting skeletal muscle relaxant. Side effects of baclofen include drowsiness, dizziness, weakness, and nausea. Drowsiness usually diminishes with continued therapy. However, the client is told to avoid activities that require alertness until the response to the medication has been established. Therefore it is not necessary to call the physician or discontinue or stop the medication.

Test-Taking Strategy: Use the process of elimination. Eliminate the options that are comparable or alike in that they suggest discontinuing or stopping the medication. To select from the remaining options, recall that baclofen is a centrally acting skeletal muscle relaxant and that drowsiness is a side effect. Review the effects of baclofen if you had difficulty with this question.

Reference: Lehne, R. (2010). Pharmacology for nursing care (7th ed., p. 241). St. Louis: Saunders.

Cognitive Ability: Applying

Client Needs: Physiological Integrity

Integrated Process: Nursing Process/Implementation

Content Area: Adult Health/Musculoskeletal Awarded 0.0 points out of 1.0 possible points. 255.ID: 383703673 Alprazolam (Xanax) is prescribed for a client to treat an anxiety disorder. Which side effect does the nurse warn the client of? Headache Urine retention Lightheadedness Correct Increased salivation Rationale: Alprazolam is a benzodiazepine that is used to manage anxiety disorders. Common side effects include muscle incoordination, lightheadedness, transient mild drowsiness, and slurred speech. The other options are not side effects of this medication.

Test-Taking Strategy: Focus on the client’s diagnosis and the subject, a side effect of the medication. Recalling that alprazolam is a benzodiazepine will assist you in answering the question. Review the side effects of alprazolam if you had difficulty with this question.

Reference: Lehne, R. (2010). Pharmacology for nursing care (7th ed., pp. 381, 382). St. Louis: Saunders.

Cognitive Ability: Applying

Client Needs: Physiological Integrity

Integrated Process: Teaching and Learning

Content Area: Pharmacology Awarded 0.0 points out of 1.0 possible points. 256.ID: 383710530 View video. The nurse is preparing to administer a tube feeding by way of a nasogastric tube. Which action does the nurse carry out as a priority before starting the flow of the solution? Flushing the tube with 30 mL of tap water Checking urine output in the previous 24 hours Scrubbing the port with povidone-iodine (Betadine) solution Checking for gastric residual volume and assessing tube placement Correct Rationale: View video. When a client has a nasogastric tube, a priority concern is the risk for aspiration. The nurse would not administer a feeding or flush the tube unless residual volume and tube placement had been checked. To check residual volume, the nurse would draw up 30 mL of air in a catheter-tipped syringe, attach it to the feeding tube, inject the air, and then pull back the plunger to aspirate all of the gastric contents. The gastric contents are checked for amount and appearance and then the pH is checked for acidity to ensure accurate placement. It is not necessary to scrub the port with povidone-iodine solution; administering a tube feeding is a clean, not sterile procedure. Although intake and output may be monitored, it is not necessary to check urine output for the previous 24 hours before starting a feeding.

Test-Taking Strategy: Note the strategic word “priority.” Use the ABCs — airway, breathing, and circulation — to identify the correct option. Checking residual and correct placement will decrease the risk for aspiration. Review the procedure for administering a tube feeding if you had difficulty with this question.

Reference: Potter, P., & Perry, A. (2009). Fundamentals of nursing (7th ed., pp. 1118-1122). St. Louis: Mosby.

Cognitive Ability: Applying

Client Needs: Safe and Effective Care Environment

Integrated Process: Nursing Process/Implementation

Content Area: Nutrition Awarded 0.0 points out of 1.0 possible points. 257.ID: 383703655 A nurse is caring for a client who has had a cast applied to the left leg and is at risk for acute compartment syndrome. For which early sign of this complication does the nurse monitor the client? Paresthesia Correct Cold, bluish toes Weak pedal pulse Severe pain relieved by medication Rationale: Acute compartment syndrome is a serious condition in which increased pressure within one or more compartments causes massive compromise of circulation to the area. Resultant edema causes pressure on the nerve endings and subsequent pain. Sensory deficits such as paresthesia generally appear before changes in vascular or motor signs. The client will also complain of severe diffuse pain that is not relieved by analgesics.

Test-Taking Strategy: Note the strategic word "early." Eliminate the options that are comparable or alike in that they represent vascular changes, which occur after sensory deficits appear. To select from the remaining options, note the option that indicates that the client obtained relief of the pain from medication. The severe pain of compartment syndrome is not relieved by analgesics. Review the early signs of compartment syndrome if you had difficulty with this question.

References: Black, J., & Hawks, J. (2009). Medical-surgical nursing: Clinical management for positive outcomes (8th ed., p. 522). St. Louis: Saunders.
Ignatavicius, D., & Workman, M. (2010). Medical-surgical nursing: Patient-centered collaborative care (6th ed., p. 1181). St. Louis: Saunders.

Cognitive Ability: Analyzing

Client Needs: Physiological Integrity

Integrated Process: Nursing Process/Assessment

Content Area: Adult Health/Musculoskeletal Awarded 0.0 points out of 1.0 possible points. 258.ID: 383710048 A physician writes a prescription for 1000 mL of 0.9% normal saline solution to be administered intravenously (IV) to a client over 10 hours. The drop factor for the infusion set is 15 gtt/mL. At what drip rate does the nurse set the infusion?
Type answer in the box provided.

_______________ gtt/min Incorrect Correct Responses: "25" Awarded 0.0 out of 1.0 possible points. 259.ID: 383704549 A nurse is performing an assessment of a client being admitted to the hospital with a diagnosis of multiple sclerosis. The client tells the nurse that she took baclofen (Lioresal) for the past 9 months but completely stopped the medication 2 days ago because it was making her feel weak. On the basis of this information, the nurse notes in the plan of care that the client should be monitored most closely for: Spasticity Drowsiness Muscle spasms Seizure activity Correct Rationale: Baclofen is a skeletal muscle relaxant, acting at the level of the spinal cord to reduce the frequency and amplitude of muscle spasms and spasticity in clients with spinal cord injuries and diseases, as well as multiple sclerosis. Abrupt withdrawal of the medication may produce hallucinations or seizures. Drowsiness may occur as a result of taking the medication but diminishes with continued therapy.

Test-Taking Strategy: Use the process of elimination. Eliminate the options that are comparable or alike (i.e., spasticity, muscle spasms). To select from the remaining options, recall that the client has stopped taking the medication and note the strategic words "most closely." This will direct you to the correct option. Review baclofen if you had difficulty with this question.

Reference: Lehne, R. (2010). Pharmacology for nursing care (7th ed., p. 244). St. Louis: Saunders.

Cognitive Ability: Analyzing

Client Needs: Physiological Integrity

Integrated Process: Nursing Process/Assessment

Content Area: Pharmacology Awarded 0.0 points out of 1.0 possible points. 260.ID: 383707951 A nurse is caring for a client who is immobilized in skeletal traction after sustaining a leg fracture in a motor vehicle crash. The nurse notes that the client is restless, and the client complains of being bored. Which problem does the nurse identify on the basis of this information? Lack of control Lack of physical mobility Lack of adequate diversional activity Correct Lack of energy to bathe and feed self Rationale: A characteristic of lack of adequate diversional activity is the expression of boredom by the client. The question does not identify client difficulties with coordination, range of motion, or muscle strength, which would lack of physical mobility. Nor does the question address client’s lack of energy to perform activities of daily living (bathing/hygiene self-care deficit) or lack of control.

Test-Taking Strategy: Use the process of elimination and focus on the information in the question. Noting the strategic word "bored" will direct you to the correct option. Review the defining characteristic for deficient diversional activity if you had difficulty with this question.

References: Ignatavicius, D., & Workman, M. (2010). Medical-surgical nursing: Patient-centered collaborative care (6th ed., p. 1185). St. Louis: Saunders.
Swearingen, P. (2008). All-in-one care planning resource: Medical-surgical, pediatric, maternity & psychiatric nursing care plans (2nd ed., pp. 517, 518). St. Louis: Mosby.

Cognitive Ability: Analyzing

Client Needs: Psychosocial Integrity

Integrated Process: Nursing Process/Analysis

Content Area: Adult Health/Musculoskeletal Awarded 0.0 points out of 1.0 possible points. 261.ID: 383708590 A client is found to have posttraumatic stress disorder (PTSD) after witnessing a terrorist attack that caused the deaths of hundreds of people. The nurse, developing a plan of care for the client, identifies posttrauma syndrome as a concern and identifies a client outcome that states, "The client will cope effectively with thoughts and feelings of the event." Which nursing interventions will assist the client in achieving this outcome? Select all that apply. Being honest, nonjudgmental, and empathetic Correct Assessing the immediate posttraumatic reaction Correct Encouraging the client to keep a journal focused on the trauma Correct Asking the client about the use of alcohol and drugs before and since the event Correct Promoting discussion of the reasons the client was responsible for the traumatic event Discouraging the use of support groups until the client is able to use effective coping techniques Rationale: An honest, nonjudgmental, and empathetic attitude helps the nurse build a trusting relationship with the client. The nurse would assess the immediate posttraumatic reaction and later coping. Numbing and denial are common reactions after a traumatic event, and knowing the range of the client’s behavior can help the nurse assess the impact and meaning of the trauma. Writing about the trauma in a journal can lessen the intensity of the client’s emotions and his or her preoccupation with the event over time.The nurse would ask the client about the use of alcohol and drugs before and since the event. It is important for the nurse to obtain this information, because attempts to self-medicate to reduce anxiety and induce sleep are common after a traumatic event. The client needs to understand that he or she is not responsible for the event, but the nurse should emphasize that the client is responsible for learning to cope. This strategy will assist the client in easing feelings of powerlessness. The nurse would encourage attendance at support groups so that the client can share experiences, feel understood, and begin to heal.

Test-Taking Strategy: Focus on the client’s diagnosis, posttraumatic stress disorder, and the traumatic event. Read each intervention carefully, recalling that support groups are an effective way for the client to share experiences and that the client should not feel responsible for the traumatic event. Review interventions for the client with posttraumatic stress disorder if you had difficulty with this question.

Reference: Varcarolis, E., & Halter, M. (2009). Essentials of psychiatric mental health nursing: A communication approach to evidence-based care (p. 418). St. Louis: Saunders.

Cognitive Ability: Applying

Client Needs: Psychosocial Integrity

Integrated Process: Nursing Process/Implementation

Content Area: Mental Health Awarded 0.0 points out of 1.0 possible points. 262.ID: 383703681 A nurse reviews the results of a total serum calcium determination in a client with renal failure. The results indicate a level of 12.0 mg/dL. In light of this result, which finding does the nurse expect to note during assessment? Decreased urine output Hyperactive bowel sounds Bounding, full peripheral pulses Correct Hyperactive deep tendon reflexes Rationale: The normal total serum calcium level ranges from 8.6 to 10 mg/dL. Hypercalcemia occurs when the total serum calcium level exceeds 10 mg/dL. Some of the key features of hypercalcemia are increased heart rate and blood pressure; bounding, full peripheral pulses; ineffective respiratory movement related to profound skeletal muscle weakness; disorientation; diminution or absence of deep tendon reflexes; increased urine output; and hypoactive bowel sounds.

Test-Taking Strategy: Specific knowledge regarding the manifestations of hypercalcemia is required to answer this question. Recalling that the heart rate and blood pressure increase will assist you in determining that bounding full peripheral pulses is a manifestation. Review the manifestations of hypercalcemia if you had difficulty with this question.

Reference: Ignatavicius, D., & Workman, M. (2010). Medical-surgical nursing: Patient-centered collaborative care (6th ed., pp. 193,194, 1619). St. Louis: Saunders.

Cognitive Ability: Analyzing

Client Needs: Physiological Integrity

Integrated Process: Nursing Process/Assessment

Content Area: Adult Health/Renal Awarded 0.0 points out of 1.0 possible points. 263.ID: 383706662 A nurse is preparing to provide information to a client who has been found to have stable angina. The nurse plans to tell the client that this type of angina: Requires surgical treatment Can be cured with medication Will eventually need to be treated with a coronary artery bypass graft Is often managed medically with medications such as calcium channel blockers and beta-blocking medications Correct Rationale: Stable angina is chest discomfort that occurs with moderate to prolonged exertion in a pattern that is familiar to the client. It is usually relieved by nitroglycerin and rest. It is often managed by medical means, such as calcium channel blockers and beta-blocking medications. Rarely does it require aggressive treatment such as surgery. The disease can be managed but not cured.

Test-Taking Strategy: Use the process of elimination. Eliminate the options that are comparable or alike in that they indicate the need for surgical intervention. To select from the remaining options, think about the pathophysiology of angina and eliminate the option that includes the word "cured.” Review the treatment of stable angina if you had difficulty with this question.

Reference: Ignatavicius, D., & Workman, M. (2010). Medical-surgical nursing: Patient-centered collaborative care (6th ed., p. 848). St. Louis: Saunders.

Cognitive Ability: Applying

Client Needs: Physiological Integrity

Integrated Process: Teaching and Learning

Content Area: Adult Health/Cardiovascular Awarded 0.0 points out of 1.0 possible points. 264.ID: 383702963 While being seen by a physician, a client complains of persistent fever, malaise, and night sweats. On physical examination, the physician palpates enlarged lymph nodes, and the client states that the nodes are painless. Hodgkin's lymphoma is suspected, and several diagnostic studies are performed. Which characteristic of this type of lymphoma does the nurse expect to note while reviewing the results of the diagnostic studies? Blast cells in the bone marrow Epstein-Barr virus in the blood Increased blood urea nitrogen (BUN) Reed-Sternberg cells on biopsy of a lymph node Correct Rationale: Hodgkin’s lymphoma is a neoplasm of lymphatic tissue. The presence of giant multinucleated cells (Reed-Sternberg cells) is the hallmark of this disease. The presence of blast cells in the bone marrow is indicative of leukemia. Viral infections have been associated with Hodgkin’s lymphoma but do not confirm the presence of the disease. An increased BUN level is indicative of a renal system disorder.

Test-Taking Strategy: Use the process of elimination. Recalling that the Reed-Sternberg cell is the defining characteristic of Hodgkin’s lymphoma will easily direct you to the correct option. Also, note the relationship between the words "lymph nodes" in the question and in the correct option. Review the clinical manifestations associated with Hodgkin’s lymphoma if you had difficulty with this question.

Reference: Ignatavicius, D., & Workman, M. (2010). Medical-surgical nursing: Patient-centered collaborative care (6th ed., p. 913). St. Louis: Saunders.

Cognitive Ability: Analyzing

Client Needs: Physiological Integrity

Integrated Process: Nursing Process/Assessment

Content Area: Adult Health/Oncology Awarded 0.0 points out of 1.0 possible points. 265.ID: 383711422 A nurse is preparing medication instructions for a client who will be taking a daily oral dose of digoxin (Lanoxin) 0.25 mg in the treatment of congestive heart failure (CHF). Which instructions should the nurse include on the list? Select all that apply. Take your pulse before taking each dose. Correct Avoid eating foods that contain potassium. Take the digoxin at the same time each day. Correct Take the digoxin with a chewable antacid to prevent nausea. If you forget to take your daily dose, double the dose on the next day. Notify the physician if you experience loss of appetite, muscle weakness, or visual disturbances. Correct Rationale: Digoxin is a cardiac glycoside that increases the force of myocardial contraction. It is used to treat CHF and to control the ventricular rate in clients with atrial fibrillation. The client is instructed to take the medication at the same time each day and to check the pulse rate before taking the medication. If the pulse rate is slower than 60 beats/min or faster than 100 beats/min, the physician is notified. The medication is not taken with an antacid, because the antacid will affect absorption of the medication. If the client forgets to take a dose, it needs to be taken as soon as remembered. The dose is never doubled. Loss of appetite, muscle weakness, and visual disturbances are signs of digoxin toxicity, and the physician is notified if any of them occurs. Hypokalemia predisposes a client to digoxin toxicity. A client is not instructed to avoid foods that contain potassium unless specifically instructed to do so by the physician.

Test-Taking Strategy: Using the general guidelines associated with administering medications and recalling the instructions for the client taking digoxin will assist you in answering this question. Remembering that digoxin is a cardiac glycoside and that toxicity is a concern will also help you find the correct answers. Review client teaching points for digoxin if you had difficulty with this question.

Reference: Ignatavicius, D., & Workman, M. (2010). Medical-surgical nursing: Patient-centered collaborative care (6th ed., p. 778). St. Louis: Saunders.

Cognitive Ability: Applying

Client Needs: Physiological Integrity

Integrated Process: Teaching and Learning

Content Area: Pharmacology Awarded 0.0 points out of 1.0 possible points. [Show More]

Last updated: 1 year ago

Preview 1 out of 328 pages

Add to cart

Instant download

document-preview

Buy this document to get the full access instantly

Instant Download Access after purchase

Add to cart

Instant download

Reviews( 0 )

$26.50

Add to cart

Instant download

Can't find what you want? Try our AI powered Search

OR

REQUEST DOCUMENT
75
0

Document information


Connected school, study & course


About the document


Uploaded On

Aug 14, 2020

Number of pages

328

Written in

Seller


seller-icon
Martin Freeman

Member since 4 years

485 Documents Sold


Additional information

This document has been written for:

Uploaded

Aug 14, 2020

Downloads

 0

Views

 75

Document Keyword Tags

More From Martin Freeman

View all Martin Freeman's documents »

Recommended For You

Get more on HESI »

$26.50
What is Browsegrades

In Browsegrades, a student can earn by offering help to other student. Students can help other students with materials by upploading their notes and earn money.

We are here to help

We're available through e-mail, Twitter, Facebook, and live chat.
 FAQ
 Questions? Leave a message!

Follow us on
 Twitter

Copyright © Browsegrades · High quality services·